A short primer: The Greenhouse Effect Explained

Guest post by Steve Goddard
There is a considerable amount of misinformation propagated about the greenhouse effect by people from both sides of the debate.  The basic concepts are straightforward, as explained here.

The greenhouse effect is real.  If there were no greenhouse gases in the atmosphere, earth would be a cold place.   Compare Mars versus Venus – Mars has minimal greenhouse gas molecules in its’ atmosphere due to low atmospheric pressure, and is cold.  By contrast, Venus has a lot of greenhouse gas molecules in its’ atmosphere, and is very hot.  Temperature increases as greenhouse gas concentration increases.  These are undisputed facts.

Heat is not “trapped” by greenhouse gases.  The earth’s heat balance is maintained, as required by the laws of thermodynamics.

outgoing radiation = incoming radiation – changes in oceanic heat content

The image below from AER Research explains the radiative balance.

Radiation & Climate Slide

http://www.aer.com/scienceResearch/rc/rc.html

About 30% of the incoming shortwave radiation (SW) is reflected by clouds and from the earth’s surface.  20% is absorbed by clouds and re-emitted back into space as longwave (LW) radiation.  The other 50% reaches the earth’s surface and warms us.  All of that 50% eventually makes it back out into space as LW radiation, through intermediate processes of convection, conduction or radiation.  As greenhouse gas concentration increases, the total number of collisions with GHG molecules increases.  This makes it more difficult for LW radiation to escape.  In order to maintain equilibrium, the temperature has to increase.  Higher temperatures mean higher energies, which in turn increase the frequency of emission events.  Thus the incoming/outgoing balance is maintained.

It has been known for a long time that even a short column of air contains enough CO2 to saturate LW absorption.  This has been misinterpreted by some skeptics to mean that adding more CO2 will not increase the temperature.  That is simply not true, as higher GHG densities force the temperature up.  There is no dispute about this in the scientific community. See the graph below:

Click for larger image

As Dr. Hansen has correctly argued, increases in atmospheric temperature cause the ocean to warm up.  Thus changes the oceanic heat content become the short term imbalance in the incoming/outgoing equilibrium equation, which is not shown in the AER diagram.
The image below shows GHG absorption by altitude and wavenumber.  As you can see, there is a strong absorption band of CO2 at 600/cm.  That is what makes CO2 an important greenhouse gas.

Spectral Cooling Rates for the Mid-Latitude Summer Atmosphere

http://www.aer.com/scienceResearch/rc/m-proj/lbl_clrt_mls.html

The important greenhouse gases are: H2O, CO2, O3, N2O, CO and CH4.  The reason why the desert can get very cold at night is because of a lack of water vapor.  The same is true for Antarctica.  The extreme cold in Antarctica is due to high albedo and a lack of water vapor and clouds in the atmosphere, which results in almost all of the incoming radiation returning immediately to space.

An earth with no CO2 would be very cold.  The first few tens of PPM produce a strong warming effect, and increases after that are incremental.  It is widely agreed that a doubling of CO2 will increase atmospheric temperatures by about 1.2C, before feedbacks.  So the debate is not about the greenhouse effect, it is about the feedbacks.

Suppose that the amount of reflected SW from clouds increases from 20% to 21%?  That would cause a significant cooling effect.  Thus the ability of GCM models to model future temperatures is largely dependent on the ability to model future clouds.  Cloud modeling is acknowledged to be currently one of the weakest links in the GCMs.  Given the sensitivity to clouds, it is perhaps surprising that some high profile climate scientists are willing to claim that 6C+ temperature rises are established science.

So the bottom line is that the greenhouse effect is real.  Increasing CO2 will increase temperatures.  If you want to make a knowledgeable argument, learn about the feedbacks.  That is where the disagreement lies.

Lisa, in this house we obey the laws of thermodynamics
– Homer Simpson

Addenddum:
The GHG/stoplight analogy
Suppose that you have to drop your child at school at 8:00 and have to be at work at 8:30.  There are 10 stoplights between the school and the office.  Your electric car has a fixed maximum speed of 30MPH.  It takes exactly 30 minutes to drive there.
If the city adds another stoplight (analogous to more CO2) the only way you can make it to work on time is to run traffic lights and/or get the city to make the traffic lights more efficient at moving cars (analogous to higher temperature.)  The radiative balance has to be maintained in the atmosphere, so the outgoing radiation has a fixed amount of time to escape, regardless of how many GHG molecules it encounters.   Otherwise, Homer and your boss will be very angry at you for violating the laws of thermodynamics.
0 0 votes
Article Rating
530 Comments
Oldest
Newest Most Voted
Inline Feedbacks
View all comments
Policyguy
February 25, 2009 11:04 pm

Steve,
Here is an excerpt from your post on 2/21
“Consider the earth 14,000 years ago. CO2 levels were around 200 ppm and temperatures, at 6C below present values, were rising fast. Now consider 30,000 years ago. CO2 levels were also around 200 ppm and temperatures were also about 6C below current levels, yet at that time the earth was cooling. Exactly the same CO2 and temperature levels as 14,000 years ago, but the opposite direction of temperature change. CO2 was not the driver.
Now consider 120,000 years ago. Temperatures were higher than today and CO2 levels were relatively high at 290 ppm. Atmospheric H20 was high, and albedo was low. According to the theorists, earth should have been warming quickly. But it wasn’t – quite the opposite with temperatures cooling very quickly at that time. CO2 was not the driver.
If CO2 levels and the claimed lockstep feedbacks controlled the climate, the climate would be unstable. We would either move to a permanent ice age or turn into Venus.”
So, if not CO2, maybe its clouds? And what drives the clouds? Is there room for a solar component too? Are the GCM’s even capable of considering these variables or should we start from scratch?

David Corcoran
February 25, 2009 11:08 pm

The argument really isn’t about a slight AGW effect (few disagree over that), it’s whether Catastrophic Anthropogenic Global Warming (CAGW) will engender a runaway greenhouse effect that will turn our planet into a mirror of Venus in no time at all, geologically speaking.
For instance: Dr. Hansen says the seas will rise 25M in 91 years, I don’t know why low lying nations are stubbornly insisting on making him a liar.

Mark N
February 25, 2009 11:15 pm

Thanks, reads like a simple explanation that I could perhaps give it to ten year olds (or is there a better source of information for them?). Was wondering about H2O. I was under the impression that it is the major GHG. Thanks for your patience.

Bill Yarber
February 25, 2009 11:24 pm

Your use of Mars, Earth and Venus to prove that CO2 putis a major green house gas is totally flawed. You completely neglect their diameters and their distance from the Sun. Venus is 2/3s the distance from the Sun than earth and less than 1/2 that of Mars. Ergo, since the watts/sqm of the Sun’s out decreased with the square of the distance from the Sun, Venus gets over twice the energy that Earth gets and four times the energy Mars gets. Venus is warmer because it has a denser atmosphere and gets twice the energy Earth gets. This would be true if Venus had no CO2 in its atmosphere. Each planet has a different diameter but the difference is insignificant when compared to the impact of their individual orbit radii.
Bill

kevindick
February 25, 2009 11:26 pm

It seems like there are actually two things you need to know. You need to know the 1st order sensitivity to a doubling of C02 and you need to know the total feedback. So it’s pretty important to know whether the 1.2C number for 1st order sensitivity is right.
What do you think of Miskolczi’s work reassessing this sensitivity? My partial differential equation days are 20 years behind me, but I’ve read his paper and his argument seems superficially coherent.
I don’t feel qualified to fully judge the results though. It would be easy to sneak a specious mathematical argument by my given the current degraded state of my skills. Do you know any serious mathematical physics types that have reviewed his work and formed an opinion? Thanks.

Molon Labe
February 25, 2009 11:30 pm

“It has been known for a long time that even a short column of air contains enough CO2 to saturate LW absorption. This has been misinterpreted by some skeptics to mean that adding more CO2 will not increase the temperature. That is simply not true, as higher GHG densities force the temperature up.”
Utterly ridiculuous and contradictory. If CO2 absorption is “saturated”, adding more CO2 cannot have any effect. Period.

Jim G
February 25, 2009 11:33 pm

Can someone double check this?
It appears the maximum CO2 effect is centered at a wavenumber of 650.
wavenumber = 1/lambda (in cm)
=> @ 650cm-1, lambda = 15.38 um
lambda (in um) = 2897/T (in deg K)
=> T = 2897/lambda
= 188K, or -121F (-85C)
The graph also shows that the CO2 absorption is at high altitude. ~10mb.
At 50F (10C) lambda should be about 977 which would be in the H2O absorption region.
Am I missing something? Obviously the heat radiated from the earth’s surface isn’t going to be -85C.
Is this why there should be a CO2 hotspot in the troposphere above 100,000 ft?

Steven Goddard
February 25, 2009 11:35 pm

Mark N,
Temperatures are very sensitive to H2O levels, particularly at night. Which is why deserts get cold at night. H2O is the most important greenhouse gas.
PolicyGuy,
Good questions. It is clear from the ice core records that there are other cyclical drivers of climate which can vary temperatures by 10C or more. CO2 is higher now than it has been in recent centuries, and Dr. Hansen asserts that the recent rises will overwhelm the natural variation.
Some climate models do consider TSI changes due to solar cycles. Cloud formation is not well understood or modeled. Weather forecasting models often mispredict clouds on the same day.

Richard111
February 25, 2009 11:39 pm

I have lived many years in desert climates and well remember the temperature drop after sunset. Part of my duties was to note the relative humidity. It drops during the day and increases during the night, so much so that you could get condensation, dew, at dawn. This is what most desert creatures seem to survive on.
This is a personal observation. I would be interested to learn how heating the air would reduce the water content. As I understand, you must cool the air to get the moisture out. Just because deserts are hot and surface dry does not mean there is less moisture in the atmosphere. The air that blows in from cooler climes to replace the rising air over the desert does not lose its moisture. Deserts are dry. Usually.

Steven Goddard
February 25, 2009 11:39 pm

Molon,
What you are not considering is that LW is absorbed and emitted many times on it’s tortuous path out of the atmosphere. The more GHG, the more times it is absorbed.
Your argument is analagous to saying that one stop light has the same effect on your commute time as does 100 stop lights.

John F. Hultquist
February 25, 2009 11:51 pm

Holy Cow! Your first diagram doesn’t show any longwave radiation in the Solar spectrum which is +/-50% depending on your choice of the wave length for the boundary. Seems that might at least be mentioned.
“There is no dispute about this in the scientific community.”
“It is widely agreed that a doubling of CO2 will increase
atmospheric temperatures by about 1.2C before feedbacks.”
Neither of these is true.
David Archibald claims an increase to 620 ppm, projected by 2150, will raise the temperature by only 0.2 degrees C. (May 2007) in “The Past and Future Climate.”
“An earth with no CO2 would be very cold.”
Proof? Why should this be as H2O and the other gases you mention don’t rely on CO2 to work as GHGs.
Also, AGW is “widely” questioned and even if it is “widely agreed” that doesn’t make it true. The references to Mars and Venus don’t help. The situations are so different you set me thinking about other topics.
Is the phrase “before feedbacks” meant to imply positive feedbacks?
How do you explain the cooling after 1940 to 1977 while CO2 was increasing and with no change in that trend a warming began in 1977?
Now for the past ten years temps have not continued there rise, even decreased some, and CO2 has not stopped going up?
Most reading this will be aware of or can easily find on WUWT material questioning the straightforward and undisputed facts you present so I haven’t bothered to add sources here. If anyone needs these, say so.
Your short primer appears not just to be short in length but short in its explanatory power. Give it another try. There are many of these short explanations on the WEB so I think this one is a set-up of some kind. I can’t figure out, though, just what the purpose is.

manse42
February 26, 2009 12:00 am

I feel lost…
Isn’t the enthalpy and specific heat capacity somehow missing?
The potential heat content of the entire atmosphere above 10mb is less than 1/100 (because of lack of water vapours high heat capacity) of the potential heat content below 10 mb.
And it is above 10 mb the big CO2 effect comes is as I read the chart…

Steven Goddard
February 26, 2009 12:01 am

Richard111,
Relative humidity means just that – relative to the temperature. You can change the relative humidity without changing the absolute humidity, by heating or cooling the air. The Utah desert can have swings of 60 degrees between day and night with no changes in the very low absolute humidity. As soon as water starts condensing due to cold, it releases heat and limits further temperature drops.

Robert David Graham
February 26, 2009 12:04 am

Thanks for the post. I’ve long been frustrated by friends claiming “there is no greenhouse effect” or “man is not responsible for the extra CO2”.
I would point at that the disagreement lies in three places. The first is that computer climate “feeback” models are bogus.
The second is that the data are inadequate, such as the UHI, the “How Not To” series posted here, fertilized tree rings, etc.
The third is obvious scientific malfeasance, such as the “Hockey Stick” affair, GISS “normalized” temperatures, and so on.

Cassanders
February 26, 2009 12:09 am

Kudos
It is useful to have clarifications on this theme, and I will store this for later revisits.
Just a minor quiggle in a hurry.
When stating your fundamental desciption of the earth as a thermodynamic unit, you do (correctly) include heat content (or eventually changes in ocean heat content).
But my old and battered brain vaguely remembers that borehole reconstructions of temperature was fairly prominent a number of years ago.
I assume borhole reconstruction of temperature is based on models wehre rock also have thermal heat content?
Bur regardless borehole reconstructions: Does not earth (rocks + soil) have heat content (with thermal inertia ) as well?
I assume it should be closer to the properties of water than atosphere.
Cassanders
In Cod we trust

Scott Gibson
February 26, 2009 12:15 am

@Richard111
Heating the air doesn’t reduce the water content, it just reduces the relative humidity, which is a measure of moisture content compared to the dewpoint of the air at that temperature. That also explains why rising air forms clouds; as air with a certain moisture content rises, it cools to its dewpoint, and water condenses into droplets. See an article in Wikipedia for a graph showing the relationship between the dewpoint and temperature.

Scott Gibson
February 26, 2009 12:19 am

Oops, clarification: relative humidity is a measure of actual moisture content compared to the amount of moisture that can be contained at the dewpoint.

Fredrik Malmqvist
February 26, 2009 12:30 am

Note that CO2 doesn’t heat but rather act as a ”radiation blanket” . Nils Bohr showed that gas molecules that absorb light are exited to a energy level and can only give off this energy with light at the same wavelength. This light can go in any direction. Some will again hit earth and create heat. Before Bohr it was believed that gas was heated by absorbed radiation.
This effect is called “greenhouse effect” in the debate. A greenhouse is however heated because the glass stops air convection. A greenhouse build with glass transparent to all wavelength would be as good as one built with window glass. This was also shown early last century (by a physicists named Wood).

Jerry
February 26, 2009 12:33 am

I very much doubt that increases in atmospheric temperature cause the oceans to warm up to any significant extent, given the vastly greater thermal mass of water compared to air. Since about 72% of the Earth’s surface is water, which shows as black on IR photographs and is therefore absorbing everthing, it is fairly obvious that the oceans warm the atmosphere. We in the UK know this very well, as the North Atlantic Drift stops us having the climate of Newfoundland.
All the theorising is fine, but before going public with the results of models you have to show that the undoubtedly correct theoretical basis for some individual parts of an immensely complex real system extends to all parts of that system. In particular, if you want me to believe your predictions you must be able to show, reliably, that any model run will always reflect accurately the state of the climate at any time over known history. You are a very long way from even approaching that.

Lindsay H
February 26, 2009 12:35 am

As Dr. Hansen has correctly argued, increases in atmospheric temperature cause the ocean to warm up.
really!
I have seen no proof of this, the average ocean temperature is about 4-5 deg c
90 % of the ocean is below the thermocline with a temperature of about 3 deg c
hardly a heat sink more like a cold sink, the surface temperature varies from -5 to + 30 deg c but its effect on the average ocean temperature is small.
The climate sensativity to co2 doubling is the issue, no one disputes the absorption effect at 650 wave no, but there is dispute about the effect in the upper atmosphere.
If there was no co2 in the atmosphere what would be the temperature using the IPCC models?
an ice age ? it wouldn’t happen : The models are flawed. The average temperature might drop 2 degrees ?
Isn’t water vapor responsible for 95% of absorption, thus maintaining the planets temperature in a +- 5degree range, from ice age to ice age !

tmtisfree
February 26, 2009 12:37 am

“It has been known for a long time that even a short column of air contains enough CO2 to saturate LW absorption. This has been misinterpreted by some skeptics to mean that adding more CO2 will not increase the temperature. That is simply not true, as higher GHG densities force the temperature up.”
The 2 last sentences contradict physical laws. Reasoning only with CO²:
1/ As you said, all LW radiations are exciting a small and finite number of CO² molecules within the first meters (<10m) above the surface. That means adding more CO² molecules will not increase this finite number of excited molecules. Thus the number of excited molecules depends exclusively on the solar irradiation.
2/ Calculation and comparison of the frequencies (which can be interpreted as probabilities of occurrence) of
a) decay rate (natural broadening),
b) relative molecular movement (Doppler broadening) and of
c) molecular collision (collisionnal broadening)
show that the frequency of molecular collision is at least 4 orders of magnitude higher than the frequency of decay rate. That means that through molecular collisions and de-excitation as kinetic energy (heat measured with a thermometer), system reaches rapidly to a pseudo thermal equilibrium.
3/ The calculation of the velocity distribution in such system shows that only a very tiny part of the CO² molecules will keep enough energy to re-emit a photon with an energy corresponding to a 2.10^13 Hz frequency (or a 15 µm wavelength) to re-excite an other (CO²/H²O) molecule. The tiny part which re-radiates does so 50% upwards and 50% downwards, but as the number of non excited CO² molecules is far more higher (ie molecular system is far from being saturated), the downwards re-radiation is rapidly trapped again by CO² (or H²O) and re-thermalized by molecular collision as kinetic energy. Thus re-radiation in this frequency range plays only a very small part in the energy “trapping” process. Adding more CO² molecules just increases the probability that radiations from the surface excite CO² molecules at a lower height of the atmospheric column.
4/ Consequently, any radiation outside this frequency range will be (except for macroscopic aerosols which will absorb and re-radiate as a blackbody) transparently returned in part to earth (slightly warming it) and in part to the upper atmosphere (and thus slightly cooling it). This is this tiny re-radiation that is thought to be the GH effect accounted for the GW. Quantitatively, the total number of these low frequency photons depends only on the total number of the photons radiated by the surface, and thus only on the solar irradiation. Thus adding more CO² molecules will not change the total number of low frequency re-radiated photon towards surface, that is to say the slight warming which results of will not be modified when doubling (or more) the number of CO² molecules. As the atmosphere is transparent for these downwards photons, their number will also be independent of the height at which CO² molecules have been excited.
5/ So, what do an increase of CO² level?
As the number of CO² molecules increases, the probability that a finite number of CO² molecules is excited tends also to increase at the same height. But as almost the totality of the radiation from the surface is “trapped” within the first meters above the surface, it will not modify the total number of exited CO² molecules and have thus no effect on the total number of low frequency re-radiations towards the surface (GH effect). And because atmosphere is transparent to them, the height at which this process occurs will have no effect on the total number of this re-radiation returned to the surface.
The slight warming is therefore independent of
a) the number of CO² molecules (at the current level),
b) the height at which CO² molecules have been excited.
and is dependent only of
c) the solar irradiation.
I hope the above, which is not true for very low level of CO² (or any GH gases), makes sense.
Bye,
TMTisFree

tmtisfree
February 26, 2009 12:39 am

As an analogy, if you use a blind over a window on a sunny day, adding more blinds will not make the room any darker.
Bye,
TMTisFree

par5
February 26, 2009 12:54 am

Is all of this CO2 at ground level? Thermodynamics aside, how does CO2 force heat back to the surface of the earth? Include thermodynamics, how can there be an energy budget or a heat budget for the earth? I am neither pro or con AGW, more interested in the science…

tmtisfree
February 26, 2009 1:01 am

I forgot the main conclusion.
As the slight warming is already accounted for when temperature is measured, consequently, and given the above, the current temperature can not increase with an increase of CO² level.
Bye,
TMTisFree

Carl XVI Gustav
February 26, 2009 1:03 am

“Its’ amosphere” indeed. Your Nobel is cancelled!

Flanagan
February 26, 2009 1:09 am

Hi Steven,
I also really appreciate your effort and honesty in the process. I simply hope now that the scientific basis we’re sure of will not be endlessly disputed – continuously re-inventing the wheel is such a stupid loss of time, energy and money.
So I suppose the “big” questions you are disputing is whether the warming we are observing is due to the increase of CO2 or not, and if yes whether it is caused by human activities, am I right?

coaldust
February 26, 2009 1:12 am

David Corcoran (23:08:26) :
The argument [is] whether Catastrophic Anthropogenic Global Warming (CAGW) will engender a runaway greenhouse effect that will turn our planet into a mirror of Venus in no time at all, geologically speaking.
This is incorrect. The argument is about feedbacks, but not runaway feedback. There will clearly be no runaway feedback since CO2 has been much higher in the past, and runaway feedback that would “turn our planet into a mirror of Venus” did not occur.

John F. Hultquist
February 26, 2009 1:30 am

Scott,
That should be “relative humidity is a measure of actual H2O(gas) content compared to the maximum of H2O(gas) that can be “contained” at the temperature. The “contained” makes the atmosphere sound like a sponge, which it is not. That’s bad science, as shown in these pages:
http://fraser.cc/ follow prompts teaching, sci., met.,
and: http://www.ems.psu.edu/~fraser/Bad/BadGreenhouse.html

February 26, 2009 1:37 am

[snip- this comment is pointless and off color]

fred
February 26, 2009 1:37 am

It is frequently argued that AGW is ‘just 200 year old physics’, and this helpfully makes clear why this is not true and is in fact willfully misleading.
It is simple physics that CO2 absorbs heat, and that a gas with more CO2 warms up more than the same one with less CO2. However, what happens next is a different, independent, and more complicated question, and is about feedbacks. The atmosphere is a system, not a gas. So it is perfectly possible that the figure of 1.2C for a doubling of CO2 could be right, but that the net effect of that doubling could be zero, 1.2C or much greater. It depends how the rest of the system reacts over time to the initial rise.
It is a bit like the argument that a given car will travel 40 miles on a gallon of gas is simple physics, and comes from the energy content of the gas, which is not subject to dispute. It is true that the energy content is not subject to dispute, but how far it takes a car of this particular design may be physics, but it is not simple physics, and is not primarily determined by the energy content of the gas. The key question about the climate is whether clouds are negative or positive feedbacks. Do clouds and rain and other atmospheric phenomena amplify smallish warmings from whatever cause, or do they lessen them?
This is connected to the argument about the MWP and RWP. If we can point to previous warmings followed by coolings which were not caused by CO2, it must be more plausible that there is something which can produce cooling in response to warming. So it must be plausible there is some form of negative feedback in the climate system.
The hard thing for AGW to explain, given the hypothesis it is obliged to make about climate sensitivity and feedback, is why cooling followed the MWP and RWP. This on the face of it is inexplicable if they are right about climate sensitivity and the direction of feedback.

February 26, 2009 1:41 am

I have a problem with the suggestion that a slightly warmer air could warm the oceans on a timescale that need cause any concern.
A warmer air increases the temperature differential between air and space and so could well accelerate energy loss to space before any significant effect on the oceans could occur.
I also see difficulties with the proposed ocean skin effect which has been put forward by AGW supporters to overcome the ocean warming problem but which has not yet been proved to be effective in the real world.

February 26, 2009 1:51 am

The saturation issue and the density issue should be treated seperately in relation to a single GHG.
It is quite true that a level of approximate saturation can be reached for any single GHG but for a planetary atmosphere as a whole the total density arising from all it’s constituents is the paramount factor.
In the case of CO2 on Earth such saturation is reached well before any increase in the quantity of CO2 could have a significant effect on total density because the proportion of CO2 is so small.

Malcolm
February 26, 2009 1:57 am

I’m sorry Steve but as TMTisFree has shown the GH effect IS still poorly understood. There are a finite number of radiant particles involved in the GH effect on this planet, simply adding more GH gasses into the atmosphere does not neccessarily equate to higher temperatures. That is what the temperature record is telling us.

John Finn
February 26, 2009 2:03 am

As Dr. Hansen has correctly argued, increases in atmospheric temperature cause the ocean to warm up.
Is that what Dr Hansen has argued? His main point seems to be that ‘missing heat’, i.e. heat not yet evident in the atmosphere is atually accumulating in the oceans. I’d like to see the argument behind this since thermal emission from the atmosphere can only penetrate a few micron into the ocean ‘skin’.

lgl
February 26, 2009 2:08 am

And what happened to the LW backradiation to the surface?

Nylo
February 26, 2009 2:19 am

With “saturated”, I have always understood that all of the energy at a given wavelength that is emitted is absorbed.
The Earth is a microwave oven and CO2 is food in it. If the effect is saturated, it means all the energy provided by the microwave (i.e. Earth emissions at the right wave length) will be absorbed by the food (CO2). When we have little food, the food will get very hot. However, as it is little food (low concentration of CO2 in the atmosphere), its total heat content or the total heat content inside the microwave will not be very big. When we have a lot of food (high concentration of CO2 in the atmosphere), it won’t get very hot but, as the concentration is greater, it will have a similar effect in the overall atmospheric temperature (heat content inside the microwave). Similar? I would rather say “exactly the same”.
If a higher concentration of molecules capable of absorbing the energy had any effect in an already saturated effect, we all would put huge plates full of food in the microwave. But this is not true. We know that if we put twice as much food, we need twice as much energy to make it reach the same temperature.

Julian Flood
February 26, 2009 2:21 am

Learn about feedbacks indeed. 30% of the ocean is covered by strato-cumulus cloud, albedo about 60, while the open ocean is albedo effectively zero. How anthropic changes alter that thin cloud layer will have massive changes on the average albedo of the Earth, more than enough to change the temps.
If you look at Tinsdale’s graphs of maritime air temps you will observe an abrupt rise in temperature from 1939 to about 1943 (this effect is smeared out in the Hadley SST graphs as they’ve added a ‘correction’ which looks dubious to me) and I have postulated that the oil-spills from the Battle for the Atlantic reduced CCN numbers, lowered reflectivity during day and emmisivity by night, hence warming the upper levels of the sea. The Kriegesmarine effect is about oil, but you get the same result from surfactant. We cover the entire ocean surface every two weeks with a thin layer of pollution — you can even see it when an ice-floe melts, an oily smoothness all around the ice. That’s probably caused by phytoplankton, but I’d not be too surprised if it turns out to be caused by stuff entrained in the ice from the atmosphere over a few years.
Nozieres is researching bacterial surfactant effects on clouds. VOCALs is looking at strato-cu off the coast of Chile. I hope they’re checking the cloud particles for oil and surfactant pollution. If they find it them you read it here first: if not, please forget I said anything!
The science of feedbacks is extremely poor — this is where I draw the line when I hear the chant ‘the science is settled’. Until cloud feedbacks are intensively researched, all the hooha is just hand-waving.
JF

Ed Zuiderwijk
February 26, 2009 2:28 am

The figure shown is for summer at mid lattitudes. But it’s the winters that have become warmer between 1970 and 2000, not the summers. So I’d like to see the same diagram for winter conditions to see what the difference is.

warm puddle
February 26, 2009 2:32 am

Thanks for the post, it is certainly an issue that needs clearing up. I am wandering what literature the explanation of the greenhouse effect is based on. Is this essentially still based on the work of Arrhenius that was never validated? (Arrhenius, 1896).
I think your explanation can be best explained using hydraulics. co2 acts like rocks piled across a river, the temperature grade acts like the hydraulic grade. As the water slows to flow through the rock, water heads up behind due to the restriction, however adding more rocks past a certain point wouldnt increase the heading up of flow once it had reached equilibrium, only the first section of rock would really determine the depth (equivelent to trapped heat at the surface) and no flow would be sent backwards due to the net energy flow being downhill. Would this be a fair explanation?
I have read several papers that claim a chamber filled with co2 becomes no warmer in the sun than one with normal air, the same applies for tests using glass and polished rock saltcrystals . Doesnt the hypothesis then fail testing?
What are your thoughts on the following papers i.e. that convection is the main factor in the GE or that atmospheric mass determines the GE? http://www.ingentaconnect.com/content/mscp/ene/2003/00000014/F0020002/art00011 or http://www.springerlink.com/content/t341350850360302/?p=eaccc84960854cb0baba8d1d3860ac53&pi=2 (also see their response to the rebuttal) also http://xxx.lanl.gov/abs/0707.1161 claims to falsify the GE.
I think the practical testing issue needs to be resolved as does the contradiction of the second law of thermo dynamics beforfe I take the GE as rock solid theory.

Mark N
February 26, 2009 2:36 am

F. Hultquist
Thanks for interesting links

Nylo
February 26, 2009 2:50 am

If the effect is saturated, it means that no single 600/cm photon escapes the Earth. Then what happens to them?
A photon emitted by the Earth surface is quickly trapped by a CO2 molecule. The molecule increases its energy, or vibratory state. The most likely next thing is that the molecule emits back another photon of the same wavelength and reduces its energy, as a good absorber is a good emitter. If this happens, the new photon will be trapped by another CO2 molecule, as none can escape the earth at that wavelength: the effect is saturated. So nothing really happens: a molecule reduced its energy but another one increased it. However, the CO2 molecule can also reduce its energy by collisions with other molecules of a different kind, “passing” to them the vibratory state. Those molecules will in the end emit photons to release energy, but will do it at different wavelengths which are not saturated, and the energy escapes. In a stable situation, the ammount of energy emitted by the Earth Surface at 600/cm will be the same energy that CO2 molecules pass to other molecules by means of collisions and utterly escape the Earth at a different wavelength. In the tame it takes for the conversion to happen, CO2 molecules have an increased energy and this raises the overall atmospheric temperature.
What happens if we suddenly increase the concentration of CO2 molecules in the atmosphere? We have the same number of 600/cm photons at the origin, the Earth Surface. What happens is that each CO2 molecule traps less photons. There will be more molecules trapping energy, but each of them will trap less energy. There are more CO2 molecules but each of them has less energy than they used to, so the atmospheric temperature shouldn’t change yet. What we need to know is how this affects the speed at which heat or energy is transferred to molecules of a different kind so that the energy can be finally radiated to space. If each CO2 molecule traps less energy, they vibrate less and transfer less energy through collisions with other molecules. But at the same time, we have more molecules doing the same thing. What is more important? In which way do we transfer more energy through collisions? Is it with a few molecules colliding a lot or with a lot of molecules colliding less? For me, that is not clear at all. How is the process of finally transforming the wavelenght of the 600/cm photon faster, with a lot of CO2 or with little CO2? Do we have more 600/cm photons trapped in the atmosphere at a given time with a lot of not very energetic CO2 or with little but very energetic CO2?

Allan M
February 26, 2009 3:10 am

“Steven Goddard (23:39:50) :
Molon,
What you are not considering is that LW is absorbed and emitted many times on it’s tortuous path out of the atmosphere. The more GHG, the more times it is absorbed.
Your argument is analagous to saying that one stop light has the same effect on your commute time as does 100 stop lights.”
This is all the SAME heat, it doesn’t matter how many times it is absorbed and re-emitted. If absorption heats, then emission cools.
You cannot have your perpetual motion machine!
And, IMHO, the analogy stinks! (As most analogies do; stick to explanation.)
AM

Sandy
February 26, 2009 3:14 am

If we are heat balancing the Earth we need to know the rate of transfer of the heat from the magma through the crust. It would be a brave man who estimates the total nuclear fission power of the earth’s core and braver still to claim that it may be considered as being released uniformly over the globe.

par5
February 26, 2009 3:24 am

I have a problem with the first graph- it implies a ‘budget’. If watt/in equals watt/out, then hotter days equal colder nights. Then, the mean temp would not change but remain a constant? I understand the chemistry because it’s so easy and provable (never met a theoretical chemist), but I’m still having trouble with the physics. This is just one of the reasons that I am still a ‘fence sitter’ on this subject. I will admit to being slightly skeptical, but that is just a personal trait. There are just so many different opinions from well respected scientists on both sides, and it isn’t nice or fair to beat up on them. Anthony- great site! Sending you a donation immediately. Enjoy your day…

Johnny Honda
February 26, 2009 3:27 am

Steve,
this is the first articel und WUWT that is disappointing for me. It is not a correct description of the situation.
Most important fact: Until now, there is NO physical correct description or definition of the greenhouse effect.

“outgoing radiation = incoming radiation – changes in oceanic heat content”

Wrong, you forget the latent heat of the water dampening out from the land (land ist mostly not total dry).
You forget the heat loss by convection (wind is blowing along the earth surface and is heated up).
The latter is important! Wind is heated up at the surface of the earth, is warmed by this, is rising, and is emitting (by GHG!) heat to the space and so is cooling. With this mechanism we have cooling by Greenhouse-Gases!

“If there were no greenhouse gases in the atmosphere, earth would be a cold place”
“The other 50% reaches the earth’s surface and warms us”

OK, if we have no greenhouse gases (no water, no CO2), almost 100% of the radiation reaches the surface (no clouds! amost no absorption!)), so it will be colder on the planet??
Why is the day temperature on the moon ca. 160 °C ? No Greenhouse gases!
Another mechanism: Water vapor is rising, it condenses, and the heat is emitted (to a large extent) to space! When the water is evaporating at the sea or landsurface, heat is taken away from the surface. Result of the process: The surface is cooled by the GHG water!
Please read the work of Prof. Gerhard Gerlich! It is essential for the understanding of the situation. Why you don’t ask him for a guest article? His E-Mail: g.gerlich@tu-bs.de

tmtisfree
February 26, 2009 3:30 am

lgl (02:08:56) :
And what happened to the LW backradiation to the surface?
It simply warms slightly the surface. It is not sure if it is possible to experimentally discriminate this ‘indirect’ heat. But it is possible (and complicated) to calculate the probability of occurrence of this LW re-radiation and thus estimate the small theoretical heat added to the surface. Anyway, because atmosphere is transparent for this LW re-radiation, this slight warm is already accounted for when measuring temperature.
Bye,
TMTisFree

weatherag
February 26, 2009 4:02 am

Thus the ability of GCM models to model future temperatures is largely dependent on the ability to model future clouds. Cloud modeling is acknowledged to be currently one of the weakest links in the GCMs. Given the sensitivity to clouds, it is perhaps surprising that some high profile climate scientists are willing to claim that 6C+ temperature rises are established science.
As a skeptic and a Meteorologist, several of my friends, co-workers and I don’t deny that the greenhouse effect does not exist. We all acknowledge that the greenhouse is important. CO2 is need for plant grow and respiration. My argument is that the computer models are taken as fact. I have used computer models to make weather forecast for years. I know from experience that the models are not reality and that they frequently wrong. They are tools used to help make an educated guess as to what the weather will do the next few days. Now, I know that weather is not climate, but the principle behind the models still stands with the climate models. They use the same equations as the forecast models. Climate scientists, who claim the 6C increase temperature from the model as fact, should consider getting more forecast experience.
Dr. Hanson argument that heating the atmosphere will heat the ocean is wrong. You have to ask yourself, how you boil water. You don’t blow hot air over the surface; you heat the water from below. The atmosphere can’t be totally responsible for the increase in SST; it might increase the temperature in the first few inches of the surface, but not the entire water column. The oceans are heated by incoming solar radiation and from underwater volcanoes.
Our climate and weather are very complex systems that have several different mechanisms working together and against each other to balance the incoming and outgoing heat. Changing a very small amount of trace gas is not going to produce a very large temperature increase. You have to consider solar constant, ocean currents and SST, atmospheric circulation and the distribution of the gases throughout the atmosphere. I do agree that an increase in CO2 will increase temperature, but not to the extent that some models and other scientist suggest.

Tomcat
February 26, 2009 4:03 am

“If you want to make a knowledgeable argument, learn about the feedbacks. That is where the disagreement lies.”
I think the disagreement lies in how well are computer models at predicting that a warmer future is bad. CO2 levels have been above 1000 PPM for the vast majority of the billion+ year history of life on Earth.
Thanks,
Tomcat

Larry
February 26, 2009 4:04 am

The number of “facts” stated in this “primer” which are, in fact, assertions without basis are mind boggling. One of the most glaring is the assertion that James Hansen’s “correct” statement that the atmosphere heats the ocean is in direct contravention of statements by the late Robert Stevenson, former director of the Scripps Institute and a Climate scientist of over 50 years experience who says categorically, there is no way the atmosphere can heat the ocean!
Another one is the assertion that Antarctica is cold because of a lack of water vapor. In fact the opposite is true. There is a lack of water vapor because it is cold. If there was enough water vapor in the antarctic atmosphere it would form clouds which would block what little sunlight manages to reach it and make it colder.
I find the whole article suspect in its approach.

Frank Lansner
February 26, 2009 4:05 am

Im not convinced that the greenhouse effect is anything but miniscule.
Temperature on the different planets are clearly dependent on the distant to the sun and pressure of a given place in the atmosphere.
For instance, if you go deep enough on Jupiter and Saturn you will find temperatures higher than on Venus surface.
If you just go down to 1 atm pressure in Venus atmosphere you will see that the temperature is just 50-60 Kelvin warmer than earth, quite predictable considdering the shorter distance to the sun.
Also, take a look at Titan (Saturns moon). Titans atmosphere is clearly NOT better isolating than Saturns atmosphere. In fact, Titan is almost as cold at 1,5 atm as Uranus!
The funny thing is, that Saturn is almost 100% pure Hydrogen and Helium, only trace amounts of Greenhouse gasses. But Titan, the cold globe, has no less than 5% Greenhouse gasses! And its not just one greenhouse gas, its Methane, Ethane and many many other substances…
No. Im not convinced at all. H2O in the form of droplets (clouds) like any other types of clouds has micro phase shift between liquid and gas. THIS is effective to back-radiate. The thing is, Droplets Backraiates ALL frequencies, not like for instance CO2 that doesnt back radiate anything. It just changes the direction of heat randomly and only for a specific frequency. The thing is: As long as there is a hole in the bucket, it will not hold anything. CO2 and the greenhouse gasses does not by long shot close the hole in the bucket. Heat with the speed of may be hold back some split seconds before – via earth or otherwise – finding a frequency that the greenhouse gasses does not stop. We are talking split seconds, and thus an inferior impact on temperatures.
Im of course not sure of this, but this makes the best sence to me.

DocMartyn
February 26, 2009 4:14 am

If you put your head in the oven, and your feet in liquid nitrogen, you will on average, have a temperature of 37 degrees.
The Earth rotates, and the surface temperature changes throughout the day.
incoming radiation changes throughout the day.
Outgoing radiation changes throughout the day.
Humidity changes throughout the day.
Air pressure changes throughout the day.
Attempting to impose an equilibrium model on a steady state system is beyond simplistic, it is stupid.
A minimum equilibrium model would slice the day into 24 intervals and examine the quasi-equilibrium and examine the heat influx and efflux into heat reservoirs.
Here is a clue, the Earths surface is >70% water and under goes a phase transition on heating and cooling, as such during an energy input water turns to vapor and, during the night, it turns to a liquid.

February 26, 2009 4:16 am

The equations I have seen for atmospheric physics of doubling CO2 (from 280ppm) seem very simple, and the spread for no feedback is 0.5-1.5C in the published literature. The gain factor added for global modelling of the consequences is usually 3 – and this is the fabled water vapour feedback. Lindzen criticised this in the first IPCC assessment as unproven, and argued it could be zero if the water vapour condensed to cloud. Hence no runaway. Hansen’s models assume the gain factor is 3. These assumptions can be tested by observation. If we look at the tropospheric air temperature – it has risen (with some variability) by 0.5C for a 35% increase in GHG. The latest MSU data shows about 1C anomaly for 2008, and January 2009 maintaining that anomaly. So it would appear the atmosphere is warming as expected.
However, surface temperatures show cooling of .3C for 2007 and 2008.
Surface temperatures are not driven by atmospheric temperature, but by ocean heat content and its transfer to land (Compo & Sardeshmukh, ‘oceanic influences on recent continental warming’ Climate Diagnostics Centre, University of Colorado.)
So the oceans are the key, and as everyone points out, there is very little power in the atmospheric heat content to affect the ocean heat content – it can only happen by lessening the heat gradient and heat loss, but this is dominated not by CO2 but by H2O, and by cloud cover.
The key is clouds. And if we look at all the satellite derived cloud data we see a steady fall in low level reflective clouds from 1983 to 2006 and a rise from 2001 in mid and high level clouds that are probably more insulating than reflecting.
Data at NASA for surface Short Wave flux confirms the cloud thinning – with the anomaly at 4-6 watts/sq metre (average global) persisting for many years in periodic pulses from 1983-2000 (vaguely mirroring the 11-yr solar cycle) and contributing far greater energy than the (computed) CO2 radiative forcing of less than 1 Watt/sq metre (increment over the period).
Thus, the ‘global warming’ of 1980-2005 (a 25 year period), and without which there would be no peripatetic James Hansen, and probably no IPCC either, has clearly been driven by extra SW radiation reaching the earth’s surface due to cloud changes. These changes could be ‘claimed’ as AGW effects, but they are not predicted in the models – which have inadequate cloud components. As we know that ocean basins have low frequency oscillations (e.g. the PDO at 30 years) that are independent of CO2 concentration, but show correlations of solar magnetic cycles, we ought to suspect that the excess SW and cloud changes are part and parcel of the natural ocean cycles. Thus, they have amplified the ‘global warming’ signal, and now they are suppressing it.
However, my nearby Hadley Centre, having become an integral part of the IPCC system, only draws attention to the latter – the power of the cycles to now bring an intermission of global cooling (they don’t call it cycles, however because their models cannot replicate cycles – so they call it natural variability – implying randomness, which is more readily simulated) .
The surface temperature of the oceans is not the best guide to the warming period. As IPCC tell, 84% of the ‘warming’ is held in the oceans as heat content. Oceanographers tell that this heat is trapped in the upper layers down to about 700m, but most is in the first 200m. Interestingly, that ‘upper ocean heat content’ (best phrase to google for data sets) is concentrated in two major zones – the northern pacific and atlantic gyres, whereas the southern oceans tend to dissipate more (John Lyman and NOAA, and Hadley Centre (HADOBS) have maps. These two zones are very sensitive to shifts in cloud cover and these are timed by the respective PDO and its Atlantic equivalents (NAO at 10-20 years, and AMO at 60-100 years), and also interact with the Arctic Oscillation (about 70 years). Incidently, the whole of the Arctic melt-down is explained by the penetration of North Atlantic warm water and excess cloud over the polar ice (14% up between 1980-2000), and the 2008 turning of the trend was predictable once the PDO shifted to its cold phase in 2006/2007. The ice is melted from above by IR radiation from the excess cloud and below by warm North Atlantic water penetrating further north than usual – due to a weakened Beaufort Sea gyre associated with the warm phase of the PDO. With the PDO in cold phase, the ice will come back, as long as the North Atlantic doesn’t get warmer – and it seems to be at the end of its own warm phase.
As we know, the oceans stopped accumulating heat around 2002, at the same time as there was a step change in global cloud cover (measured as percentage increase). However, percentage may not be as important as a spatial shift (e.g. southerly shift of the jetstream). The jetstream is very sensitive to the solar cycle – both due to UV flux and it would seem, the magnetic cycle (though the two are related). During the Little Ice Age there is evidence the jetstream shifted south for some considerable period – as it did in 2007 and 2008 in the current solar low.
If significant cloud banks moved to uncover and expose the main gyres, they will lose heat.
These two ocean heat stores affect the planetary heat budget via teleconnections to other ocean basins – though Antarctica seems insulated and tends to go in the reverse direction to what happens in the northern hemisphere.
There is no direct evidence in the Long Wave data for CO2 build-up effects because the flux is so dominated by LW emissions from clouds (whether upward or downward).
Thus, the only direct evidence for causal drivers of the observed global warming and recent ‘pause’ relates to cloud cover and SW flux, ocean absorption of heat and the remix of ocean heat content. If CO2 contributes, that contribution cannot be greater than 20% (by comparision to the wattage from SW compared to the computed radiative forcing wattage) and as the computed CO2 signal is uncertain, I would say there is a range of 10-20%.
If we assume the world emission control programme halves current emissions by 2050 (very unlikely, I know), this will address only 5-10% of the driving force and hence, whatever other benefits might accrue, be ineffective in the stated aim of ‘avoiding climate chaos’.

MattN
February 26, 2009 4:49 am

Stunning. Reasic hasn’t shown up yet to tell you how wrong you are.

MarkW
February 26, 2009 4:52 am

I thought that Mars’ atmospheric pressure was about 1% the earths. And it’s atmosphere was about 100% CO2.
If so, that would give it more CO2 in it’s atmosphere than the earth has.

manse42
February 26, 2009 4:57 am

How much would a drop of boiling water heat my pint of beer?
I posted earlier

Isn’t the enthalpy and specific heat capacity somehow missing?
The potential heat content of the entire atmosphere above 10mb is less than 1/100 of the potential heat content below 10 mb.
And it is above 10 mb where the big CO2 effect is, as I read the chart…

Or is the difference of potential heat content of no importance?

Simon Evans
February 26, 2009 5:02 am

Steven – thank you for such a full answer. I agree with you entirely that climate sensitivity is the most significant area of discussion.

Johnny Honda
February 26, 2009 5:04 am

“outgoing radiation = incoming radiation – changes in oceanic heat content”
Oh, OK, I saw that you made a balance at the edge of the amtosphere. But the mechanisms that I described are correct (I hope)
And what happened to the LW backradiation to the surface?
It simply warms slightly the surface

Nooo! We must obey the rules of thermodynamics, like Homer. So also the second rule, i.e. no colder matter (atmosphere) can warm a warmer matter (earth).
Correct: The LW backradiation is lessening the net radiation of the surface

Hasse@Norway
February 26, 2009 5:04 am

Hmm, I don’t understand how CO2 radiation is going to heat the oceans considering the IR radiation is absorbed within the first 1-2 mm. Add to that the weight of the atmosphere of only 0.389% of the oceans and that only 380ppm of those 0.389% are CO2. I don’t see how that should be possible in any shorter therm period or at all…

February 26, 2009 5:09 am

OT – Sorry Guys
Federal US Budget calls for Carbon Cap and Trade as a part of Obama’s cut the deficit in half pledge by end of first term.
“Another official said the budget included hundreds of billions of dollars in revenues, starting in 2012 and going over many years, from a greenhouse gas emissions trading system, one of Obama’s key proposals to fight global warming.”
See my site for more Info and read my Dangers of Implementing Cap and Trade in a Recession post

DocWat
February 26, 2009 5:15 am

Tell me where I am wrong…
Energy arrives at the earth in units (a rose by all your other names is confusing)
This energy (neither created nor destroyed, but changing forms) hangs around for a while, then makes it’s exit into the vastness of space.
If this unit of energy (and all its companions) hangs around for a longer time, Earth gets warmer. If it exits sooner Earth gets cooler.
This is slightly complicated by the fact that the Earth generates some of its own energy. (for those who care, measured in milliwatts per meter squared) And, the number of units, in coming, changes for various (and in some cases unknown and unmeasured) reasons.
Then calculating global warming/cooling is a simple calculation in probability: What is the probability a unit of energy will hang around for a while. High probability=warming, low probability=cooling.
The unit of energy strikes a molecule and is reflected or absorbed (and changes form). It is retained or changes form again as it is sent on its merry way. The unit of energy then strikes another molecule and is reflected or absorbed. It is retained or changes form again as it is sent on its merry way, again. Etc.
How hard can this be? A little physics, a little chemistry, a little probability and a huge computer…

Mary Hinge
February 26, 2009 5:17 am

Steven Goddard (23:39:50) :
What you are not considering is that LW is absorbed and emitted many times on it’s tortuous path out of the atmosphere. The more GHG, the more times it is absorbed.

A good point often missed. Isn’t it also true that each time the LW is absorbed and emitted the wave length increases, indeed isn’t this true of all electromagnetic radiation?

John Judge
February 26, 2009 5:18 am

Steve
I take strong exception to your statement, “There is no dispute about this in the scientific community” when, from what I see, that there is some very strong dispute about this whole subject of AGW. If you are not able to acknowledge this plain fact, why should we give credence to anything else you say?
Also, Venus is hot because it is so close to the sun. Stand near to the fireplace and you’ll understand.

Allen63
February 26, 2009 5:20 am

Yes, the greenhouse effect of CO2 is real. And, CO2 seems to be rising along with temperature. That’s why Anthropogenic Global Warming is a “plausible” hypothesis — and worth studying.
Thing is, AGW via greenhouse effects as a significant factor in Earth’s climate is an “unproven” hypothesis. Moreover, even if AGW takes place, genuinely catastrophic AGW seems “implausible” as one digs deeper into all the aspects (assuming humankind can adapt to a few feet of sea level rise and substantially improved global food production).

Steven Goddard
February 26, 2009 5:23 am

Good questions coming in.
Remember that,heat is not trapped. If it were trapped, the earth would be incredibly hot. The energy remains in balance, as Homer clearly stated.
The article does mention convection, and re-radiation is equally likely in any direction (including downwards.) That is why a cloudy nights are warmer.
Heat flow is driven by temperature differences. If the atmosphere is warmer over the ocean, then less heat flows out of the ocean into the atmosphere, and the ocean warms up. And vice-versa.

jae
February 26, 2009 5:23 am

Steven Goddard:
“Temperatures are very sensitive to H2O levels, particularly at night. Which is why deserts get cold at night. H2O is the most important greenhouse gas.”
This is a myth. It is true only for high-altitude deserts. The night-time temperatures in Phoenix in the summer are WARMER than they are in the South at the same latitude. For example, the average minimum July temp. in Phoenix is 27.2 C; whereas in Birmingham, AB and Atlanta, GA the average July minimum is 20.8 (30-year averages).

jae
February 26, 2009 5:30 am

BTW, please explain to me why it gets so much hotter in deserts than in humid areas at the same elevation and latitude, if the greenhouse effect works as shown here. As noted, water is the most important greenhouse gas, and there is about 3-4 times as much of it in humid areas than in deserts. Also, explain why it almost never gets over 33 C in the humid tropics (where water vapor levels are always 4 times those in deserts); whereas 50 C is very common in deserts. Negative feedback by water is a fair explanation, IMHO.

Editor
February 26, 2009 5:39 am

Steven Goddard (23:35:29) :

Mark N,
Temperatures are very sensitive to H2O levels, particularly at night. Which is why deserts get cold at night. H2O is the most important greenhouse gas.

That’s one of the reasons. One other is that dry soil is a decent insulator so the heat flux to the surface is low. Another is that in humid areas, the heat released by dew formation significantly slows down cooling. Even when dew forms in the desert, there’s so litte water vapor to work with it cooling isn’t impeded as much as in humid areas.
There are some areas in Eastern Oregon where surrounding basalts acted like the urban heat island from hell. It did not get cold at night! With daytime temps over 100F (40C), it was not the most comfortable bicycling. Soaking our jerseys in any water we could find helped a lot though.
BTW, I’ve written a “State of the Climate” report that starts with a very similar graphic as Steven uses at the top of this article, see http://wermenh.com/climate/climate2009.pdf
I’ll make it the link from my name.

jae
February 26, 2009 5:39 am

All the greenhouse gas molecules can do is slow down (a little) the rate at which the energy leaves the atmosphere. They cannot in any way ADD heat; only the Sun can do that. One problem with all the nice little radiation cartoons is that they ignore convection, which counterbalances the slow-down caused by the radiation. It’s much more complicated than shown by the radiation cartoons.
And I’ll bet that water completely overwhelms any small effects by CO2, probably making them negligible.

Steven Goddard
February 26, 2009 5:51 am

Ric,
I did a canoe trip down the Green River in Utah about 25 years ago in July. It was too hot to sleep when you went to bed, and too cold to sleep when you woke up!

John Peter
February 26, 2009 5:52 am

Here is an interesting statement made by William Happer Cyrus Fogg Brackett Professor of Physics Princeton University to U.S. Senate Environment and Public Works Committee Senator Barbara Boxer, Chair on 25 February 2009:
http://nzclimatescience.net/images/PDFs/happer.pdf
At least some parts of the US Congress would appear to be prepared to listen to well reasoned and balanced arguments about the true state of Climate Change research and the role played by CO2.
Maybe this is part of an overall slow moving process resulting from the “braking” of global warming since 1998. Who knows? Time will show.

MattN
February 26, 2009 5:53 am

BTW, excellent post Steve. The strawman argument that all alarmist use is that we (skeptics/deniers/flat-earthers) do not believe adding CO2 increases temperature when the ENTIRE argument is in the feedbacks.

MarkW
February 26, 2009 5:53 am

Allen63 (05:20:14) :
Yes, the greenhouse effect of CO2 is real. And, CO2 seems to be rising along with temperature. That’s why Anthropogenic Global Warming is a “plausible” hypothesis — and worth studying.
————–
During the 60’s and 70’s, temperatures fell, even though CO2 was rising.
Over the last decade, temperatures fell, even though CO2 was rising.

Steven Goddard
February 26, 2009 5:55 am

Note the upwards curve from renowned skeptic Dr. Lindzen in the quadrupling graph above. Skeptical scientists do believe that increasing CO2 increases temperature. Dr. Spencer says the same thing.

Gary Palmgren
February 26, 2009 5:55 am

http://miskolczi.webs.com/
As I understand F. Miskolczi’s theory, he discovered a negative feedback term when the differential equations for radiation transport were solved with the correct finite thickness for the atmosphere. This eliminated a 20° discontinuity between the surface an the bottom of the atmosphere that was present in the old incorrect solution with the infinitely thick atmosphere. This shows the new solutions are correct.
The earth has a infinite supply of green house gases in the form of water in the oceans. The earth underwent a runaway greenhouse effect eons ago due to all this water and the temperature went up until the negative term became important and an equilibrium was reached. The earth has been existing at a balance point ever since.
According to Miskolczi, as we add CO2 to the atmosphere, the balance is maintained by a little water raining out. The optical density of the atmosphere remains constant. The amount of water in the atmosphere has decreased over the last fifty years as CO2 has risen.
http://www.climateaudit.org/phpBB3/viewtopic.php?f=4&t=556
The climate models have a built in positive feedback for CO2 warming because the models assume constant relative humidity. This is wrong by the data (and the theory.) Most important is that a runaway greenhouse effect is impossible.

Allan M
February 26, 2009 5:56 am

Steven Goddard:
“If the atmosphere is warmer over the ocean, then less heat flows out of the ocean into the atmosphere, and the ocean warms up.”
No it doesn’t. If less heat flows out of the ocean, it COOLS LESS.
Also an important effect of warm cloudy night is the reduction of convection by the denser clouds of water vapour ( The REAL greenhouse effect).

Jon H
February 26, 2009 5:58 am

If you only consider CO2, and historical surface data for the last 100 years in a model, then you get a +6C rise.
If you factor in cosmic rays, Earths relative distance from the sun, Suns relative position in the galaxy, Ocean current fluctuation, humidity, etc. You come to an answer of ???
Then you have 90% of surface stations with significant human growth and building development near them, and the data you gather is tainted, and all conclusions based on the data is suspect or outright wrong.
Will the real scientist please stand up!

schnurrp
February 26, 2009 6:02 am

jae
Why are deserts so hot?
The cooling effect of evaporation is key.

Steven Goddard
February 26, 2009 6:05 am

The AER chart does show convection as “sensible heat.”
http://en.wikipedia.org/wiki/Sensible_heat
Sensible heat is potential energy in the form of thermal energy or heat. The thermal body must have a temperature higher than its surroundings (see also latent heat). The thermal energy can be transported via conduction, convection, radiation or by a combination thereof.

February 26, 2009 6:06 am

Ich denke, dass das Umweltbewusstsein langsam besser wird. Außerdem wird die Marktlücke Umweltschutz immer grösser, da ja auch der Bedarf steigt. So nimmt die Entwicklung auch langsam einen positiven Verlauf. Desweiteren sollte man auch die Wirtschaftskriese als Chance sehen, denn wenn alte Strukturen vernichtet werden, werden neue Strukturen wachsen. Wie die Natur so will wenn etwas Neues entsteht kann um weiten besser und moderner sein. Lass die Politik nur machen, die wollen alle nur Ihr Geldwelt retten und nicht unsere Umwelt. In der Politik geht’s nur um Macht und nicht um Idealismus.
Soll kein Spam sein. Ich finde diese Seiten nur interessant.
Hier ist auch ein Tipp für euch zum Posten NEUER Rekord bei Kohlendioxidausstoß Umweltschutz im Bog
Mit nachhaltigem Gruß
Heinz

Jon H
February 26, 2009 6:10 am

weatherag “Dr. Hanson argument that heating the atmosphere will heat the ocean is wrong. You have to ask yourself, how you boil water. You don’t blow hot air over the surface; you heat the water from below.”
Yes and no. It is more efficient to heat from below, warm water rises heating the whole of the basin relatively evenly, but heating from above will also work but slower and less efficient.
The main problem with “heating the ocean” argument I find is time. The oceans volume is so large, and local, or recent environmental change will not be felt for hundreds of years. Much of the oceans liquid volume is in the same “mode” as the little ice age, and not the modern warming period.
Some theories about the little ice age is that the ocean caused us to leave it, more than the sun did. This is yet another area we do not know enough about.
Anyone know of a study about the movement in the magnetic poles and global volcanic activity? The magnetic core moving or changing it’s axis slightly or slightly more cause magma fluctuation resulting in more activity?? and can you localize where the resulting activity will be centralized due to the waves? Have we even gotten a good image of the planets core, it’s shape, etc?
Volcanic activity would also help us predict warming and cooling periods since it is a major cooling factor as well.. Just an idea.

Bill Illis
February 26, 2009 6:16 am

Nice post Steve,
As others have noted, greenhouse theory and the very greenhouse formulae themselves are incomplete because they are missing the “Time” dimension.
There never really is an energy balance between the energy coming and the energy leaving. Time is a crucial element in the system.
341 W / m2 comes in from the Sun for an average 12 hour period and during the next 12 hour period, 170 W / m2 is either reflected back to space or escapes back to space overnight.
How long does additional GHGs delay the time period that those photons of light / EM radiation stay in the Earth system before they escape to space.
The average energy held in the Earth system is only 170 W / m2 or a delay of 12 hours and increased CO2 could only delay that for a period of minutes or one hour at the outside.
As the summer season warms, perhaps an additional 0.5 W /m2 is absorbed per day and then the opposite occurs during the winter season.
As the deep oceans warm up to catch up with surface temps (which can take 800 to 1,500 years) perhaps an additional 0.1 W / m2 is absorbed per day.
Greenhouse theory needs to incorporate the Time element and we are talking about the speed of light and particle physics types of timelines here.
And the physics show that Temperatures are 0.2C to 0.3C per W / m2, not the 0.75C / W / m2 that Hansen like to quote. [that formula alone invalidates global warming theory and even GISS’ Model E incorporates only 0.32C / W / m2 in its hindcast to match the temperature record.]

February 26, 2009 6:22 am

The way that I heard this is that when the insulating effects first became understood, there was an intellectual proposal that a greenhouse could be constructed without the need for glass to make it work. Before this was tried, someone interjected the proposition that without the glass convection would release all of the gas and that it wouldn’t work. So it was never tried. When the unexpected the high temperature of Venus was discovered (it had been thought to be warm and moist like primeval earth) an explanation was needed. 3 scientists (headed by Carl Sagan) were allowed to propose that this constituted a “Runaway Greeen House effect”. Venus is hotter at its north pole than elsewhere and radiates more heat than it receives from the sun. No one knows WHY Venus is so hot. It has no glass shell around it and neither does earth.
“Science” comes from the Latin scio “I Know”–scientists only WISH that they knew–but want to convince others that they actually do.

george h.
February 26, 2009 6:22 am

Steve,
My understanding of the skeptic argument regarding saturation is this: At current CO2 concentrations, all of the available IR in the relevant bands (2.7, 4.3 and 15 µm) is already captured. This is about 8% of the whole IR spectrum, which means that 92% of the IR passes right through without being absorbed by CO2. If the entire atmosphere were composed of nothing but CO2, it would still only be able to absorb 8% of the radiant heat. So if all of the available IR in that spectrum is being captured at current concentrations or lower, then adding more CO2 to the atmosphere won’t matter a bit. Don’t claim to be expert here. This is not my field.

jae
February 26, 2009 6:30 am

Also, the “ocean effect” is pure heat storage (as is some of the atmospheric effect), and has very little to do with ghgs. Water is quite transparent to visible light, so the energy does get “trapped” by the water. Another BIG reason you cannot compare Venus and Mars to Earth. I think that most of the so-called “greenhouse effect” is simply a heat storage effect. The IR radiation is the “result” of this, not any “cause.” LTE requires that the 98% of the atmosphere that doesn’t absorb/radiate IR must be thermalized. This represents stored kinetic and potential energy. Too many folks are overly-hung up on radiation cartoons.

Mark
February 26, 2009 6:36 am

Has there ever been a study that involved a bunch of greenhouses where each had different amounts of CO2 (all other factors equal) and all subjected to the same amount of sunlight to see how the different CO2 levels affect temperature?

Peter
February 26, 2009 6:36 am

I hold no scientific credentials so my apologies if my questions seem overly simple.
I’ve been led to understand that the CO2 released by man-made activities represents about 3% of the atmospheric CO2. Can anyone attest to the accuracy of that claim?
Is there something different about man-made CO2 that gives that 3% more impact in Steve’s model?
Thanks,
Peter

Edward
February 26, 2009 6:39 am

tmtis free 00:39:45
You stated earlier: “As an analogy, if you use a blind over a window on a sunny day, adding more blinds will not make the room any darker”
I like your analogy, however I partially disagree and suggest an improvement . Think of the gases in the atmosphere as if the “shade” was cut into narrow strips. Pulling these strips down individually would block particular segments of the incoming light. The more strips you brought down the darker the room would become. Some strips would overlap while others would still allow light into the room through gaps between each other. To completely block light from coming into the room you would need to add multiple layers of shading to eliminate all of these “gaps”.
Thanks
Edward

schnurrp
February 26, 2009 6:41 am

Here is an interesting site. The author attempts to determine an upper level for global warming caused by co2 by simply observing the warming for the 20th century, assuming it was all caused by co2 increase, and projecting additional warming into the future using log functions. Since forcings are ignored as a cause they are “in” the future calculations of an upper limit.

Malcolm
February 26, 2009 6:56 am

I’m sorry, I have read this primer several times but it doesn’t work for me.
The basic problem I have is that in the real world heat is not trapped in the atmosphere, it escapes, and as a result you cannot transfer heat from a colder atmosphere to a warmer surface without the application of work. Where is the AGW engine that heats up the planet?

February 26, 2009 6:57 am

The image below shows GHG absorption by altitude and wavenumber. As you can see, there is a strong absorption band of CO2 at 600/cm. That is what makes CO2 an important greenhouse gas.
This is incorrect, the Clough & Iacono graph is of Radiational cooling, i.e. emission.

Steven Goddard
February 26, 2009 6:58 am

jae,
Phoenix is an urban heat island. I’ve driven south on I-10 out of Phoenix in the summer when the temperature dropped 20 degrees between Tempe and McCormick Ranch. That was before McCormick Ranch was part of the UHI. Southern Arizona also gets very high humidities in July and August. I remember once when the dew point was 80F after a thunderstorm.

mathman
February 26, 2009 6:58 am

I have read all of the earlier posts on this thread.
Would someone please explain to me how the absorption of carbon dioxide in the oceans is affected by
1) ocean surface temperature;
2) ocean saturation;
3) incident light levels (cloudy or not);
4) microorganism carbon dioxide uptake (plants in the ocean) as a function of carbon dioxide level, salinity, pollution, and other factors;
5) quantity of carbon dioxide currently sequestered in the ocean and in subsea deposits;
6) atmospheric (surface) concentration of carbon dioxide;
7) other ocean/air boundary effects?
What should be glaringly obvious is that we are not well-informed on the overall (intake versus output) carbon dioxide budget. I have not mentioned volcanic activity: how do you assess the contributions of volcanoes to atmospheric carbon dioxide?
The Elephant in the Room is the widely known and equally widely ignored propensity of all plant life to use carbon dioxide in metabolism.
AGW proponents propose a positive feedback phenomenon.
What if the feedback is negative and self-correcting?

John Galt
February 26, 2009 7:01 am

jae (05:23:16) :
Steven Goddard:
“Temperatures are very sensitive to H2O levels, particularly at night. Which is why deserts get cold at night. H2O is the most important greenhouse gas.”
This is a myth. It is true only for high-altitude deserts. The night-time temperatures in Phoenix in the summer are WARMER than they are in the South at the same latitude. For example, the average minimum July temp. in Phoenix is 27.2 C; whereas in Birmingham, AB and Atlanta, GA the average July minimum is 20.8 (30-year averages).

You are describing the Urban Heat Island effect (UHI).
I lived in Las Vegas for a few years. The microclimate in the Las Vegas valley differs from that of the surrounding desert. The relative humidity in the valley is much higher because of all the non-native plants, swimming pools, outdoor watering, etc., etc., that occurs in the metro area.
The urban heat island effect is primarily caused by all the concrete and asphalt which retain heat and electric motors, internal combustion engines and air conditioning exhaust which produce heat.
Your comparisons to other cities with the same latitude as Phoenix are also incorrect. Atlanta is not in a desert and does not have the same day-time highs as Phoenix.

Deanster
February 26, 2009 7:04 am

Steve …
As I understand it, much of the LW energy absorbed by CO2 is transformed in the collision process to different wavelengths. Thus, the LW energy at 600 cm will not remain a constant, but will degrade over distance, and as such, the re-emittance will not necessarily be in a band relevant to further concentrations of CO2.
Thoughts??

gary gulrud
February 26, 2009 7:18 am

“The other 50% reaches the earth’s surface and warms us.”
Where does this 341 W/m^2 come from? We are constantly told 1360 is the value.
“As Dr. Hansen has correctly argued, increases in atmospheric temperature cause the ocean to warm up.”
Help me out. By raising the temp of precip and by increased KE in conduction? What is the consequence of increased temp over 70% of the surface?

Hank
February 26, 2009 7:18 am

Two quick questions
1. Does increased CO2 also decrease the LW radiation coming from the sun from reaching the surface?
2. Does radiation absorbed at a particular frequency by a greenhouse gas get converted to heat (meaning vibrational motion at the molecular level), get reradiated or a combination of the two?

David L. Hagen
February 26, 2009 7:22 am

Fails due to omissions
Steve Goddard
Compliments on an impressive tutorial on conventional wisdom and your efforts to support it.
Unfortunately it fails due to all the major parameters that it leaves out. For starters consider:
1) Differing Reflection Feedback
The Cloud reflection feedback differs between short and long wave radiation.
See: Roy Spencer What About the Clouds, Andy?
2) Cosmic ray variations modulate clouds
See discussion about Svensmark
Center for Sun-Climate Research
“Cosmic rays” And “cosmoclimatology”
See efforts by:
Nir Shaviv

Shaviv, N. J. (2008), Using the oceans as a calorimeter to quantify the solar radiative forcing, J. Geophys. Res., 113, A11101, doi:10.1029/2007JA012989.

We find that the total radiative forcing associated with solar cycles variations is about 5 to 7 times larger than just those associated with the TSI variations, thus implying the necessary existence of an amplification mechanism, although without pointing to which one.

3) Feedback Causation
Spencer & Braswell point to a critical flaw of causation in feedback analysis.
Potential Biases in Feedback Diagnosis from Observational Data: A Simple Model Demonstration
Roy W. Spencer and William D. Braswell, Journal of Climate, Volume 21, Issue 21 (November 2008)pp. 5624–5628
3) Convection
The temperature in the troposphere is dominated by convection with a strong change in the temperature lapse rate with altitude above and below the tropospause.
As kevendick noted, Miskolczi is a world expert on radiation modeling. He has developed alternative climate theory to explain the major errors in IPCC models not fitting the temperature lapse rate data.
<a href=”http://www.landshape.org/dokuwiki/doku.php?id=introductionThe new climate theory of Dr. Ferenc Miskolczi
See also discussion on Miskolczi
at Niche Modeling
4) Precipitation
The Prof. Emeritus William Alexander shows actual precipitation trends are actually linked to sunspot cycles which IPCC models ignore. See:
Linkages between solar activity, climate predictability and water resource development
JOURNAL OF THE SOUTH AFRICAN INSTITUTION OF CIVIL ENGINEERING
Vol 49 No 2, June 2007, Pages 32–44, Paper 659
5) Ocean Cycles ENSO, PDO
Ocean cycles cause major changes in atmospheric temperature. By excluding these in climate modeling, they have been missinterpreted warming as due primarily to CO2.
E.g. see articles on PDO at ICECAP.US
And on PDO and Clouds Roy Spencer
etc.
Look forward to your incorporating these effects into your tutorial.

HasItBeen4YearsYet?
February 26, 2009 7:23 am

warming on other planets, a short overview…
http://motls.blogspot.com/2006/05/global-warming-on-jupiter.html

February 26, 2009 7:29 am

It seems that we could melt metals down (except, of course, mercury and gallium) using “the green house effect”!!. You forget that the volumetric heat capacity of air is 3,227 times less than that of water, in case of air convection takes all heat to stratosphere and there dissipates and in the case of water you have a limit: it evaporates( Remember?: The “water cycle”). Atmosphere has its limits also, then you can not suppose it having an infinite height.Have you experienced a total solar eclipse? Temperature drops inmediately.

gary gulrud
February 26, 2009 7:34 am

“this is the first articel und WUWT that is disappointing for me. It is not a correct description of the situation.’
I’m afraid I must agree. This is an “Atmospheric Science” paradigmatic presentation.

mangodscott
February 26, 2009 7:45 am

I’m likely late to the party, but has everyone signed the Global Warming Petition at http://www.oism.org/pproject/ and read the paper there ( http://www.oism.org/pproject/GWReview_OISM300.pdf ) ?
Or even better, has everyone read the paper “Falsi cation Of The Atmospheric CO2 Greenhouse Eff ects Within The Frame Of Physics” by Gerhard Gerlich and Ralf D. Tscheuschner at http://arxiv.org/PS_cache/arxiv/pdf/0707/0707.1161v2.pdf ?
These papers seem to thoroughly debunk anthropogenic global warming by CO2 emissions, unless I’m missing something. I would like to read comments on that second paper, if anyone will comment.

February 26, 2009 7:53 am

I own a large greenhouse, 28ft wide by 96 ft long by 12 ft high in the center. Please come visit me sometime, and we will go and sit in my greenhouse while you explain your correlation with the Earth’s greenhouse theory.

John Galt
February 26, 2009 7:57 am

gary gulrud (07:34:42) :
“this is the first articel und WUWT that is disappointing for me. It is not a correct description of the situation.’
I’m afraid I must agree. This is an “Atmospheric Science” paradigmatic presentation.

Can you explain what’s wrong with it?
One commenter noted it didn’t include cosmic rays, convection, precipitation, ocean cycles, etc. This post isn’t about the entire climate system, it’s about the greenhouse effect. Hence, the title “A short primer: The Greenhouse Effect Explained”. It doesn’t claim to be anything else.
Now where does the post go wrong? What is factually incorrect, and why? What’s the correct explanation of the ‘greenhouse’ effect?

Alan Siddons
February 26, 2009 8:02 am

Steve Goddard’s tutorial consists of the same old assertions. But you can’t prove a theory by simply repeating its tenets.
Let’s be clear: Electromagnetic energy has no temperature, yet it heats material bodies. So the greenhouse effect is not regarded to trap heat but IS regarded to trap and amplify radiant energy, bringing about the same result. This “trapped radiation” notion harks back to a misconception about glass enclosures, thus the name this theory goes by. It was thought that greenhouses got hot by preventing the exit of “dark radiation.” So radiant energy increased inside the enclosure until it could overpower the glass barrier such that outgoing energy was equal to incoming. Steve’s version is nothing more than a restatement of this fictitious mechanism.
In short, a false belief about greenhouses was the basis of the atmospheric theory — yet the atmospheric theory was RETAINED even after the physics was disproved. Meaning that the atmospheric theory doesn’t have an empirical leg to stand on, no analog in the real world.
The temperature inside a glass house has nothing to do with suppressing the exit of infrared. If solid, IR-reflective glass panes do nothing to amplify thermal radiation within a tight enclosure, however, how much less than nothing do a few swirling gas molecules accomplish in an open atmospheric canopy?

February 26, 2009 8:06 am

Ric your hyperlink is not working.
Am I getting the message from this thread, that among thoughtful scientific skeptics there is no clearly agreed science of the CO2 GHG efect – somewhat at odds with what Steve Goddard asserts?
What baffles me is, surely the CO2 GHG effect should be testable under lab conditions. A column of pure CO2 equivalent to the CO2 content in the air column naturally – what height would that be? then double it, halve it, wet it, microwave it with different wavelengths, etc. Has this been done? if not why not? Or is this a very naive question?

Dave
February 26, 2009 8:06 am

What about the feedback affect on vegetation?
I understand that if you put more CO2 into the atmosphere, plants grow better. And one of the things that plants can do overtime is to take CO2 out of the atmosphere.
It is one of the ways in which everything is kept in balance.

Innocentious
February 26, 2009 8:08 am

jae,
What is going on with the humid areas versus arid areas is the rate of absorption and emission of radiation. It is actually the main point of contention between everyone. In a humid area you have a much larger water content in the atmosphere. This requires more time to warm to its maximum temperature since waters absorption rate for heat is 23 times that of air. If you were to look at when a peek temperature occurs in an arid area versus a humid one you would see a time differential as the radiation takes longer to absorb in a humid area versus an arid one ( this would be considering that the atmosphere around the area was static which it isn’t but that is a different issue ) It also explains the rapid decrease in temperature once the sun is no longer as direct for arid zones where as with humid areas it takes longer to then radiate the heat away.
To be honest I think the main problem with the AGW hypothesis is that it says temperatures are going to increase. Technically what should be happening is the long run is that the temperatures should become MORE STABLE instead of the wild swings that we currently have. Much like the temperature in a humid tropic zone does not get too warm during the day or too cold during the night the same should become more true as more Carbon Dioxide, Water Vapor, and Methane are dumped into the Atmosphere.
Of course that probably would not get the same attention as were all going to die unless we so something now. Can you imagine a headline like this, “Earth’s Atmosphere to become more stable through release of greenhouse gases, Film at eleven!”
Plus what incentive is it to change your spending habits, lifestyle, etc. If all that is really going to happen is it will take longer to get warmer in the daytime and longer to cool off at night? If you actually look at most temperature records the increase of temperature is not surprisingly, at night. Which makes sense as it takes a little while longer to radiate that heat back out into space.
Now what a die hard AGW will tell you is everything I have told you is right BUT what is going to happen is that in the morning the temperature from the morning is still going to be warmer then it was the day before and thus cause run away Global warming because the next day it will warm up more and more and more. But we already have a perfect example in Humidity as to the actual effects of this process so I would suggest that this is pure conjecture without any real evidence. Could it happen or be happening… Doubtful, but I am always open to convincing evidence, which I have yet to see myself. Density of the atmosphere and how much radiation hits the planet would have a much more significant impact on the planets temperature then the STABILIZATION power of CO2 or Water Vapor.

HasItBeen4YearsYet?
February 26, 2009 8:09 am

@Adolfo Giurfa (07:29:01) :
“Have you experienced a total solar eclipse? Temperature drops inmediately.”
….unless it’s really humid.
And, it gets cold at night. In New England, at the end of Autumn, there were some days when it was in the 90’s, but at night it would go down into the mid 30’s, then back into the 90’s next day. It was also very dry that week.

jae
February 26, 2009 8:10 am

6:58:33, Steve:
OK, pick some other low-elevation desert. How about Daggett, CA, which has one of the lowest average July humidities of anywhere in the USA at . 24 % RH. The average minimum July temp. is still above Atlanta at 23.3 C. There is certainly no UHI in Daggett!!!

Ed Scott
February 26, 2009 8:12 am

Is this the Steve Goddard who is department chair, computer science and engineering at the University of Nebraska-Lincoln? If so, this is unbelievable.

Steven Goddard
February 26, 2009 8:14 am

Deanster and Mary Hinge,
Your points are partially correct. Successive reemissions tend to change the wavelength of the IR, and move into H20 absorption bands. If you assume (perhaps incorrectly) that H20 is fixed, then the effect of increased CO2 diminishes. However, some climate scientists argue that increased CO2 leads to increased atmospheric H2O, thus the need for accurate climate models.
This is not a problem which can be solved qualitatively.

February 26, 2009 8:15 am

Johnny Honda (03:27:08) :
Why is the day temperature on the moon ca. 160 °C ? No Greenhouse gases!
No gas at all, so temperature as we measure it here on Earth [Stevenson screens, etc] is meaningless. The SURFACE temperature [of the lunar soil] is 160°C.
General: the whole discussion borders on complete chaos, people stating this and that with wild abandon, all claiming it is ‘simple physics’. No amount of factual information can change all these misconceptions. The greenhouse effect is not that CO2 warms up, or that the air around the CO2 warms up, but simply that any energy CO2 absorbs [most of it coming from below] is immediately re-emitted, half up into space and lost and half downwards back to the surface. The surface thus heats up and warms the air by conduction and convection.

R Stevenson
February 26, 2009 8:15 am

If LW radiation is saturated by a short column (3600m) of air containing 350ppm of CO2. A higher density of CO2 say 700ppm will not absorb any more LW radiation and the atmosphere will not get hotter. Energy will be absorbed in a shorter distance (2000m) and heat energy will be mixed by convection. What you say above is not true.

jae
February 26, 2009 8:16 am

John Galt: see above comment, re: UHI. Look at ANY low elevation desert. Here’s a link: http://rredc.nrel.gov/solar/old_data/nsrdb/redbook/sum2/state.html
“Your comparisons to other cities with the same latitude as Phoenix are also incorrect. Atlanta is not in a desert and does not have the same day-time highs as Phoenix.”
?? What are you trying to say, here?? No, Atlanta is not a desert, and it has three times as much greenhouse gases as Phoenix, yet it’s temperatures (day, night, average) in the summer are lower. Why doesn’t the “greenhouse effect” make Atlanta warmer, especially at night?

February 26, 2009 8:16 am

As a layperson running a web site on global warming issues, I’m always seeking articles and postings that help novices with the understanding of the complex science involved with the climate. When I saw the headline of this post, I was very hopeful that I would be able to link to it. Unfortunately, after reading it twice, I’ve come to the conclusion it represents a mangled explanation of the greenhouse effect and just continues (increases) the confusion many have on this critical issue. Although your science may be correct, it is poorly explained.
With my last statement, I would recommend a re-write of the post to make it an understandable explanation, with these suggestions:
1. Remove the “feedback” issue from this post and do a Primer #2 on that topic.
2. If you are going to speak about Mars and Venus, put their unique situations in context versus Earth. The way you use them as examples, suggests there is an apple to apple to apple comparison regarding CO2.
3. Drop the “There is no dispute about this in the scientific community” attitude. When I see this, and like-phrases, an immediate B.S. alarm goes off. I especially like your use of this phrase right before you point the reader to a graph that has 3 representations from 3 different studies – the graph tells me there is a scientific dispute. Learn to state to your readers whether something is scientific theory or scientific fact (actual empirical evidence). If it is scientific theory, then it is in a state of “dispute.”
4. In reference to point #3, when you do a re-write, list those points/issues that are scientifically proven. When you are speaking to points that have not been scientifically proven, label them as such. I have great interest in reading your opinions and speculations but don’t mislead by implying certain issues are scientific fact beyond dispute.
5. Don’t ever use the phrase “As Dr. Hansen has correctly argued” or ever refer to him as a means to convince a reader, as it totally undermines any credibility that you are hoping to establish with the reader. It is well understood that he is a political advocate who will manipulate the data and “models” to embellish his arguments. Honestly, how hard would it be to find a reputable scientist and source to make your point?
C3H Editor, http://www.c3headlines.com

HasItBeen4YearsYet?
February 26, 2009 8:20 am

L. Hagen (07:22:05) :
Nice! Although, in part I think you are only confirming what he said at the end, “If you want to make a knowledgeable argument, learn about the feedbacks. That is where the disagreement lies.” (Even for the Svensmark and Shaviv stuff which aren’t direct actions since their mechanism seems to be more in modulating the “feedbacks.”)

jae
February 26, 2009 8:24 am

It is getting harder and harder to attribute a significant temperature change to CO2, since there has been no significant increase in temperature for 10-11 years, now. It looks like something else is totally overwhelming any CO2 effects. Oh, maybe it’s the SUN 🙂 See Shaviv’s latest ideas over at Lubos’ site.

Simon Evans
February 26, 2009 8:34 am

MattN (05:53:05) :
BTW, excellent post Steve. The strawman argument that all alarmist use is that we (skeptics/deniers/flat-earthers) do not believe adding CO2 increases temperature when the ENTIRE argument is in the feedbacks.
Have you read through this thread, Matt? It might be a strawman argument if aimed at you, but clearly not so in the case of many others (unless I have imagined all the resistance expressed here to the basics of SG’s article). This is what becomes so bewildering in any of the debates on this site, the fact that anyone expressing scepticism about the prevailing view here is then assailed by a multitude of mutually contradictory arguments. I wish the argument was as you suggest (or mainly at least), but it isn’t so here.

Alan Chappell
February 26, 2009 8:34 am

CNN is showing some spectacular shots of the volcano eruption in Chile, looks like a real big bang, now I wonder who is going to tax that?

Mark O
February 26, 2009 8:38 am

Anybody who uses the Earth, Mars, Venus analogy when discussing AGW without mentioning the difference in solar radiation these three bodies receives is either ignorant or trying to fool people.
Yet Mr. Goddard thought this argument was so persuasive that it is the fourth sentence in his article.

tmtisfree
February 26, 2009 8:44 am

Johnny Honda (05:04:23) :
And what happened to the LW backradiation to the surface?
It simply warms slightly the surface

Nooo! We must obey the rules of thermodynamics, like Homer. So also the second rule, i.e. no colder matter (atmosphere) can warm a warmer matter (earth).
Correct: The LW backradiation is lessening the net radiation of the surface
You are right, thanks for the correction.
george h. (06:22:56) :
My understanding of the skeptic argument regarding saturation is this: At current CO2 concentrations, all of the available IR in the relevant bands (2.7, 4.3 and 15 µm) is already captured. This is about 8% of the whole IR spectrum, which means that 92% of the IR passes right through without being absorbed by CO2. If the entire atmosphere were composed of nothing but CO2, it would still only be able to absorb 8% of the radiant heat. So if all of the available IR in that spectrum is being captured at current concentrations or lower, then adding more CO2 to the atmosphere won’t matter a bit.
This is fundamentally correct.
Bye,
TMTisFree

February 26, 2009 8:45 am

This is plain political warming. Did WUWT changed sides?

David Corcoran
February 26, 2009 8:49 am

coaldust (01:12:56) :

This is incorrect. The argument is about feedbacks, but not runaway feedback. There will clearly be no runaway feedback since CO2 has been much higher in the past, and runaway feedback that would “turn our planet into a mirror of Venus” did not occur.

You are correct that the geological record contradicts such an outcome, but believe me, it is still taught as a probable outcome on Earth. Here’s a school lecture:
Science Lecture
“The sobering warning for us is obvious: we have to be extremely concerned about processes such as burning of fossil fuels in large volumes that might (we don’t know for sure because the scientific questions are complex) have the potential to trigger a runaway greenhouse effect and produce on the Earth atmospheric conditions such as those found on Venus.”
And many scientists do continue to warn of a likely runaway greenhouse effect on Earth.
Survey Says…
Life doomed by climate woes: Top British scientist
Scientists who back CAGW rarely if ever contradict their brethren who spead fanciful, apocalyptic scenarios, and benefit from scaring the public.

Ed Fix
February 26, 2009 8:51 am

As evidenced by the graph from junkscience.org “Estimating Clear Sky Grenhouse Effect from Quadrupling CO2” there is a wide range of estimates of how large the actual greenhouse effect is. I have read Miskolczi’s paper “Greenhouse effect in semi-transparent planetary atmospheres”. Like kevindick, my most recent course in differential equations is over 20 years in the past, but I couldn’t spot any glaring errors.
The criticisms I’ve seen assert (without support, analysis, or explanation) that Miskolczi mis-applied the virial theorem or Kirchoff’s law, but what they completely fail to attempt to explain is why Eddington’s 1916 solution is better for this application than Miskolczi’s.
Any thoughts?

Steven Goddard
February 26, 2009 8:52 am

I have no affiliation with the University of Nebraska, and in fact have never been to Nebraska.

Brendy
February 26, 2009 8:53 am

Patronizing lectures from “omniscient” experts illustrate little more than their own hubris.

Steven Goddard
February 26, 2009 8:56 am

All, there are a million points which this article does not cover. It is just a first order explanation of the greenhouse effect.
Have at it!

Steven Goddard
February 26, 2009 8:58 am

Hank,
There isn’t a lot of LW arriving from the sun. Temperatures on the sun are too hot.

Barry Foster
February 26, 2009 8:59 am

Interview with James Lovelock (he os the bizarre Gaia theory) currently on BBC Radio4: He says there are a thousand climate scientists on the IPCC, and that the seas are rising at twice the level the IPCC predicted. He also says that sea level rise is a barometer on global warming and that if you google sea level rise you’ll see it’s rising.
Hmmm. Is any of that correct, or has this tired man gone off the silly end of the pier completely?
Interview will be available on BBC’s ‘Listen Again’ very soon under ‘Material World’. Listen and weep. Anyone got his address, as I’d like to send him a letter?

Joel Shore
February 26, 2009 9:01 am

Steve Goddard,
Thanks very much for this post. You and I may not agree on that much but I am very pleased to see you clearly spell out those things that there is at least some reasonable scientific uncertainty about (feedbacks) vs those things that are really silly to debate (e.g., whether CO2 is a greenhouse gas that causes a forcing of ~4W/m^2 when its concentration doubles). There are lots of commenters on this website who seem to spend their time talking about things where the science really is so settled that they don’t do themselves any favors by continuing to bring it up.
David Corcoran says:

For instance: Dr. Hansen says the seas will rise 25M in 91 years, I don’t know why low lying nations are stubbornly insisting on making him a liar.

Do you have a cite for Hansen making such a claim?
Bill Yarber says:

Your use of Mars, Earth and Venus to prove that CO2 putis a major green house gas is totally flawed. You completely neglect their diameters and their distance from the Sun.

Diameter doesn’t matter. Distance from the sun obviously does. However, compare Venus to Mercury. Mercury, with very little atmosphere has a huge range in temperatures between the side facing the sun and the side facing away but even the side facing the sun is not as hot as Venus, despite being closer.

Steven Goddard
February 26, 2009 9:01 am

If the atmosphere heats up, then the oceans will too. This is because the difference in temperature has to remain fixed to maintain equilibrium of heat flow.
I have no agenda for this article, other than the WUWT standard – to find out the truth, and get people talking and thinking. This is the #1 science blog after all.

Mike T
February 26, 2009 9:02 am

Question – what is the concentration of CO2 at the various levels in the air collumn? Seems to me that CO2 is heavier the O2 and normal air, it will be concentrated near the ground, not high up. But, according to your graph, CO2 doesn’t even begin to have an effect till you get to around 100 millibars. That translates to over 50,000 feet & I seriously doubt we have anything like even a measurement of CO2 concentration at that altitude and, if we do, I’ll bet it’s very, very low and can be shown to have little to no effect on warming the surface.

MattN
February 26, 2009 9:03 am

Simon sez:
“Have you read through this thread, Matt? It might be a strawman argument if aimed at you, but clearly not so in the case of many others (unless I have imagined all the resistance expressed here to the basics of SG’s article). ”
Yes I have. Can you point me to the posts that flat out deny that CO2 causes any warming? I’m not seeing them. I do not think any of the “regulars” on here believe that.

Hank
February 26, 2009 9:03 am

Chuckle, chuckle. Skepticism on the rampage.
A thing I always wondered about. Since an element of the greenhouse gas explanation involves the sun’s radiation penetrating the atmosphere. How does this play out in the oceans where sunlight penetrates into the ocean and must likewise re-radiate there? Isn’t there anything comparable to greenhouse gases in the ocean? Dissolved CO2 perhaps?
Isn’t it also important to explain that the greenhouse metaphor has a defect since most of the heating in a greenhouse is due to the fact that the heat can’t convect away?

February 26, 2009 9:09 am

Steve Goddard wrote:
“Heat flow is driven by temperature differences. If the atmosphere is warmer over the ocean, then less heat flows out of the ocean into the atmosphere, and the ocean warms up. And vice-versa.”
This is a juvenile way to look at the phenomenon!
The energy source here in this planet is the ocean and no one can think that air warms the ocean.
All the energy that the ocean gets every day from the Sun has to be re-radiated back to the outer space, after some processes, i.e. convection, meteorology in general and greenhouse effect, all processes that pertain to the atmosphere.
The more the atmosphere is prone to let energy pass through it, the more the ocean loses energy.
So, please, stop saying that the ocean is warmed by air.
The only correct way to express the concept is that a change in those processes pertaining to the atmosphere can change the rate of energy that is lost towards the outer space.
Moreover, since meteorology has a fundamental part in the redistribution of energy from the source to other parts of the planet (and then lost) as well as it influences the planetary albedo, only climate modelists can think that the problem is a linear one!

February 26, 2009 9:10 am

We are living in “interesting times” really. In the next NH summertime GWrs´will attack again, they will feel reinforced in their beliefs as La Nina produces droughts in the US midwest and west.

gary gulrud
February 26, 2009 9:11 am

Courtesy of George Smith from a recent thread:
“GHG molecules act as individual molecules; they don’t even know there is another like them in the whole universe. At 385 ppm, a CO2 molecule is one of 2597 molecules, and its nearest neighbor CO2 is 14 molecules distant in any direction. The next thing that happens to ANY GHG molecule, either water or CO2, is that it immediately collides with the normal atmospheric gases of Nirogen, Oqygen and Argon, and that energy from the absorbed photon becomes energy of thermal agitation of the ordinary air molecules. Only at extreme altitudes would any GHG molecule have a long enough mean free path to re-emit the photon it absorbed out of the thermal IR or solar spectrum; at lower altitudes the energy becomes thermalized in the normal atmospheric gases.”
The emissivity of CO2 at STP is 9*10^-4 or 1/1000 that of green leaves. The emissivity is directly proportional to the time required for the interaction, the emission of a photon, to occur.
Back-radiation has no practical consequence of heating the surface whatever.

Joel Shore
February 26, 2009 9:11 am

Molon Labe says:

Utterly ridiculuous and contradictory. If CO2 absorption is “saturated”, adding more CO2 cannot have any effect. Period.

Wrong for two reasons: first there are always the wings of the band where things are not saturated. Second, there is always a height in the atmosphere where the atmosphere is low enough density that this saturation no longer occurs. At the end of the day, what matters is not just if the IR is absorbed but WHERE it is absorbed…Or, more precisely, the last altitude it is absorbed before it escapes the earth because the temperature at this altitude is what will determine the rate of radiative emission int space. (Power radiated ~ T^4).
Jim G says:

Is this why there should be a CO2 hotspot in the troposphere above 100,000 ft?

If you are talking about the “hotspot” expected in the tropical troposphere then no. It is not a “CO2 hotspot” because it is expected to occur independent of the mechanism causing the warming. (See here for a comparison of the structures of GHG-caused and solar-caused warming: http://www.realclimate.org/index.php/archives/2007/12/tropical-troposphere-trends/ ) It turns out that the structure of the warming in the tropical troposphere is not determined by where the energy is absorbed but rather by moist adiabatic lapse rate theory. I.e., the atmosphere has lots of convection in it, so the temperature structure is determined in large part by this convection.

John Galt
February 26, 2009 9:13 am

jae:
I’m sorry, but I’m not really sure what you’re trying to say.
I’m saying both Phoenix and Atlanta have man-made micro-climates and both warm more during the day and cool less at night because of it.
I am also saying I don’t believe your comparison of Atlanta and Phoenix is a good example of what you’re trying to illustrate. The natural climate of those cities is very different and being at the same latitude does not make them comparable.
As to whether high-altitude deserts and low-altitude deserts cool differently at night, I made no comment. If that is really you’re point, Atlanta is not a good example to use as it not in any kind of desert.
If I misinterpreted what you’re saying, I apologize.

Steven Goddard
February 26, 2009 9:15 am

Leif,
Your explanation is suspect. If LW emission from a molecule of CO2 had only two possible destinations (space or ground) then the fact that CO2 absorption saturates in the lower two meters would imply that increased CO2 has no effect.

John Galt
February 26, 2009 9:19 am

Lucy Skywalker (08:06:25) :
Ric your hyperlink is not working.
Am I getting the message from this thread, that among thoughtful scientific skeptics there is no clearly agreed science of the CO2 GHG efect – somewhat at odds with what Steve Goddard asserts?
What baffles me is, surely the CO2 GHG effect should be testable under lab conditions. A column of pure CO2 equivalent to the CO2 content in the air column naturally – what height would that be? then double it, halve it, wet it, microwave it with different wavelengths, etc. Has this been done? if not why not? Or is this a very naive question?

All good questions, Lucy. If science can’t agree on how the greenhouse effect works, then we really don’t know much about the climate, do we?
Follow-up questions: How does increases in atmospheric CO2 affect the climate? Do lab results reflect the real world?

Steven Goddard
February 26, 2009 9:20 am

Gary Gulrud,
You wrote: Back-radiation has no practical consequence of heating the surface whatever.
I bicycle year round day and night, and am quite certain that my hands stay warmer on cloudy nights in the winter – due to back radiation from the clouds.

gary gulrud
February 26, 2009 9:21 am

“If the atmosphere heats up, then the oceans will too. This is because the difference in temperature has to remain fixed to maintain equilibrium of heat flow.”
This is manifestly not equivalent to “increases in atmospheric temperature cause the ocean to warm up.”
Now you have need of two clarifications.

February 26, 2009 9:22 am

tmtisfree (08:44:09) :
george h. (06:22:56) :
“My understanding of the skeptic argument regarding saturation is this: At current CO2 concentrations, all of the available IR in the relevant bands (2.7, 4.3 and 15 µm) is already captured. This is about 8% of the whole IR spectrum, which means that 92% of the IR passes right through without being absorbed by CO2. If the entire atmosphere were composed of nothing but CO2, it would still only be able to absorb 8% of the radiant heat. So if all of the available IR in that spectrum is being captured at current concentrations or lower, then adding more CO2 to the atmosphere won’t matter a bit.”
This is fundamentally correct.

This is apparently a correct description of the ‘skeptic argument’, however as far as the science goes it is fundamentally flawed.

R Stevenson
February 26, 2009 9:26 am

SW radiation cannot penetrate the atmosphere of Venus. The heat from the planet’s centre is conducted through the thin crust and passes by convection and radiation into the atmosphere; there is no greenhouse efect on Venus. A short column (3600m) of air (350ppm CO2) filters out the relevant wavebands from LW radiation. Increasing the CO2 concentration would not absorb more radiation, it would only shorten the absorption distance. The heat energy would mix by convection and the temperature of the atmosphere would not rise. What you have said above is incorrect.

John Galt
February 26, 2009 9:27 am

Steven Goddard (08:52:13) :
I have no affiliation with the University of Nebraska, and in fact have never been to Nebraska.

Nebraska is just like Kansas, but without all the glitz.

Roger Knights
February 26, 2009 9:30 am

OT: Bloomberg News story:
“Greenland, Antarctica Glaciers Speeding Faster Toward the Sea”
http://www.bloomberg.com/apps/news?pid=20601124&sid=aTg9EF2NtBCg&refer=home
Sample:
“Altogether, the glaciers in the West Antarctic are losing about 103 billion tons a year of ice in discharge,” he said. “This discharge from west Antarctica would add an additional 10 to 20 centimeters” to the existing UN predictions of sea level rise this century, he said.”

David Y
February 26, 2009 9:33 am

Anthony (and Steve)–Great post, and excellent discussion/contributions. While I’m not a scientist, for ‘climate hobbyist’ like me this is a really enjoyable ’roundtable’ exchange and a great example of what an outstanding forum WUWT is–and why I ping your site several times each day. Thank you!

Jim the Layman
February 26, 2009 9:33 am

Anyone happen to know what is the estimated amount of CO2 in the atmosphere and the amount that human activity puts into the atmosphere? It might be helpful to be able to put some perspective on this as it would seem that human activity would be so miniscule as to be dismissable, no?

jae
February 26, 2009 9:34 am

“I bicycle year round day and night, and am quite certain that my hands stay warmer on cloudy nights in the winter – due to back radiation from the clouds.”
Maybe it’s because water vapor has four times the thermal capacity of the rest of the air?

DR
February 26, 2009 9:35 am

Joel Shore,
Please explain this statement via Gavin Schmidt et al 2005:
http://www.osti.gov/energycitations/servlets/purl/881407-xk2Sdg/881407.PDF
“Tropospheric warming is a robust feature of climate model simulations driven by historical increases in greenhouse gases (1–3). Maximum warming is predicted to occur in the middle and upper tropical troposphere.”
Or, as in Steve McIntyre’s words, has the Team decided to “move on”? Have they abandoned the “basic planetary physics” learned in high school we’ve been told about?
Who has falsified Miskolczi’s hypothesis?
Miskolczi
http://www.landshape.org/dokuwiki/doku.php?id=introduction
Everyone, including Steve Goddard, should read it.
BTW, where are the experiments demonstrating the greenhouse effect? Also, the notion that from 1993-2003 OHC is driven by increases in GHG is supported by what first principles in physics? It appears to be a perpetuum mobile.

February 26, 2009 9:41 am

“This is apparently a correct description of the ’skeptic argument’”
Wrong again, Phil.
Dr. Roy Spencer succinctly states the AGW/CO2 skeptics’ argument:
No one has falsified the hypothesis that the observed temperatures changes are a consequence of natural variability.
Get back to us when you think you’ve falsified the theory of natural climate variability.

Simon Evans
February 26, 2009 9:41 am

MattN (09:03:00) :
Can you point me to the posts that flat out deny that CO2 causes any warming? I’m not seeing them. I do not think any of the “regulars” on here believe that.
I must have been dreaming. Thank you for your assurance that all the ‘regulars’ believe that “adding CO2 increases temperature” (as you first put it).

Steven Goddard
February 26, 2009 9:44 am

John,
I’m sure Nebraska is a lovely place, but I’m still trying to figure out where Springfield is.

tmtisfree
February 26, 2009 9:44 am

Leif Svalgaard (08:15:03) :
The greenhouse effect is not that CO2 warms up, or that the air around the CO2 warms up, but simply that any energy CO2 absorbs [most of it coming from below] is immediately re-emitted, half up into space and lost and half downwards back to the surface.
Calculation shows that collisional frequency is at least 4 order of magnitude higher than the frequency of a re-radiation. Thus the de-excitation of CO² molecules mostly occur by kinetic energy and pseudo thermalization of the molecular system. Even if re-radiation occurs, see below.
The surface thus heats up and warms the air by conduction and convection.Second law of thermodynamic prevents that the cooler atmosphere could warm the warmer surface without work. It is just not possible that surface is additionally warmed by this re-radiation.
General: the whole discussion borders on complete chaos, people stating this and that with wild abandon, all claiming it is ’simple physics’. No amount of factual information can change all these misconceptions.
What is true is that physical laws can not be broken to fit misconceptions.
Bye,
TMTisFree

HasItBeen4YearsYet?
February 26, 2009 9:49 am

p.s. – the correct term is “steady state” when the flows in and out are equal, which is virtually never, not “equilibrium.” (that was one of my prof’s pet peeves, and so I made it one of mine.)

alex verlinden
February 26, 2009 9:50 am

pardon my french, but this is just as much baloney as would be expected from Mann, or Hanssen, or any other Wall Street guru … namely trying to quantify something that is unquantifyable … trying to invent Ohm’s law where Ohm law isn’t feasable, possible, where Ohm’s law simply DOESN’T EXIST … I hardly can read it, but “70 outgoing” ? … where’s the measuring station ? .. why not 60 ? … why not 80 ? … I can understand that everybody wants to mirror their own science to Sir Newton’s F=ma, but first he was a genius, and second he tackled a very simple problem … let’s revisit William of Occam before we try to “prove” something … or Yogi Berra …

Robert Wood
February 26, 2009 9:54 am

The “Estimated Clear Sky … ” graph rather assumes a spherical horse.
The point about Mars and Venus is bogus. Given similar atmospheseres to Earth’s, MArs would still be colder and Venus hotter.

Gerald Machnee
February 26, 2009 9:56 am

Steven Goddard says: “Heat flow is driven by temperature differences. If the atmosphere is warmer over the ocean, then less heat flows out of the ocean into the atmosphere, and the ocean warms up. And vice-versa.”
From oceanographers, and I do not have the names handy, my understanding is that the oceans receive virtually all their heat from shortwave radiation from the sun, which means that the temperature of the air has little to do with ocean temperature. Oceans cool by radiating long wave radiation. This has little to do with whether the air is warm or cool above but more whether the sky is clear. Oceans in tropical areas are warmer that the polar ones because they receive more radiation. Currents rediatribute the heat as is also done in the atmosphere. Steve’s post stopped at the feedbacks. Yes, this is very little understanding of this, modelling as a result is poor, there are no accurate measurement, therefore we have a difficult time attributing what factors are causing the temperature changes. Yes, there is a greenhouse effect, but its main result is keeping a heat balance above freezing. The first parts of hreenhouse gases give the most warming, then the effect decreases logarithmically. Then there are the disputed feedbacks.

February 26, 2009 9:56 am

Leif Svalgaard (08:15:03) :
The greenhouse effect is not that CO2 warms up, or that the air around the CO2 warms up, but simply that any energy CO2 absorbs [most of it coming from below] is immediately re-emitted, half up into space and lost and half downwards back to the surface. The surface thus heats up and warms the air by conduction and convection.

Leif, the bolded statement is only true high in the atmosphere, ~stratosphere, near the surface any energy is exchanged via collisions with neighboring molecules almost immediately (timescale less than a nanosecond).

Steven Goddard
February 26, 2009 9:57 am

Increased CO2 is of course not the only factor affecting in the climate. If a huge volcanic eruption occurred, like Krakatoa, I suspect things would get a bit chilly regardless of how many Hummers and cows were belching out GHGs.
Glacial cycles vary the temperature by 10C, so one might convince themselves that there are also other factors influencing the climate. Over the last 10 years, temperatures have declined, despite all the Hummers and cows.

David Porter
February 26, 2009 9:58 am

Steven,
Would it be possible to give an explanation/interpretation to the spectral cooling rate graphic shown in your discussion above. Alternatively could you provide a reference to read. BTW the link to AER does not provide the info, as far as I can tell.
Thanks.

gary gulrud
February 26, 2009 9:59 am

“I bicycle year round day and night, and am quite certain that my hands stay warmer on cloudy nights in the winter – due to back radiation from the clouds.”
Steve, I think you may have published prematurely. Presenting the “Climate Science” POV would have been unexceptionable.
Cloudy nights in winter are often examples of thermal inversion and higher relative humidity. Another pillar here recently had a similar example. You’ve dropped convection and evaporation out of your mix, however anecdotal.
In any case, we’ll give you a mulligan.

schnurrp
February 26, 2009 10:00 am

Isn’t the effect of co2 logarithmic? It will never stop having an effect but the additional warming effect will become less and less, approaching zero but never reaching it. This is why without unprecedented “feedbacks” co2 cannot cause a “runaway”. I won’t lose any sleep over < +2.0C with the next doubling sometime in the 23rd century.

February 26, 2009 10:01 am

Smokey (09:41:07) :
“This is apparently a correct description of the ’skeptic argument’”
Wrong again, Phil.
Dr. Roy Spencer succinctly states the AGW/CO2 skeptics’ argument:
No one has falsified the hypothesis that the observed temperatures changes are a consequence of natural variability.

Really, and what relevance does that have to the ‘skeptic argument regarding saturation’?

george h.
February 26, 2009 10:03 am

Phil. (09:22:57) :
tmtisfree (08:44:09) :
george h. (06:22:56) :
“My understanding of the skeptic argument regarding saturation is this: At current CO2 concentrations, all of the available IR in the relevant bands (2.7, 4.3 and 15 µm) is already captured. This is about 8% of the whole IR spectrum, which means that 92% of the IR passes right through without being absorbed by CO2. If the entire atmosphere were composed of nothing but CO2, it would still only be able to absorb 8% of the radiant heat. So if all of the available IR in that spectrum is being captured at current concentrations or lower, then adding more CO2 to the atmosphere won’t matter a bit.”
This is fundamentally correct.
“This is apparently a correct description of the ’skeptic argument’, however as far as the science goes it is fundamentally flawed.”
Perhaps you can enlighten us all as to “how” this is “fundamentally flawed”

Steven Goddard
February 26, 2009 10:04 am

Gerald Machnee,
Cold air over ocean causes fog, which blocks LW. During the miserably cold California summer of 1998, I remember that we were lucky to get an hour of sunshine at noon on the beaches south of Santa Cruz. I also remember bundling up in several blankets to watch the fireworks.
“The coldest winter I ever spent was my summer in San Francisco”
Mark Twain

Simon Evans
February 26, 2009 10:06 am

Steven Goddard (09:57:16) :
Glacial cycles vary the temperature by 10C
To clarify, whilst the ice cores show that change in Antarctica, the global mean is judged to have been less – in the range of between 4C and 7C.

Steven Goddard
February 26, 2009 10:07 am

gary gulrud,
Cycling on a 20F cloudy night is much more comfortable on your hands than a 20F clear night. This is because of back radiation from the clouds.

Timo Hämeranta
February 26, 2009 10:08 am

Dear Steve & all, about oceanic heat uptake and Jim Hansen’s warming “in the pipeline” due to the lagged response of the oceans, pleasae see the study
Urban, Nathan M., and Klaus Keller, 2009. Complementary observational constraints on climate sensitivity. Geophys. Res. Lett., 36, L04708, doi:10.1029/2008GL036457, February 25, 2009, preprint online http://www.geosc.psu.edu/~kkeller/Urban_Keller_grl_08_submitted.pdf
Abstract
“A persistent feature of empirical climate sensitivity estimates is their heavy tailed probability distribution indicating a sizeable probability of high sensitivities. Previous studies make general claims that this upper heavy tail is an unavoidable feature of (i) the Earth system, or of (ii) limitations in our observational capabilities. Here we show that reducing the uncertainty about (i) oceanic heat uptake and (ii) aerosol climate forcing can—in principle—cut off this heavy upper tail of climate sensitivity estimates. Observations of oceanic heat uptake result in a negatively correlated joint likelihood function of climate sensitivity and ocean vertical diffusivity. This correlation is opposite to the positive correlation resulting from observations of surface air temperatures. As a result, the two observational constraints can rule out complementary regions in the climate sensitivity-vertical diffusivity space, and cut off the heavy upper tail of the marginal climate sensitivity estimate.”
The authors write e.g. as follows:
“A given surface air temperature change is consistent with either
a relatively large heating which is penetrating rapidly
into the oceans and delaying some of the surface warming
(i.e., a high climate sensitivity and a high ocean diffusivity),
or a relatively small heating which is penetrating slowly into
the oceans so the surface warming is quickly experienced
(i.e., a low climate sensitivity and a low ocean diffusivity).”
As far as I can see, this fundamental point is still unsolved, otherwise the authors needed none of these considerations.

Editor
February 26, 2009 10:10 am

Lucy Skywalker (08:06:25) :
> Ric your hyperlink is not working.
It worked for me just now from work.
http://wermenh.com/climate/climate2009.pdf
What sort of error did you get? It is a pretty big file (about a MB), I think there’s one image that is bloated, but it shouldn’t cause trouble.
http://wermenh.com/climate/index.html is small and easy on browsers, does that fail too?
I have had some trouble with various ISPs blacklisting the ISP that is hosting wermenh.com, but those have been mainly Email issues, not HTTP issues.

Am I getting the message from this thread, that among thoughtful scientific skeptics there is no clearly agreed science of the CO2 GHG effect – somewhat at odds with what Steve Goddard asserts?
What baffles me is, surely the CO2 GHG effect should be testable under lab conditions. A column of pure CO2 equivalent to the CO2 content in the air column naturally – what height would that be? then double it, halve it, wet it, microwave it with different wavelengths, etc. Has this been done? if not why not? Or is this a very naive question?

There are pressure and temperature effects that complicate things, and you want the nitrogen and oxygen around, as I believe there are energy transfer issues between CO2 and surrounding gases.
Some of that work has been done, but I think quite a long time ago and hence not done to look at everything we’d look at now.

SandyInDerby
February 26, 2009 10:13 am

This is probably OT but perhaps someone could enlighten me a bit.
From reading (traditional books as well as on-line) I am puzzled by ionizing radiation and cloud formation.
C T R Wilson developed the cloud chamber in the late 19th century. He removed dust from the chamber and still was able to create “cloud”; he fired X-Rays at the chamber and “cloud” formed. When Wilson showed strange cloud tracks to Rutherford, the chamber was used by Rutherford to study sub-atomic particles and radio-active decay. Therefore ionising e-m radiation and sub-atomic particles have the potential for cloud formation.
I have just read the supernova events are detectable in ice core through excess nitrogen oxide created as a result of ionisation of Oxygen and Nitrogen by gamma radiation.

My guess is that ionising radiation entering the atmosphere cycles in intensity over time. So if there are measurable changes in the ice record is there any correlation to climate.
I did wonder if there were any historical records of climate warming/cooling after SuperNova but all I found was unusually severe winters in 1046,1048 and 1054 in the Anglo Saxon Chronicle.

These pre-date the supernova of 1056. Neither are they an indicator of a globally cooling climate. But I did wonder if a Nova was preceded by an increase in radiation or are they “digital” in character with no prior indication?
Are there any studies of ice cores, tree rings or written records which show any cause and effect or is it too small to be measurable?

Colonel Sun
February 26, 2009 10:14 am

Mark (06:36:27 wrote :
“Has there ever been a study that involved a bunch of greenhouses where each had different amounts of CO2 (all other factors equal) and all subjected to the same amount of sunlight to see how the different CO2 levels affect temperature?”
Winner, winner, chicken dinner. Excellent question.
Or take two “aquariums” with walls of whatever material is most transparent to infrared.
One filled with normal atmosphere and one filled with pure CO2. Both partly filled with water.
Shine infrared lights on both. Any difference in the rise in and final equilibrium temperature of the atmospheres and water?
REPLY: The experiment is too simple to replicate our atmosphere – Anthony

foinavon
February 26, 2009 10:14 am

Steve, I suspect you’re illustrating part of your article with a spurious description of climate sensitivity (“Estimated Clear Sky Greenhouse Effect from Quadrupling CO2”). This seems to be a figure taken from a site entitled “Junk Science”.
Not only do these illustrations (Charnock and Shine; Kondratjew and Moskalenko; Lindzen) bear very little relationship to our current understanding of climate sensitivity, the examples chosen are dubious in themselves. Here’s why:
Charnock and Shine (+2.92 oC per quadrupling of CO2)
There are only two publications by Charnock and Shine (together) in the data base. These are both correspondences (letters) written in the magazine “Physics Today”. The letter that addresses climate sensitivity states:
“Corresponding estimates of the effect on the mean surface temperature of the Earth are much more complicated, as both Campbell and Tomkin say. But using a simple radiative convective model,1 with no other change, one finds that doubling the CO2 produces a 1.5 °C warming and removing it a 12 °C cooling. Including a simple relative humidity feedback (but no ice-albedo feedback) changes these values to 2.4 °C warming and 17 °C cooling”
Charnock H, Shine KP (1993) CO2s Greenhouse Contribution Debated Physics Today 4646, 66-66
So Charnock and Shine consider that doubling atmospheric CO2 results in around 2.4 oC of warming even without ice-albedo feedbacks. This equates to near 5 oC for a quadrupling of CO2…not 2.92 oC according to the Junk science picture. The albedo feedback will increase that value.
Can you help us out with this? Perusal of KP Shine’s recent publications (H Charnock focuses on ocean science at present) indicates that Shine is in the mainstream scientific view that raising atmospheric CO2 levels has problematic consequences and in fact has written recently on the fact that some of the other anthropogenic greenhouse gases are going to add significantly to the problem and need to be addressed too [e.g. K.P. Shine and W. T. Sturges (2007) CO2 Is Not the Only Gas. Science 30 March 315, 1804 – 1805]
Kondratjew and Moskalenko (+ 1.75 oC per quadrupling of CO2)
I can’t find a paper by these two in the database. I’ve tried some alternative spellings without luck. Could you please clarify the citation?
Lindzen
The only published paper of Lindzen’s I can find that addresses this point specifically is:
Schneider EK, Kirtman BP, Lindzen RS (1999) Tropospheric water vapor and climate sensitivity J. Atmos. Sci. 56, 1649-1658
But here he does a load of modelling under different parameterizations/scenarios and doesn’t come up with a particular sensitivity. Otherwise one can find “advocacy” web articles by Lindzen in which he suggests that enhanced CO2 will cause the troposphere to “dry” such that the water vapour feedback is negative (real world measurements indicate categorically that that notion is incorrect) or papers on a hypothesis about an “IR Iris Effect”.
So again it would be helpful to know where the junk science number came from!
Of course there is a very large number of analyses of climate sensitivity from modelling, from analysis of transient temperature responses to the solar cycle, from transient responses to volcanic eruptions, from analysis of paleotemperatur/paleoCO2 data during the recent and the deep past. A recent review[***] discusses around 30 of these analyses. These indicate that the climate sensitivity (equilibrium temperature rise for doubling atmospheric CO2) is likely to be between 2.0 and 4.5 oC with a most likely value around 3 oC These equate to a temperature rise from quadrupling CO2 of between 4 and 9 oC with a most likely value near 6 oC.
Of course one may not like these numbers, but they are what the scientific evidence supports. I wouldn’t use manifestly dubious junk science in any discussion of climate sensitivity.
[***]R. Knutti & G. C. Hegerl (2008) The equilibrium sensitivity of the Earth’s temperature to radiation changes Nature Geoscience 1, 735 – 743 (2008)

REPLY
: As usual, you overanalyze. The graph is simply there to show laypersons the logarithmic nature of CO2 response. And clearly with such broad disagreement, nobody has a handle on the magnitude of doubling or quadrupling CO2. – Anthony

February 26, 2009 10:17 am

David Porter (09:58:46) :
Steven,
Would it be possible to give an explanation/interpretation to the spectral cooling rate graphic shown in your discussion above. Alternatively could you provide a reference to read. BTW the link to AER does not provide the info, as far as I can tell.

It’s the radiational cooling of the atmosphere by altitude and wavenumber (& hence species).
S A Clough & M J Iacono, J Geophysical Research, vol 100, pp16519, 1995.
Note it’s not ‘GHG absorption by altitude and wavenumber’.

jae
February 26, 2009 10:18 am

“Isn’t the effect of co2 logarithmic?”
It is debatable as to whether the effect is exactly logarithmic, but it is close enough, at least for the present concentration levels. It is important to realize that the water greenhouse effect is also logarithmic (at least according to MODTRAN). This probably helps to explain my connundrum about why it’s not hotter in Atlanta than in Daggett during July–there is still a sufficient amount of water vapor in Daggett to cause most of the “greenhouse effect,” so that the extra water vapor in Atlanta doesn’t add that much. By the same token, any additional water vapor that results from a 1.2 C heating from 2 X CO2 (about 8%, by my calcs) cannot possibly cause a significant “positive water vapor feedback.”

jae
February 26, 2009 10:22 am

“The coldest winter I ever spent was my summer in San Francisco”
Mark Twain”
Yes. Your hands don’t feel so warm when you ride your bicycle in the winter, right? This is because the water vapor makes your hands much colder in winter. This is due to the fact that it has a thermal capacity about 4 times that of the rest of the air. Same reason it makes your hands feel warmer when the air is warmer. I doubt that it has anything to do with “backradiation.”

Steven Goddard
February 26, 2009 10:33 am

Gerald,
Sorry I meant to say that fog blocks SW (not LW) which is the dominant factor during the day. LW blocking is more interesting at night.
Timo,
Thanks for the timely article!
David Porter,
Spectral cooling is a tricky concept, but basically correlates with IR absorption. A reasonable explanation here:
http://books.google.com/books?id=6xUpdPOPLckC&pg=PA409&lpg=PA409&dq=spectral+cooling+rate&source=bl&ots=NnNKiOSiRA&sig=Fny7_S8C9BIAXbp6iWbOtPFesfc&hl=en&ei=SN6mSbW4OoK2sQOizvngDw&sa=X&oi=book_result&resnum=9&ct=result#PPA409,M1

February 26, 2009 10:34 am

george h. (06:22:56) :
“My understanding of the skeptic argument regarding saturation is this: At current CO2 concentrations, all of the available IR in the relevant bands (2.7, 4.3 and 15 µm) is already captured. This is about 8% of the whole IR spectrum, which means that 92% of the IR passes right through without being absorbed by CO2. If the entire atmosphere were composed of nothing but CO2, it would still only be able to absorb 8% of the radiant heat. So if all of the available IR in that spectrum is being captured at current concentrations or lower, then adding more CO2 to the atmosphere won’t matter a bit.”
Perhaps you can enlighten us all as to “how” this is “fundamentally flawed”

Since we’re talking about the earth’s outgoing IR radiation only the 15µm band is relevant, this covers approximately 25% of the radiated spectrum (not 8%). The absorption spectrum isn’t saturated (only at the line centers) so an increase in [CO2] causes an ~logarithmic response in the lower troposphere. It also ignores the fact that IR emitted from clouds passes through a higher, thinner part of the atmosphere.

tmtisfree
February 26, 2009 10:37 am

A new paper by Nir J. Shaviv entitled:
Using the oceans as a calorimeter to quantify the solar radiative forcing
Abstract:
Over the 11-year solar cycle, small changes in the total solar irradiance (TSI) give rise to small variations in the global energy budget. It was suggested, however, that different mechanisms could amplify solar activity variations to give large climatic effects, a possibility which is still a subject of debate. With this in mind, we use the oceans as a calorimeter to measure the radiative forcing variations associated with the solar cycle. This is achieved through the study of three independent records, the net heat flux into the oceans over 5 decades, the sea-level change rate based on tide gauge records over the 20th century, and the sea-surface temperature variations. Each of the records can be used to consistently derive the same oceanic heat flux. We find that the total radiative forcing associated with solar cycles variations is about 5 to 7 times larger than just those associated with the TSI variations, thus implying the necessary existence of an amplification mechanism, although without pointing to which one.
In:
Shaviv, N. J. (2008), Using the oceans as a calorimeter to quantify the solar radiative forcing, J. Geophys. Res., 113, A11101, doi:10.1029/2007JA012989.
Bye,
TMTisFree

Ray
February 26, 2009 10:39 am

If you want the full, and I mean FULL picture of the Climate Change science, check this compilation by Ken Gregory of Alberta… http://members.shaw.ca/sch25/FOS/Climate_Change_Science.html
It is very complete and full of details.

Steven Goddard
February 26, 2009 10:42 am

Simon Evans,
During the last ice age, Chicago was thousands of feet deep in ice. Summers then were undoubtedly more than 4C cooler than they are now.

Bob Shapiro
February 26, 2009 10:53 am

DocWat: That’s a good layman’s description of what’s going on. The problem in calculation comes from not knowing the specifics well enough. Knowing the specifics is 99%, and making the calculations is 1%
Steve: R.W. Wood debunked the idea that a greenhouse works by trapping radiation 100 years ago. If his experiments were wrong, please explain where his mistake was.
If he’s right, then the various posters have it right when they say: 1. More solar energy hitting the earth (higher solar flux or proximity to the sun) makes the earth warmer (and vice versa), 2. The albedo or land, sea, & ice determine how much of that energy is “usable” to warm the earth, and 3. A “thicker” atmosphere means convection energy loss takes longer, allowing the temperature to be higher.

Joel Shore
February 26, 2009 10:57 am

warm puddle says:

also http://xxx.lanl.gov/abs/0707.1161 claims to falsify the GE.
I think the practical testing issue needs to be resolved as does the contradiction of the second law of thermo dynamics beforfe I take the GE as rock solid theory.

Gerlich and Tscheuschner are wrong. This paper debunks some of what they claimed: http://lanl.arxiv.org/abs/0802.4324
I’m a physicist and their claim that the atmospheric greenhouse effect violates the 2nd law of thermodynamics is easily shown to be wrong. What the 2nd law says is that the net flow of heat would have to be from the (warmer) earth to the (cooler) upper atmosphere. G&T seem to believe that the idea that the earth is warmer with an IR-absorbing atmosphere than in its absence then implies that the net heat flow is in the other direction. However, that is not the case. The net flow is still from the earth to the atmosphere. The warming occurs because the case one is comparing to is the case when there is no IR-absorbing atmosphere and hence NONE of the radiation that the earth emits is returned to it. Thus, the fact that the atmosphere returns any of the radiative energy emitted back to the earth, even if it returns only a fraction of what it receives from the earth, is enough to cause the temperature to be higher than it would be in the absence of an IR-absorbing atmosphere.
Or, as is succintly stated here http://www.ems.psu.edu/~fraser/Bad/BadGreenhouse.html , “The surface of the Earth is warmer than it would be in the absence of an atmosphere because it receives energy from two sources: the Sun and the atmosphere.”

Joel Shore
February 26, 2009 11:04 am

Steve Goddard says:

Simon Evans,
During the last ice age, Chicago was thousands of feet deep in ice. Summers then were undoubtedly more than 4C cooler than they are now.

Global mean means just that, GLOBAL MEAN. Some places cooled by more than 4C – 7C and some places cooled less. The polar regions cooled the most and the tropics the least. There was also presumably more cooling over continents than over the oceans. There also may have been some seasonal differences although I am less sure about that.

February 26, 2009 11:06 am

I tackled this general subject a while ago and it can be found here:
http://co2sceptics.com/news.php?id=1562

HasItBeen4YearsYet?
February 26, 2009 11:07 am

http://www.wilderness-survival.net/desert-2.php

“The temperature of desert sand and rock averages 16 to 22 degrees C (30 to 40 degrees F) more than that of the air. For instance, when the air temperature is 43 degrees C (110 degrees F), the sand temperature may be 60 degrees C (140 degrees F).
Wide Temperature Range
Temperatures in arid areas may get as high as 55 degrees C during the day and as low as 10 degrees C during the night. The drop in temperature at night occurs rapidly and will chill a person who lacks warm clothing and is unable to move about. The cool evenings and nights are the best times to work or travel. If your plan is to rest at night, you will find a wool sweater, long underwear, and a wool stocking cap extremely helpful. “

Stevo
February 26, 2009 11:09 am

I’ll add yet another conflicting explanation.
The IPCC cites Held & Soden 2000 to describe the mechanism as due to the pressure difference between the average altitude at which IR radiates into space, and the surface. When air is compressed it gets hotter, which means that as it moves vertically it changes temperature, leading to the adiabatic lapse rate. As air near the surface is warmed, convection speeds up until the lapse rate is restored. The effective temperature at which the Earth radiates, and hence the temperature of the ‘top’ of the atmosphere, is fixed by the temperature at which the heat out as calculated by the Stefan Boltzmann equation is equal to the total heat in. The greenhouse effect is the difference between this top-of-atmosphere radiative temperature that would apply without atmosphere, and the surface temperature, which is hotter because of the adiabatic lapse rate. The greenhouse effect is nothing to do with “trapping” IR, it’s physical mechanism is pressure.
And the enhanced greenhouse effect is because increasing CO2 means the average height of emission is raised, so you’ve got a bigger pressure difference.
If temperature increased with altitude, then more CO2 would result in cooling. This shows even clearer that it isn’t a “trapping” effect.
And Venus works the same way, except that with 90 times more atmosphere and high altitude clouds, the emitting layer is more like 50 km up instead of only 5 km up, and so the pressure-induced temperature difference is massive. (Because of its higher albedo, Venus actually absorbs less energy from the sun than Earth, despite being closer.)
Oh, and the tropical troposphere ‘hotspot’ is a result of the water vapour feedback mechanism predicted to treble global warming. It’s absence is a blow not to the validity of CO2 greenhouse physics, but to the postulated feedbacks.
I’d like to hope that might help, but I’m afraid it will only add to the confusion.

Joel Shore
February 26, 2009 11:12 am

coaldust says:

This is incorrect. The argument is about feedbacks, but not runaway feedback. There will clearly be no runaway feedback since CO2 has been much higher in the past, and runaway feedback that would “turn our planet into a mirror of Venus” did not occur.

While I tend to agree that you are probably right, your argument is not airtight, as Hansen has pointed out. Firstly, the sun has gradually brightened over time. Secondly, Hansen argues that some negative feedbacks that operate at geologic timescales (mainly involving the GHGs themselves, I believe) could be inoperable here due to the very fast pace with which we are increasing the GHGs.
So, to summarize, the general scientific view has been that a true runaway cannot occur on earth at the moment…I.e., that the positive feedbacks are sufficient to magnify the effects of the GHGs but not enough to lead to an instability that would lead to a runaway effect. However, Hansen claims otherwise at least if we really go to town in burning all available fossil fuels…And, while I remain skeptical of his claim, I don’t think one can immediately dismiss it out-of-hand.

D. King
February 26, 2009 11:12 am

You can’t model a system with this many variables.
This is a moot debate!
Cap and Trade will be implemented.
Energy prices will go up.
“Smart” meters will be installed.
And while you back and forth on this,
…………you know what….forget it!

February 26, 2009 11:12 am

tmtisfree (09:44:54) :
The surface thus heats up and warms the air by conduction and convection.Second law of thermodynamic prevents that the cooler atmosphere could warm the warmer surface without work. It is just not possible that surface is additionally warmed by this re-radiation.
It is not the cooler air that warms the surface, but radiation that goes through the transparent air to reach the surface and be absorbed by the opaque ground.

MartinGAtkins
February 26, 2009 11:15 am

Mark N (23:15:03) :

Thanks, reads like a simple explanation that I could perhaps give it to ten year olds (or is there a better source of information for them?). Was wondering about H2O. I was under the impression that it is the major GHG. Thanks for your patience.

Unfortunately there is no simple explanation. We are dealing with atomic science and the way molecules of gas react when energised. H2O has a higher expansion ratio than CO2 when given the equivalent energy input per volume.
It follows that H2O has an exhilarated contraction when energy is lost or diluted.
At reasonably high temperatures compared with CO2 water molecules form bonds with other H2O molecules and become liquid.
CO2 doesn’t form these bonds until the energy level has fallen a long way below that of H2O.
On top of that pressure plays a part in deciding at what point both molecules start to bond.
If I’m wrong then I hope someone with someone will give a better explanation of why H2O is both a positive and negative feedback in our climate and CO2 is more or less benign.
The off shot is that H2O conserves energy in the lower troposphere but leaks huge amounts of energy in the upper troposphere.

John Galt
February 26, 2009 11:15 am

Colonel Sun (10:14:32) :
Mark (06:36:27 wrote :
“Has there ever been a study that involved a bunch of greenhouses where each had different amounts of CO2 (all other factors equal) and all subjected to the same amount of sunlight to see how the different CO2 levels affect temperature?”
Winner, winner, chicken dinner. Excellent question.
Or take two “aquariums” with walls of whatever material is most transparent to infrared.
One filled with normal atmosphere and one filled with pure CO2. Both partly filled with water.
Shine infrared lights on both. Any difference in the rise in and final equilibrium temperature of the atmospheres and water?
REPLY: The experiment is too simple to replicate our atmosphere – Anthony

I cringe everytime I hear ‘CO2 is a greenhouse gas and must be causing warming.’
Our climate system is much more complex than that, yet the entire AGW argument seems to boil down to that statement.

Brian B
February 26, 2009 11:16 am

It is widely agreed that a doubling of CO2 will increase atmospheric temperatures by about 1.2C, before feedbacks.
Is there a citation for this based on strong science, or is it just one of those givens everyone agrees with but no one can precisely say why?
Steve McIntyre has been asking for such a citation for quite awhile now and has apparently given up on ever having one provided.

stumpy
February 26, 2009 11:17 am

Is this explanation of the greenhouse effect based on the hypothesis developed by Arrhenius that was never validated (Arrhenius, 1896). I know of no successful attempts to test this hypothesis practically or theoretically, yet is seems to be wholly accepted as fact. Are there any modern works that prove a greenhouse effect as you explained exists?
The concept that extra co2 would enable more re-radiated energy to be directed back down to the earths surface has always troubled me, yes energy would be directed back on collision, but the net energy flow (warm to cold) would prevent it from going all the way back to the earths surface by collision. The re-emission argument fails when there is a net flow of energy in one direction. It is in breach of the 2nd law of thermodynamics and is essentially a perpetuem mobile of the 2nd kind. Surely radiative forcing forms only part of the GE, with atmospheric mass, stored energy and convection also being key factors.
Suggesting infrared energy that only penetrates the seas surface by 1mm can actually warm the sea is odd, especially when evaporation is considered. Leave a large glass container of water in the shade and one in the sun and see which one warms the most.

lgl
February 26, 2009 11:19 am

tmtisfree
Second law of thermodynamic does not prevent a body to be heated by radiation.
Radiation is not heat, it has no temperature.

schnurrp
February 26, 2009 11:25 am

jae (10:18:07)
What’s missing in the dry desert situation is that central pink column labeled: “sensible and latent heat” on “The annual mean global energy balance for the earth-atmosphere system” diagram. This is the energy necessary to change liquid water into gas (water vapor). This evaporative coooling has a significant effect on the ground temperature and may add to the cloud albedo as well. The lack of liquid water in the desert means that this cooling effect would be minimal.
None of this has much to do with the greenhouse effect though.

Simon Evans
February 26, 2009 11:26 am

Steven Goddard (10:42:24) :
Simon Evans,
During the last ice age, Chicago was thousands of feet deep in ice. Summers then were undoubtedly more than 4C cooler than they are now.

Local variation being much greater than the global mean, most obviously in the case of northern hemisphere glaciation. For the LGM Stenni et al. 2001 found about 9°C in Antarctica, Dahl-Jensen et al. 1998 found about 21°C in Greenland, whilst Sarnthein et al. 2003 and Kucera et al. 2005 confirm
moderate cooling of tropical SST, generally 0°C to 3.5°C, and so on. There are plenty of other studies, of course, but I suggest my 4C to 7C change in the global mean is a fair summary.

Edward
February 26, 2009 11:36 am

Barry
If you google James Lovelock the second thing you’ll find is his website.
Here it is if you want to send him a letter.
http://www.jameslovelock.org/
thanks
Edward

Steven Goddard
February 26, 2009 11:38 am

foinavon ,
This article is intended to be a qualitative overview, not quantitative. The figure just shows that noted skeptic Lindzen believes that increasing CO2 will lead to increasing temperature. As does Spencer, Pielke, etc……

Will
February 26, 2009 11:39 am

Regarding relative humidity and temperature, I’ve noticed RH will often begin to show decline preceding a temperature drop, and then as temps fall RH tracks with it. Having lived much of my time in northern Alaska temperature is an interesting subject…will it be just -20F or should we prepare for -50F. The thing I have come to watch as first sign is whether RH is starting to drop off on a cold night, and keeps dropping as temps fall, to get an idea if it will be cold (superstition?). I’ve wondered if there was such a thing as “apparent” RH when temps are low that confuses the issue. Has anyone else noticed or have an explanation for this seeming not inverse relationship between RH and temp?

aurbo
February 26, 2009 11:42 am

Re:Frank Lansner (04:05:14) :
I’m pleased to see that someone finally mentioned the most significant reason for the differences between the atmospheric temperatures at the surface of Mars and Venus relative to Earth.
Does anybody out there remember what Boyle’s Law describes? In regard to gasses, it was a staple of high-school phsyics when I went to school (over 50 years ago).
It’s PV = nRT
Since n and R are constants, the product of the variables P (pressure) and V (volume) are proportional to T (temperature). That means that if you hold the volume of a gas constant, Temperatures will rise or fall in reponse to like changes in pressure.
The temperature at the surface of Venus is about 465°C as compared to a mean temperature of the Earth of about 15°C. The surface atmospheric pressure on Venus is 92 bars vs a mean pressure of about 1 bar at the Earth’s surface. So one could relate the high surface temperature on Venus to the fact that it’s surface pressure is 92 times that on Earth.
Similarly, the the atmospheric surface pressure on Mars is 0.006 bar or about 6/1000ths the pressure on Earth. The mean surface temperatures ia bout -46°C.
The atmospheres of both Venus and Mars are about 95-97% CO2.
The constituency of the various atmospheres have a lot to do with the Planets’ gravity, a function of its mass. None of these Planets are capable of holding onto hydrogen or helium which have a low escape velocity because of their low mass. Earth’s gravitation is strong enough to hold both oxygen and nitrogen and all three Planets…Mars, Venus and Earth can hold CO2.
Incidentally, a Planet as massive as Jupiter has an atmosphere that’s essentially 100% hydrogen.
If one looks at the upper atmosphere of Venus at a height where the pressure is about the same as at the Earth’s surface….1 bar…guess what? The mean temperature at that level is about the same as on Earth.
Venus lies close enough to the sun so that solar energy is strong enough to vaporize water leaving no significant amounts of compounds in which CO2 can readily dissolve, so once created, it remains in and comprises most of the atmosphere.
As for other non GH gases like Nitrogen (N); although N is only about 3% of Venus’s atmosphere, there is actually 4 times as much N in Venus’ atmosphere as there is on Earth despite the fact that Earths atmosphere is about 79% N. It’s a matter of the density difference.
Despite the attribution by many that Venus’ high surface temperature is a result of the GH effect due to its CO2 rich atmosphere, one might ask why then isn’t Mars (with more CO2 in its atmosphere than here on Earth) warmer than it is? It’s the sun……
In summation, there are lots of other aspects of physics and thermodynamics that have yet to be fully reconciled before a conclusion that GW in general and AGW in particular is responsible for temperature stability, or even more dubious, will be reponisble in the future for Global Warming.
Steve’s primer offers nothing new to the AGW argument.

RH
February 26, 2009 11:43 am

Another exercise that I would suggest people try periodically is to use Weather Underground to compare the daily weather histories for two contrasting locations. As an example look at the record for Tamanrasset Aguenna, Algeria http://english.wunderground.com/history/airport/DAAT/2009/2/26/DailyHistory.html?req_city=NA&req_state=NA&req_statename=NA
and Mombasa, Kenya http://english.wunderground.com/history/airport/HKMO/2009/2/26/DailyHistory.html?req_city=NA&req_state=NA&req_statename=NA .
Tamanrasset Agenna has a relative humidity of around 10% and Mombasa has a relative humidity of around 94%. Notice the temperature changes between midnight and 7am at both locations. The location with highest humidity should cool the least because of the greenhouse effect. Now we must realize that there are other conditions that can affect the temperature changes between the locations, but if you do this little exercise hundreds of times like I have trying to find locations of similar elevations and alike in all respects but humidity you will begin questioning the importance of the greenhouse effect. We may not see the forest for the trees. Actually if you see this, http://www.telegraph.co.uk/earth/environment/globalwarming/4839985/Scientists-to-stop-global-warming-with-100000-square-mile-sun-shade.html , you might agree that man is rapidly moving back to the trees.

Mike Davis
February 26, 2009 11:45 am

Steven:
Steven Goddard (05:51:38) :
Ric,
I did a canoe trip down the Green River in Utah about 25 years ago in July. It was too hot to sleep when you went to bed, and too cold to sleep when you woke up!
You seem to be confusing evaporation from the river with lower humidity.
Having lived in the desert for 54 years I can say that Jae is correct.
I rode bicycles through the desert at night and new where all the underground streams were by the loss of warmth.

February 26, 2009 11:47 am

Steve Goddard,
Thank you to you and Anthony for posting this, as it does explain many things. However, I believe there is a discrepancy.
You wrote: “The radiative balance has to be maintained in the atmosphere, so the outgoing radiation has a fixed amount of time to escape, regardless of how many GHG molecules it encounters. Otherwise, Homer and your boss will be very angry at you for violating the laws of thermodynamics.”
It appears to me that the global radiative balance cannot be maintained in the atmosphere, as suggested above, if the atmosphere and oceans are to increase in temperature over time due to greenhouse gas effects.
Surely, a hotter atmosphere implies more energy per molecule, and that energy is heat. The same is true for the oceans. Thus, as long as the globe is warming, the input of energy to the earth must exceed the output.
If the GCMs have the energy input equal to the energy output, then that is another bust in the models.
One could argue that the difference is negligible, (input-output) as the earth is very large in mass. I have not run the calculations to confirm.
The first and second laws of thermodynamics require an imbalance between input and output for global warming.
To make this perfectly clear, the normal energy balance equation for a body is:
(mass IN x heat content IN) + (radiation IN) + (heat generation) =
(mass OUT x heat content OUT) + radiation OUT + (heat accumulation)
where:
mass IN and OUT are in pounds per hour, (negligible for the Earth)
heat content is in BTU per pound,
radiation is in BTU per hour,
heat generation is in BTU per hour, and
heat accumulation is in BTU per hour. (British units used)
Also, heat accumulation can be either positive or negative. Heat accumulation is required for the Earth to grow warmer.
Heat generation may be, for the Earth, nuclear power, burning fossil fuels, volcanic and geothermal effects, but not wind power or wave power.

Gibsho
February 26, 2009 11:52 am

Best discussion I’ve seen here.
Relatively free of political science and snarkiness.

Alan Siddons
February 26, 2009 11:52 am

It’s obvious from these posts that belief in this 19th century piece of nonsense is very persistent. Many people are still willing to make excuses for this nutty theory, like astrologers loading more corrective epicycles onto a Ptolemaic model. Given what’s known today about thermodynamics, however, had no one ever heard about a “greenhouse effect” before, no physicist would be foolhardy enough to propose one to explain the earth’s temperature.

Paddy
February 26, 2009 11:52 am

Please excuse my layman’s level of understanding. I am confused by conflicting statements about CO2 levels historically. Steve states that atmospheric CO2 was about 200ppm 14,000 years ago. Dr Will Happer testified before a Congressional committee today (See ICECAP “Scientist tell Congress: Earth in CO2 Famine”). He stated:
“Many people don’t realize that over geological time, we�re really in a CO2 famine now. Almost never has CO2 levels been as low as it has been in the Holocene (geologic epoch) – 280 (parts per million – ppm) – that’s unheard of. Most of the time [CO2 levels] have been at least 1000 (ppm) and it’s been quite a bit higher than that, . . .”
“Earth was just fine in those times,” Happer added. “The oceans were fine, plants grew, animals grew fie. So it’s baffling to me that we’re so frightened of getting nowhere close to where we started,” Happer explained. Happer also noted that “the number of (skeptical scientists) with the courage to speak out is growing” and he warned “children should not be force-fed propaganda, masquerading as science.” In December, Happer requested to be added to the groundbreaking U.S. Senate Minority Report Update: More Than 650 International Scientists Dissent Over Man-Made Global Warming Claims
“Happer was pressed by the Committee on whether rising CO2 fears are valid. “I don’t think the laws of nature or physics and chemistry has changed in 80 million years. 80 million years ago the Earth was a very prosperous palace and there is no reason to suddenly think it will become bad now,” Happer added. Happer is a professor in the Department of Physics at Princeton University and former Director of Energy Research at the Department of Energy from 1990 to 1993, has published over 200 scientific papers, and is a fellow of the American Physical Society, the American Association for the Advancement of Science, and the National Academy of Sciences. Happer was reportedly fired by former Vice President Al Gore in 1993 for failing to adhere to Gore’s scientific views.”
So what was the CO2 concentration 14,000 years ago? If the low end historically is 280ppm, we should be working actively to increase CO2 levels to 1000ppm.
I know that mine safety rules allow for extended exposure to 3000ppm and US Navy allows submarine air to have up to 5000ppm. This should exclude concerns of danger to animal and plant life.
It seems that that no research has been conducted to determine the optimum CO 2 level or the amount below which it becomes dangerous.
Clear up my confusion and ignorance please.

Tom
February 26, 2009 11:58 am

Can somebody tell me what’s wrong with this critical examination of the traffic light analogy:
In the traffic light analogy, the energy is driving from point A (the outside extreme of the atmosphere) to point B (the surface). The author’s point is that the commute takes longer, and since the driver spends more time driving, this equates to an increase in heat.
The problem I have with the analogy is that the energy can only go in one direction. The reality is that when the energy is absorbed (stopped at a light), and emitted (the light turns green), it can be emitted in any direction, including back out towards space. It would be as if to say that there is a 50% chance at every light that the driver will get frustrated waiting for the light to change, and turn around and leave before dropping the kid off for school. Now, at every subsequent light, in each direction, the same choice is made.
It seems to me light as you increase the number of traffic lights (CO2 molecules), the more heating you’d get in the upper atmosphere, and the less would eventually make it to the surface; and here is why – as you approach the surface, the density of the atmosphere (and therefore CO2) increases (more chance to get turned back) – as you travel away from the planet, the density decreases (less chances to get turned back).

February 26, 2009 11:58 am

Phil. (09:56:18) :
near the surface any energy is exchanged via collisions with neighboring molecules almost immediately (timescale less than a nanosecond).
Where do people get these ideas from? Googling the internet? How about some thinking?
Here goes: The heat content of the air is 7/2kT per diatomic molecule (N2 and O2, which is essentially all there is). For each CO2 molecule there are about 3000 N2 and O2 molecules. About 4% of the CO2 molecules will be thermally excited at 300 K. The vibrational frequency of the bend in the molecule is 667 cm-1, kT at 300 K is about 200 cm-1, so the relative population will be exp(-667/200). Therefore, for each excited CO2 molecule there will be 75000 O2 and N2 molecules. The total heat content of those molecules will be 7/2 kT x 75000 ~ 5*10^6 cm-1 [The unit cm-1 as an energy unit is standard for spectroscopy]. The relative amount of deposited energy per excited CO2 is thus about 0.00001of the total energy. The temperature rise per CO2 excited is therefore negligible.

Johnski
February 26, 2009 12:00 pm

Goddard you ~snip~ , when CO2 emits LW radiation it is at a frequency it cannot then reabsorb.
Cripes alive.

John Lish
February 26, 2009 12:02 pm

Steven,
the IPCC describes the forcing for a doubling of CO2 as 3.7 watts/m2. When I do the maths, that comes out at a fraction over 1 degree Kelvin. A tad lower than your claim of 1.2K.
The problem with this forcing is the still unexplained cooling period that started in the 40s through to the mid 70s. There just isn’t enough aerosol pollution to justify the masking claim by Jim Hansen and others.
However, the 3.7 watts/m2 is simply a hypothesis described by a one-shell model of the Earth’s atmosphere. If we add increased complexity to the model, the forcing for a doubling of CO2 reduces in effect. A lower forcing of say 2.3 watts/m2 gives approx. 0.6K for a doubling of CO2. That would explain more effectively the post-war dip in temperatures rather than relying on the aerosol handwaving.
There is also another logistical problem I have with the positive feedbacks claims. Atmospheric CO2 increases have a near instantaneous effect on temperature. We also know that CO2 effect on temperature is logarithmic so currently we have seen 60% of the forcing that a doubling of CO2 brings. So where are these positive feedbacks? There should be greater evidence of these occurring than the actual empirical data shows.
Also positive feedbacks build upon themselves. This stoppage in temperature rises cuts off that process. If natural variations have that effect then the positive feedbacks have to be considerably weaker than is assumed.

February 26, 2009 12:10 pm

Sir, you are a complete tool of the international banking cartel and corporate fascists. They want carbon taxes not to fix the environment but to fund global government … to enslave the labor of the world, to steal the resources of the world without competition, and to bomb nations who don’t submit to their empire. Taxes do nothing but hurt the poor.
You, like all the other tools conveniently leave out the percentage of greenhouse effect from CO2 and other greenhouse gases. CO2 is roughly 3% of the total greenhouse effect. Around 0.2% is man-made CO2. CO2 is an insignificant greenhouse gas, especially in terms of man-made contribution. Water is the only significant greenhouse gas.
Furthermore, you leave out the impact of the sun spot activity, which is thoroughly studied, and it’s impact of warming/cooling on the solar system as a whole. Futhermore, you leave out that from the data that CO2 levels tend to follow warming rather than cause warmer.
Statistics and models are lies, lies, and damned lies. Any decent college professor in scientific modeling and advanced mathematics will tell you that.
~snip~

Richard
February 26, 2009 12:11 pm

Can anyone explain how the man-made portion of atmospheric CO2 is responsible for GLOBAL temperature change? The UN estimate of man-made CO2 is: 0.000348% by volume: “Mass of Global Atmospheric Gases” chart.
http://lce.folc.ca/2008/08/03/man-made-global-co2-emissions/
Then, factoring CO2 absorbtion of IR of about 8% spectrum yields the volume of atmosphere with sensitivity to thermal absorbtion is 0.00002785. Or, what have I done wrong?

February 26, 2009 12:11 pm

Peter (06:36:58) :
I’ve been led to understand that the CO2 released by man-made activities represents about 3% of the atmospheric CO2. Can anyone attest to the accuracy of that claim?
That comes from the IPCC, Climate Change 2001.
So I won’t attest to its accuracy.

David Ermer
February 26, 2009 12:17 pm

Time spent at stop lights is ttotal = n * tlight where n is the number of stop lights.
light absorbed by a column of air is Atotal = 1-exp(-(a1*c1+a2*c2+…)*l) where a is an absorption coefficient, c is a concentration and l is the length of the column.
So this is not a good analogy. Because the absorbed light is re-emitted and re-absorbed, I’m not saying that adding more GHG to a saturated system will not result in less energy emitted to space, but adding more should have a diminishing effect.

Steven Goddard
February 26, 2009 12:18 pm

Johnski,
You are correct about the narrow band of CO2 absorption/emission frequencies, but there are other greenhouse gases in the atmosphere besides CO2, which interact with each other. If you note from the spectral diagram above, H2O absorbs nearly continuously across the spectrum. No need to get snippy.

February 26, 2009 12:19 pm

The facts that Al Gore’s carbon tax collection company doesn’t want you to see.
CO2 is roughly 3% of the total greenhouse effect. Around 0.2% is man-made CO2. CO2 is an insignificant greenhouse gas, especially in terms of man-made contribution. Water is the only significant greenhouse gas.
Furthermore, you leave out that oceans and volcanos release more CO2 than humans.
Furthermore, you leave out that CO2 is part of the life cycle.
Furthermore, you leave out the impact of the sun spot activity, which is thoroughly studied, and it’s impact of warming/cooling on the solar system as a whole.
Futhermore, you leave out that CO2 levels tend to follow warming rather than cause warmer.
Furthermore, you leave out that carbon taxes will not solve the problem but will fund global government and help big corporations to eliminate the competititon.
Furthermore, you leave out that polar bears can easily swim over 100 miles and swim as much as 300 miles. They hunt for seals and buluga whales.
Furthermore, you leave out that the ice caps are always melting and refreezing with the seasons.
Furthermore, you leave out that the solar system has a whole had been warming.
Furthermore, you leave out that the solar system and earth has cooled down due to lack of sun spot activity this past year.
Furthermore, you leave out that the government was caught making up temperature data.
Furthermore, you leave out that scientistics receive tons of funding by promoting global warming scares while skeptics don’t receive funding.
Statistics and models are lies, lies, and damned lies. Any decent college professor in scientific modeling and advanced mathematics will tell you that.

February 26, 2009 12:21 pm

delete my post, i’m entering you into a database of obvious foundation funded criminals engaging in high crimes of fraud and corruption when the people wake up to the truth and tear down the new world order to stop global enslavement and global depopulation by the international banking cartel.
~snip~ [Please, if you disagree explain why, but don’t call other commenters liars here.]

Tom
February 26, 2009 12:22 pm

I agree with Steven Goddard that it is all about feedbacks. The case against the existence of positive feedback, however, is extremely strong. Positive feedback means an internal amplification factor greater than 1, which means a self reinforcing, runaway process. Best example of such a process is a chemical – or nuclear – explosion. There are many arguments against the Earth’s climate being a system that is controlled by positive feedback.
1., Historical evidence. There is no evidence in climate history, that temperature swings are being reinforced by positive feedback leading to runaway climatic changes. The most recent examle was the 1998 El Nino event, which generated an almost 1degree C spike in global temperature in less than one year. If positive feedback were to be present, the temperature would have kept climbing. Instead, after the El Nino was over, temperatures returned to their pre El Nino values.
2., Recent work by Dr. Roy Spencer, who, based on his research, in his recent congressional testimony stated, in front of Senator Boxer, that there is no positive feedback in the ( Earth’s ) climate. I guess he was willing to risk the charge of perjury.
3., Dr Hansen’ circular argument. He is clearly worried about a Venus like runaway heating caused by positive feedback. At the same time, he is postulating that positive feedback is already the operating principle of the Earth’s climate as witnessed by his projected climate sensitivity factor of 6.5, which implies existence of a strong positive feedback. If this were the case, we are already doomed. There is no magic “tipping point” . A positive feedback driven runaway process cannot be stopped any more than a nuclear explosion can be stopped halfway through the process.

maksimovich
February 26, 2009 12:22 pm

foinavon (10:14:41) :
Kondratjew and Moskalenko (+ 1.75 oC per quadrupling of CO2)
I can’t find a paper by these two in the database. I’ve tried some alternative spellings without luck. Could you please clarify the citation?
Kondratyev and Moskalenko 1983, 1984 These are in Russian,though translated extracts are in Houghton 1985
Essentially these are radiative/convection models similar to Ramanathan 1976,and as such more accurately ascertain for dissipation eg Landau and Lifshitz 1965(ie mathematically correct) as opposed to the empirical methodology used in GCM(where they “appeal” to a closure relation )
EG http://i255.photobucket.com/albums/hh133/mataraka/ramanathanco2x.jpg

David Corcoran
February 26, 2009 12:27 pm

Joel Shore (09:01:02) :

Do you have a cite for Hansen making such a claim?

I turned to Professor James Hansen, the director of Nasa’s Goddard Institute for Space Studies, whose climate calculations have proved to be more accurate than anybody else’s. He believes the melting of the Greenland ice cap being picked up by his satellites today, now, suggests we are facing a 25-metre rise in sea levels this century – which would drown Bangladesh entirely.
Bangladesh is set to disappear under the waves by the end of the century
Scary. Citation and a confirming letter from Dr. Hansen found here:
American Thinker
but wait:
Bangladesh landmass is growing
Not so scary.

Peter
February 26, 2009 12:30 pm

tmtisfree:

Second law of thermodynamic prevents that the cooler atmosphere could warm the warmer surface without work. It is just not possible that surface is additionally warmed by this re-radiation.

The re-radiation doesn’t additionally warm the surface, but rather slows down the rate at which the surface loses heat – thereby keeping it warmer for longer than it would otherwise have been.
However, it only slows down the rate at which the surface loses heat by radiation – it doesn’t affect the rate of heat loss from conduction, convection or evaporation.

Steven Goddard
February 26, 2009 12:31 pm

David Ermer,
Like I said, this is not intended to be a qualitative article. The analogy is just to point out that more stoplights increase the difficulty of getting from point A to point B. The relationship with CO2 is of course not linear as you pointed out – a doubling of CO2 only impacts by 0.5% in temperature.

sunsettommy
February 26, 2009 12:31 pm

How can anyone state that CO2 is a “greenhouse” gas when the atmosphere it is in NOT a greenhouse?
http://www.globalwarmingskeptics.info/

sunsettommy
February 26, 2009 12:32 pm

The link in the comment was a mistake.It was supposed to be in the website box.Not related to my question.

Joel Shore
February 26, 2009 12:33 pm

gary gulrud says:

Where does this 341 W/m^2 come from? We are constantly told 1360 is the value.

The factor of 4 difference comes from the factor of 4 difference between the total surface area of the earth, which is 4*pi*r^2 and the earth’s cross-section as viewed from the sun (which is a circle of area pi*r^2). [Another way of thinking about this is that the 1360 W/m^2 is the intensity you would have if the sun were directly overhead; however, at any given time, half of the earth isn’t facing the sun at all and the other half all has the sun at some non-normal angle (except for the one point where it is directly overhead). So, averaged over the entire surface of the earth (or top of the atmosphere, if you want to get technical), the amount is 341 W/m^2.]

Steven Goddard
February 26, 2009 12:40 pm

jeepndesert,
Thanks for the listing as a Gore climate conspirator. I’m already listed as an oil company shill trying to destroy the climate. I’m just trying to get at the facts, whatever they may be. Maybe this year will be the hottest year ever, as Hansen predicted?
Speaking of which my UAH forecast for February is down from January at 0.24-0.26. I missed by 0.01 last month.

February 26, 2009 12:41 pm

You inspired me to quickly write a great new blog post in response to the inconvenient truths you left out of your article.
http://newworldliberty.wordpress.com/2009/02/26/global-warming-is-a-hoax-quick-and-dirty/

Roger Knights
February 26, 2009 12:42 pm

Holy cow. Well, at least the issues have been laid on the table, and the points-in-dispute have been made apparent. I guess a mulligan is in order.

Peter
February 26, 2009 12:44 pm

Steve Goddard:

There isn’t a lot of LW arriving from the sun. Temperatures on the sun are too hot.

Not a lot, comparatively speaking, but still a substantial amount.

Joel Shore
February 26, 2009 12:44 pm

Tom says:

Positive feedback means an internal amplification factor greater than 1, which means a self reinforcing, runaway process.

No it doesn’t. If the positive feedback is sufficiently strong that the first order response is greater than the original effect (e.g., if the temperature response from feedbacks is greater than the bare response), then you have a diverging series and you are correct. However, if the positive feedback is weaker than this, then what you get is amplification, for example: 1 + 1/2 + 1/4 + 1/8 + 1/16, an infinite series that converges to 2.

1., Historical evidence. There is no evidence in climate history, that temperature swings are being reinforced by positive feedback leading to runaway climatic changes. The most recent examle was the 1998 El Nino event, which generated an almost 1degree C spike in global temperature in less than one year. If positive feedback were to be present, the temperature would have kept climbing. Instead, after the El Nino was over, temperatures returned to their pre El Nino values.

Your statement is erroneous for the reasons that I noted above. In fact, the evidence from past historical events such as the eruption of Mt Pinatubo and the ice age – interglacial oscillations is the main line of evidence leading to estimates of what the feedbacks and resulting climate sensitivity is. And, most scientists have concluded that the sensitivity is 2-4.5 C for a doubling of CO2, which corresponds to about a doubling to a quadrupling of the “bare” response by net positive feedbacks.

2., Recent work by Dr. Roy Spencer, who, based on his research, in his recent congressional testimony stated, in front of Senator Boxer, that there is no positive feedback in the ( Earth’s ) climate. I guess he was willing to risk the charge of perjury.

It is not perjury if he sincerely believes this, which I think he does. However, that doesn’t mean that he is right.

3., Dr Hansen’ circular argument. He is clearly worried about a Venus like runaway heating caused by positive feedback. At the same time, he is postulating that positive feedback is already the operating principle of the Earth’s climate as witnessed by his projected climate sensitivity factor of 6.5, which implies existence of a strong positive feedback. If this were the case, we are already doomed. There is no magic “tipping point” . A positive feedback driven runaway process cannot be stopped any more than a nuclear explosion can be stopped halfway through the process.

Again, you are confusing the issues of positive feedback, true runaway, and also the possibility of tipping points. These are three distinguishable things. I think that most climate scientists would say that a net positive feedbacks is very likely, tipping points are quite possible (given the historical record) but we don’t know where they are, and an actual runaway is unlikely (although Hansen would claim otherwise…if we really don’t restrain ourselves at all in burning fossil fuels).

February 26, 2009 12:44 pm

A positive feedback…that´s alchemy. We need some to overcome financial crisis. Kind of Keynesian meteorology!!

February 26, 2009 12:45 pm

Steven,
I appreciate the attempt to find some common ground on what we agree on and where uncertainties remain. I agree that the radiative forcing of CO2 is one of the areas where we can be fairly confident in our projections, but that overall climate sensitivity (taking into account various feedbacks) is where the major debate lies.
Unfortunately, it seems by the comments that many folks have reached a point where any suggestion of the role of CO2 in warming the planet is dismissed out of hand. A bit of common ground would go a long way toward having a more constructive dialogue between both sides of the climate debate (which, as an aside, is why I enjoy Lucia’s place; no need over there to waste time arguing with people who conflate CO2 flux with CO2 stocks, for example).

February 26, 2009 12:46 pm

Positive feedback: the perpetual movement

James Griffiths
February 26, 2009 1:05 pm

Joel Shore says: (albeit a quote from someone else):
“The surface of the Earth is warmer than it would be in the absence of an atmosphere because it receives energy from two sources: the Sun and the atmosphere.”
I suppose that should be quantified (at least as I understand it)
The atmosphere (and the oceans) cool the surface while the sun is shining upon it, and prevent heat from leaving as quickly it might otherwise when the sun is not shining on it, thereby maintaining a mean temperature that we calculate is higher than might otherwise be the case.
In my interpretation, that puts the bulk of the mean difference as happening on dark surfaces of the earth when no SW radiation is incoming. Certainly, it seems plausible that LW radiative heating may have a stronger effect when there is no higher energy source of radiation entering the system., but it seems a stretch to attribute the warming that is claimed to it.
As the oceans have a huge thermal inertia compared to the atmosphere, and the surface temperatures are definitely not as hot as they would be without an atmosphere, I wonder why the atmosphere is the prime candidate for “carrying” the heat through the nighttime hours. Surely the medium that loses least energy over a certain period when an incoming source is terminated would be a more reasonable “culprit” for maintaining an average temperature in a system.
I am no expert, I am just applying my own twisted logic to the argument. I’m perfectly happy to be educated by anyone who knows better, even if it involves an egg/face incident.

Ray
February 26, 2009 1:08 pm

Without the sun to feed it, there is no feedback! Like pretty much all Law of nature, we should be dealing with an exponential relationship and nit just some “constant” number to represent the feedback.

Joel Shore
February 26, 2009 1:09 pm

Stevo says:

Oh, and the tropical troposphere ‘hotspot’ is a result of the water vapour feedback mechanism predicted to treble global warming. It’s absence is a blow not to the validity of CO2 greenhouse physics, but to the postulated feedbacks.

Sort of…but a little clarification is in order. If the hotspot in the tropical atmosphere is really absent (which I think is doubtful but the observational data is inconclusive at this point) then the most direct consequence of this is that the lapse rate feedback in the models, which is a NEGATIVE feedback that counters part of the positive water vapor feedback, is absent. So, the most direct consequence would be to imply that there should be more warming than the models predict. [The rough logic for this is that the place in the atmosphere that has to heat up a certain amount in order to put the earth back in radiative equilibrium is the upper troposphere. So, if the tropical upper troposphere is not heating more than the surface (as the models now predict), then the surface would have to heat up more in order to get the tropical troposphere heated up the necessary amount.]
Now, one can argue that if the models are not getting these convective aspects correct, they are likely wrong on the water vapor feedback too. However, since we have some independent observational verification that the upper troposphere is moistening approximately as expected (from Brian Soden’s work and also Dessler’s work), this seems unlikely.

February 26, 2009 1:29 pm

Princeton Physicist Tells Congress Earth in ‘CO2 Famine’ — Increase ‘Will Be Good for Mankind’
http://www.businessandmedia.org/articles/2009/20090225213407.aspx

Robert Wood
February 26, 2009 1:45 pm

Stevo (11:09:20) :
As good an explanation as I’ve seen.

gary gulrud
February 26, 2009 1:54 pm

“The factor of 4 difference comes from the factor of 4 difference between the total surface area of the earth, which is 4*pi*r^2 and the earth’s cross-section as viewed from the sun (which is a circle of area pi*r^2). ”
Thank you.

james griffin
February 26, 2009 1:55 pm

It may well be a good explanation of the AGW theory and clearly greater minds than mine will debate and argue for a long time to come.
However the article at this stage smacks of the desperation of the AGW’s.
Those in the know do not dismiss the basic theory…it is the outcome that is the sticking point.
As Prof Bob Carter said in his presenatation in Sept 2007…they put in the positives of their argument but not the negatives.
There has been no warming for around 10 years and the Aqua satellite did not find the hot spots in the upper atmosphere it should have done.
The Polar Ice Caps are returning and the changes we are experiencing from the change in the sun cycle are clearly at odds with the dire warnings of the AGW theory.
In fact the BBC even published an article from the Guardian regarding a recent admission by the Hadley centre that some of the more catastrophic outcomes of the increased CO2 levels are somewhat wide of the mark.
Having no scientific training I just use commonsense.
References to James Hansen being “correct” are in fact part of the admission that although his theory is correct…his wild predictions on the eventual outcome are not.
Good luch all of you who are qualified to debate this…we await the outcome with interest.

Drew Latta
February 26, 2009 2:00 pm

What in the world are the units on the x-axis of the second figure (1st plot with estimated temperature vs. something?)?
Huge pet peeve… not labeling axes!!

Simon Evans
February 26, 2009 2:02 pm

Paddy (11:52:11) :
Of course the “Earth was just fine” millions of years ago when CO2 levels were much higher. It was just fine when temperatures were higher or lower, just fine when sea levels were higher or lower, and so on. The changes from one state to another, however, have gone along with the evolution of life on the planet. In the case of rapid changes (which might be measured on the scale of tens or hundreds of thousands of years), some such changes are associated with mass extinction events. We’re currently considering the impact of changes on a century scale or less. The concerns are not for whether the planet will surivive (it will) but whether we can support our current population and its geographical distribution, along with out current biodiversity.

jae
February 26, 2009 2:03 pm

Hasit been, 11:07:20
“Wide Temperature Range
Temperatures in arid areas may get as high as 55 degrees C during the day and as low as 10 degrees C during the night. The drop in temperature at night occurs rapidly and will chill a person who lacks warm clothing and is unable to move about. The cool evenings and nights are the best times to work or travel. If your plan is to rest at night, you will find a wool sweater, long underwear, and a wool stocking cap extremely helpful. “
These types of accounts are what cause the MYTH of the frigid desert night and super hot days. This ONLY occurs at high elevations (and high latitude). It does NOT occur at low altitude deserts below 45 N/S latitude. In fact, It has nothing at all to do with deserts; it occurs at high elevations EVERYWHERE, not just in deserts. Look, for example, at Alamosa, CO, which is at about 2300 m elevation. It is not considered a desert, because it gets more than 5 inches of precipitation per year (mainly snow there). But a typical July day will vary from 27.8 C during the day to 8.8 C at night, for a diurnal variation of 19 C. Here you do need a coat at night! But it is no worse in any high altitude desert. For example, look at Tonopah, NV, which is at 1653 m elevation (even somewhat lower than Alamosa). Here the average high in July is 32.8 C and the average low is 13.6 C, for about the same diurnal variation. Now look at a typical low-elevation desert, like Daggett, CA, 588 m elevation. Here the variation is from 39.9 to 23.3 C, for a diurnal variation of 16.6 C. (The average relative humidities for these locations are 56 (Alamosa), 26 (Tonopah), 24 (Daggett)).
Also note that the LOW temperature of 23.3 C in Daggett is still higher than the average low temperature for Atlanta (20.8). You do not have to dress warmly for the night-time in July in either Atlanta or Daggett.
Data at http://rredc.nrel.gov/solar/old_data/nsrdb/redbook/sum2/state.html

Roger Knights
February 26, 2009 2:05 pm

OT: Bloomberg story: “Obama Plan Has $79 Billion From Cap-and-Trade in 2010”
http://www.bloomberg.com/apps/news?pid=20601087&sid=akMqKTP2B1wo&refer=home
Sample quote:
“President Barack Obama’s budget plan assumes $78.7 billion in revenue in 2012 from the sale of greenhouse-gas emission permits to polluters, putting pressure on Congress to pass legislation by early next year.
“A “cap-and-trade” program would generate a total of $645.7 billion by 2019, according to the budget blueprint Obama sent to Congress today. Initial funds would be used to invest in “clean” energy, help finance Obama’s tax credit for workers as well as offset higher energy costs for low- and middle-income people and clean up costs for small businesses.
“The budget calls for the Environmental Protection Agency to get $19 million to begin setting up an inventory of greenhouse- gas emissions”

jae
February 26, 2009 2:15 pm

Joel Shore, 12:44:28
“I think that most climate scientists would say that a net positive feedbacks is very likely, tipping points are quite possible (given the historical record) but we don’t know where they are, and an actual runaway is unlikely (although Hansen would claim otherwise…if we really don’t restrain ourselves at all in burning fossil fuels).”
If you believe this, I have a bridge to nowhere to sell you…. 🙂
Seriously, I don’t think a significant positive water vapor feedback is possible for the following reasons. Go ahead and assume that a doubling of CO2 causes a 1.2 C rise in temperature. It is trivial to show with the Classius-Clapeyron equation that this increase in temperature can cause, at most, an 8% increase in evaporation of water. Like CO2, the relationship between concentration and radiation is logarithmic. MODTRAN shows this, if you play with the water vapor concentration and leave everything else constant. According to MODTRAN a doubling of water vapor would increase radiation by about 15 w m-2 (first doubling). Thus, an increase of 8% water vapor would amount to only (0.08)(15) = 1.2 wm-2. Negligible.

jae
February 26, 2009 2:16 pm

Dammit. The above should read: “Like CO2, the relationship between concentration and radiation FOR WATER VAPOR is logarithmic.”

lgl
February 26, 2009 2:17 pm

Leif
Where do people get these ideas from? Googling the internet?
No ,this is textbook knowledge, which you usually embrace, why not this time?

David Porter
February 26, 2009 2:17 pm

foinavon (10:14:41) :
As usual you disparage all the previous theories and replace them with the newly concocted papers from the early 2000 to 2008. That makes me very suspicious and it should make you suspicious also. The fact that it doesn’t, speaks volumes to me. You should ask yourself how the font of all knowledge appears only in the last eight to nine years.
On the theme that you think life began only nine years ago you may be aware that there was a medieval warm period and a little ice age. Outside of your own fledgling scientific field all other facets of science acknowledged these historical periods. Unfortunately these periods run contrary to the CO2 thesis. Then along came Mann and his bent hockey stick and the rewrite of historical temperature records commenced. Even though Mann’s work has become the most discredited piece of climate science, along comes a whole raft of other studies purporting to show variants of the hockey stick. Naturally all of these studies are post Mann. None of them pre-date him. Coincidence? Not likely.
And Anthony, it’s not a question of over analysing. Foinavon will argue black is white as long as it is in defense of that beloved molecule, CO2.

February 26, 2009 2:17 pm

When I was in college, we were taught how to solve equations with two or three variables. We applied this math to try and figure out how electric motors really worked so we could predict horsepower output and efficiency. When the professor started writing down all the variables (about 15 or twenty in all), we found it impossible to calculate the output. Of course, the professor already knew this. It got worse when we tried to figure out how radio antennas really work (some configurations are pretty exotic). Even the experts admit that some of this is black art with trial and error because of all the variables and our lack of understanding of certain aspects of electromagnetics.
So after reading all this back and forth. I started thinking of the THOUSANDS (if not more) of variables far beyond the very few mentioned in the arguments in the comments to this posting).
There is no way any supercomputer can be set up to model all the variables on the Earth and coming at us from the sun, etc.
Mr. Goddard’s simplistic post on the greenhouse effect doesn’t do anything to show CO2 affects us at all, and is a disappointing primer no better than telling my son that the reason a car works is because you put gas in it and push on the pedal. To understand it, you need to know what’s under the hood.
So how did we finally figure out the horsepower and efficiency of the motor, or the effectiveness of a radio antenna? We tried it out, tweaked stuff, recorded data and drew conclusions. We dropped the idea of calculating or modeling how the stuff worked.
So back to the Greenhouse: I simply watch the satellite data and see the temperature statistics over time. The Arctic icecap did not melt last summer, and it doesn’t look like it will before I’m gone. And no one seems to be able to find arguable evidence of the oceans rising at historic tidal water marks at various locations around the Earth from these past centuries.
Sometimes observation over time, is better than trying to predict 50 years into the future with math we just can’t solve.

Peter
February 26, 2009 2:23 pm

Joel Shore:

However, if the positive feedback is weaker than this, then what you get is amplification, for example: 1 + 1/2 + 1/4 + 1/8 + 1/16, an infinite series that converges to 2.

That being the case, and I agree btw, how can they predict temperature increases of several times what can be attributable to CO2 forcing?

AlexB
February 26, 2009 2:28 pm

Steve,
Looking at three planets and saying that as greenhouse gas concentration increases so does temperature does not really settle the issue for me I’m affraid. The proxy record of fluctuating CO2 and temperature indicates quiet clearly to me that radiative heat transfer is by far not the dominant heat transfer mechanism in the earths atmosphere. Increased temperature increases verticle wind shear. To put it in terms of your car analogy CO2 would add an extra traffic light on the road between your school and place of work, but then verticle wind shear would go and put an expressway between them.

Colonel Sun
February 26, 2009 2:28 pm

Colonel Sun (10:14:32) :
Mark (06:36:27 wrote :
“Has there ever been a study that involved a bunch of greenhouses where each had different amounts of CO2 (all other factors equal) and all subjected to the same amount of sunlight to see how the different CO2 levels affect temperature?”
Winner, winner, chicken dinner. Excellent question.
Or take two “aquariums” with walls of whatever material is most transparent to infrared.
One filled with normal atmosphere and one filled with pure CO2. Both partly filled with water.
Shine infrared lights on both. Any difference in the rise in and final equilibrium temperature of the atmospheres and water?
REPLY: The experiment is too simple to replicate our atmosphere – Anthony
The experiment is not about replicating the atmosphere, but empirically evaluating the efficacy of CO2 as a greenhouse gas relative to the atmosphere.
As for Venus and Mars, the 1/r^s of solar radiation is the dominant effect.

David Porter
February 26, 2009 2:29 pm

Joel Shore (12:44:28) :
“No it doesn’t. If the positive feedback is sufficiently strong that the first order response is greater than the original effect (e.g., if the temperature response from feedbacks is greater than the bare response), then you have a diverging series and you are correct. However, if the positive feedback is weaker than this, then what you get is amplification, for example: 1 + 1/2 + 1/4 + 1/8 + 1/16, an infinite series that converges to 2.”
Not again Joel please. It’s getting like Ground Hog day.

Mark_0454
February 26, 2009 2:29 pm

Joel Shore 12:44:28
“I think that most climate scientists would say that a net positive feedbacks is very likely,….”
I watched the you-tube debate at the JLF foundation. It was my impression that experimental evidence at this point is that feedback is negative. No evidence for positive feedback.

Simon Evans
February 26, 2009 2:32 pm

jae (14:15:27) :
Thus, an increase of 8% water vapor would amount to only (0.08)(15) = 1.2 wm-2. Negligible.
Accepting your figures for now, that’s equal to 75% of the current net anthropogenic forcing. Why negligible? CO2 doubling is likely to be reached by the 2050s at current rates. So, to put it in simplified terms then, by your calculations we are looking at 1.2C + 0.9C = +2.1C by the 2050s. How ‘negligible’ is that?

Gerald Machnee
February 26, 2009 2:37 pm

Steven Goddard (10:04:26) :
**Cold air over ocean causes fog, which blocks LW. During the miserably cold California summer of 1998, I remember that we were lucky to get an hour of sunshine at noon on the beaches south of Santa Cruz. I also remember bundling up in several blankets to watch the fireworks.
“The coldest winter I ever spent was my summer in San Francisco”
Mark Twain
Sorry I meant to say that fog blocks SW (not LW) which is the dominant factor during the day. LW blocking is more interesting at night.**
Steve, you have it backwards in the first sentence. The cold air over the ocean does not cause fog because it would get heated and develop into convective cloud. That is how you get showers or snow showers in the winter further north. Advection fog develops when warm moist air moves over cooler water. Radiation fog develops over land at night when land cools due to LW radiation.
SW radiation heats the water, but because it penetrates up to hundreds of feet the daily change can only be a fraction of a degree. It takes a long time to heat water. The Great Lakes take a couple of months to get to 15 to 20 degrees. There is a similar effect for cooling. Water will cool by emitting LW radiation, but it also emits from the depths so it can only cool a fraction of a degree overnight and therefore radiation fog cannot form over water. This is why the Great Lakes do not freeze during an average winter. Lake Erie being the most shallow will freeze first.
On the other hand LW radiation emitted from land can drop the temperature 20 to 30 degrees overnight(on a clear night).
Cloud cover will reradiate the LW back down and slow the cooling.

Colonel Sun
February 26, 2009 2:42 pm

That should be 1/r^2

jae
February 26, 2009 2:44 pm

Simon: That’s WATTS, not degrees. 3.7 watts translates to 1.2 C; so 1.2 watt translates to only 0.39 C. And that’s a worst-case scenario.

John Galt
February 26, 2009 2:45 pm

jae:
Death Valley is about as low a desert as you can get (it’s below sea level). In July, the average temperature range is for the high is 115 with an average low of 86, according to the weather channel.
Disclaimer: I don’t know exactly where the official temperatures are recorded in Death Valley. Death Valley is about as dry and barren as you can get, and is surrounded by mountains on all sides. There is no urbanization there, but official temps could be affected by land use changes near the weather station.
There should be comparable high desert sites nearby for comparison. What’s the altitude of Baker, CA? It’s not far from Death Valley (and is home to the world’s largest thermometer, btw) and except for possible local siting issues, there isn’t any urbanization to speak of in Baker, either. Weather.com shows the July average ranges from 108 to 76.

Hank
February 26, 2009 2:48 pm

Would it be wrong of me to wonder, after reading through this primer and the reactions, that the “greenhouse effect” might be an overly simplistic metaphor. Greenhouses do not warm for reasons anything like the greenhouse effect. It almost seems that the atmosphere is acting as “thermal mass” slowing the process of re-radiation.
I understand that absent an atmosphere the surface of the moon has greater temperature extremes than the earth, but maybe it would be good to consider how subsurface soil temperatures might behave when comparing the moon to the earth. After all, absent an atmosphere, the moon’s surface is more akin to the our boundary of space than the earth’s surface.

John Galt
February 26, 2009 2:53 pm

Mike Strong (14:17:57) :

There is no way any supercomputer can be set up to model all the variables on the Earth and coming at us from the sun, etc.

It’s easy to program: Just put in the observed temperature rise from 1979 to 1998, then program the computer to maintain the trend! Viola – runaway AGW!
The next step is to concoct an explanation of how this might work, and vigorously defend your results no matter what the actual temperatures are.
The point being: Computer programs only do what the programmer tells them to do. GIGO is the first thing I learned from my first programming class (Fortran, btw). This means Garbage In, Garbage Out.
Climate models show future warming simply because that’s what they were written to do. If you assume more CO2 causes more warming and you also assume there will be increased CO2 in the future, what other output could the models possibly have?

Gerald Machnee
February 26, 2009 2:53 pm

James Griffiths (13:05:33) :
Joel Shore says: (albeit a quote from someone else):
“The surface of the Earth is warmer than it would be in the absence of an atmosphere because it receives energy from two sources: the Sun and the atmosphere.”
I suppose that should be quantified (at least as I understand it)
The atmosphere (and the oceans) cool the surface while the sun is shining upon it, and prevent heat from leaving as quickly it might otherwise when the sun is not shining on it, thereby maintaining a mean temperature that we calculate is higher than might otherwise be the case.
The surface of the earth receives little energy from the atmosphere, it receives the energy from the sun. A proper diagram would show this.
SUMMARY: The earth receives all its energy from the sun by SW radiation. This heats the surface of the earth. The surface of the earth (land and water) then heat the air next to it by conduction and this is redistributed by convection, horizontal motion, evaporation and condensation. A small amount of heating of the atmosphere is caused by LW radiation emitted by the land and water.
Briefly, when the average temperature of the earth is not changing, the earth emits (loses)by LW radiation the same amount of energy it has received by SW radiation.
The earth is always emitting LW radiation. Any object above absolute zero emits LW radiation. When the sun is shining you have a net gain of SW over LW. AT night you have a net loss. AS you go towards the poles you have a net loss especially over the south pole where there is little dark surface to absorb the SW. Ocean and atmospheric circulation redistribute the heat energy.
Clouds modify the rate at which parts of the surface lose energy.

Joel Shore
February 26, 2009 2:53 pm

tmtisfree says:

Second law of thermodynamic prevents that the cooler atmosphere could warm the warmer surface without work. It is just not possible that surface is additionally warmed by this re-radiation.

Others have already responded to this but I will just add my voice: The second law says only that the NET flow of heat can’t be from the cooler atmosphere to the warmer surface without work. I.e., all that is required is that there is more heat flowing from the surface to the atmosphere than vice versa.
You might think, “Well, how then can the atmosphere heat the surface if the NET heat flow is from surface to atmosphere?” The answer to this conundrum is that the comparison case (the “control” if you will) is the case where all the heat from the earth’s surface escapes into space with none of it being returned to the surface by the atmosphere. So, anything that the atmosphere does return, even if only a fraction of what it receives, is extra heat to the surface.

What is true is that physical laws can not be broken to fit misconceptions.

Indeed. However, it is also true that misconceptions about a process breaking physical laws also need to be considered. The Second Law of Thermodynamics is a very powerful tool…But, with that power comes the possibility that it can be abused. G&T abused it by applying it incorrectly.

Reed Coray
February 26, 2009 2:56 pm

First, I want it clear that I don’t buy into Steve Goddard’s simplified radiation diagram. As many previous comments have noted, the earth’s absorption, distribution, retention, and transmittance of heat is an extremely complex phenomenon. With that caveat, however, I’m going to take Steve Goddard’s suggestion to heart and try to learn more about “feedbacks”. As I see it, AGW alarmists treat increasing atmospheric CO2 levels as both an original forcing function and as a feedback process. I’d like to know which if either (both) of these roles is paramount.
First role, increasing CO2 levels are feedbacks. AGW alarmists argue (or at least at one time Al Gore so argued) that based on data from Antarctic ice core samples, (a) over the past approximately 600,000 years there was a high correlation between Antarctic temperature fluctuations and Antarctic CO2 levels, and (b) that this correlation proves that increasing atmospheric CO2 levels caused increasing atmospheric temperatures. Aside from the logical fallacy that “correlation establishes cause and effect”, the AGW alarmists had to adjust their “proof” when it was leanred that Antarctic temperature changes preceded Antarctic CO2 changes by approximately 1,000 years. The AGW alarmist’s adjusted argument is that although some physical phenomenon other than increasing CO2 levels originated Antarctic temperature increases, the original increased temperatures released CO2 into the atmosphere, and the released CO2 amplified the original temperature changes. Thus in the adjusted AGW alarmist argument, atmospheric CO2 release is a “positive feedback”, not the originating phenomenon.
Second role, increasing CO2 levels are an original forcing function. AGW alarmists argue that via the burning of fossil fuels, man’s release of CO2 into the atmosphere will cause the average global temperature to increase by anywhere from 0.5 to 1.2 degrees centigrade. Since it’s hard to argue that this modest temperature rise will induce the catastrophic effects needed to impose draconian reduction in fossil fuel energy production, the AGW alarmists further argue that man’s release of CO2 will trigger non-CO2 feedbacks that will result in larger temperature increases–by as much as five degrees. In this second AGW argument, atmospheric CO2 release is the originating phenomenon, not a feedback.
So the AGW alarmists have increasing CO2 levels acting as both an originating and a feedback phenomenon. I’d like to know which if either is correct. Now it’s conceivable that increasing CO2 levels are both. However, if both, then what stopped runaway global warming in the past? The second AGW alarmist argument (an increasing CO2 level is the originating forcing function) states that a doubling of atmospheric CO2 levels will via non-CO2 feedbacks cause a temperature increase of approximately three degrees more than the originating temperature increase from increasing CO2 levels. The first AGW alarmist argument (an increasing CO2 level is a positive feedback) states that an increase in temperature will cause the release of additional CO2, which will act as a feedback and cause additional increases in temperature. Somewhere in all this confusion it seems to me we may have a runaway feedback process. After all, the AGW alarmists argue that temperature deltas at the system input produce larger temperature deltas at the system output. If the larger temperature deltas at the system output are “fed back” into the system input, why won’t they cause even larger temperature deltas at the system output, which when fed back to the system input cause even larger temperature deltas to the system output, etc. If it exists, I’d like to see a “circuit diagram” of the AGW alarmist’s temperature model where (a) the model inputs and outputs are defined, (b) all feed-forward effects (delays and multiplication factors) are shown, and (c) all feedback effects (delays and multiplication factors) are shown. For a specified input, such a diagram could then be used to predict the time behavior of the system output. If such a diagram doesn’t exist because (a) the delays are time varying, or (b) the multiplication factors are time varying, or (c) the system can’t be modeled using a linear feed-forward/feedback structure, or (d) [my choice] the system is just so complicated it can’t be represented using feed-forwards, feedbacks, and multipliers, then please give me a mathematical representation of the system (not just words, but a well-defined set of rules) so that others can analyze the system’s behavior. If the AGW alarmist’s can’t provide such a mathematical description, then PLEASE QUIT TELLING THE WORLD WHAT IT MUST DO TO PREVENT CATASTROPHIC GLOBAL WARMING.

Joel Shore
February 26, 2009 2:59 pm

Would it be wrong of me to wonder, after reading through this primer and the reactions, that the “greenhouse effect” might be an overly simplistic metaphor. Greenhouses do not warm for reasons anything like the greenhouse effect.

Well, in a very rough sense, both act by trapping heat that would otherwise escape. However, the mechanism by which the heat is trapped is very different for a greenhouse as it is for the atmospheric greenhouse effect. So yes, it is well-understood that “greenhouse effect” is somewhat of a misnomer (see, e.g., what Wikipedia says about it: http://en.wikipedia.org/wiki/Greenhouse_effect ) but it happens to be the name that we ended up with historically.

Joel Shore
February 26, 2009 3:02 pm

Mark_0454 says:

I watched the you-tube debate at the JLF foundation. It was my impression that experimental evidence at this point is that feedback is negative. No evidence for positive feedback.

That may be your impression from watching that debate but the actual fact is that most of the observational evidence in the peer-reviewed literature is precisely the opposite of your impression.

Joel Shore
February 26, 2009 3:08 pm

John Galt says:

It’s easy to program: Just put in the observed temperature rise from 1979 to 1998, then program the computer to maintain the trend! Viola – runaway AGW!

Climate models show future warming simply because that’s what they were written to do. If you assume more CO2 causes more warming and you also assume there will be increased CO2 in the future, what other output could the models possibly have?

That is not really a correct description. The global climate models are programmed with the best understanding of the actual physical processes involved. Some of this knowledge is admittedly better understood than others (e.g., radiative forcing due to greenhouse gases – well understood, radiative forcing due to aerosols – less well understood).
There is also a lot of data out there to test the models against (current climatology, the 20th century climate record, the Mt. Pinatubo eruption, the ice age – interglacial cycles, …)

Simon Evans
February 26, 2009 3:10 pm

jae, I know you’re talking watts. Actually your figure of 1.2 isn’t all that far away from the IPCC’s models’ mean of 1.80 ± 0.18. Lapse rate is negative, -0.84 ±0.26 W m–2. The big uncertainty is cloud feedback, with a mean of 0.69 W m–2 but a large spread of ±0.38 W m–2. Surface albedo feedback is 0.26 ±0.08 W m–2 .

pkatt
February 26, 2009 3:10 pm

Dave L I have a few questions for you:) (never met anyone with a greenhouse) In your greenhouse when you raise Co2 do you have to turn on the airconditioner? From day to night at an increased level of Co2 does your daytime temp spiral out of control unless you adjust it manually?.. And Im guessing you have to constantly replenish the Co2.. it just doesnt get there and stay does it?
Venus and Mars are for all general purpose devoid of life as we know it. No plants, no people. You cannot compare. We are part of a rather large cycle of biological chemistry that those planets do not know.
Another question for everyone. If Co2 can absorb reflected energy at a specific wave length, doesnt it also absorb incoming energy at that same wave length, keeping it from its destination? So what percentage of the incoming energy is absorbed and immediately dispursed back to space by a higher Co2 content. What percentage is redirected into a different wave before it even reaches earth. What percentage is slowed to the longest wave and “trapped” within our atmosphere unable to reach escape velocity. And doesnt it seem strange we are talking about such a narrow bandwith changing our entire climate?
Ok lets add to that. At the height of a solar cycle, particle density arriving from the sun increases. We have seen in years of the last solar maximum events that drove our meters off the chart. Not all of these were deflected by our magnosphere. In fact, in some cases our “tail” snapped back infusing our atmosphere with energised particles. So what percentage of our warming was due to having energised matter infused into our atmosphere. I have yet to see a study of what happens to that energy on whatever frequency it comes at except that sometimes it makes pretty northern lights. The sun earth interaction needs way more study than Co2. I think our quiet sun is teaching us a lot right now. We are seeing individual events and are more able to study the individual effects. It could be the real age of enlightenment, if the dark ages folks would just give up gloom and doom and open their eyes, but that would require admitting we have barely begun to scratch at the surface of the causes of climate change. But .. I dont want to derail the current discussion so I will leave you what I’ve been reading, maybe we can discuss that another time.
http://geomag.usgs.gov/downloads/pt2008.pdf

gary gulrud
February 26, 2009 3:10 pm

“The second law says only that the NET flow of heat can’t be from the cooler atmosphere to the warmer surface without work.”
Yea, Joel!

Carrick
February 26, 2009 3:10 pm

Steve,
Great contribution. I’d like to seek clarification if you have time:
As I understand it, the classic greenhouse effect itself represents a feedback process. Namely LW radiation is reflected back to the ground, causing the ground to heat, which in turn radiates more LW radiation. (Part of which in turn is radiated back to the ground, yada yada yada). This of course describes a feedback process.
As I understand it, the dispute is over the feedback associated with the effect of the increase in CO2 on water vapor in the atmosphere, but I think this classic greenhouse gas effect is properly a feedback process too.
Secondly, it is my understanding that simple horizontal-layer models of the Earth’s atmosphere exist, for which numerical computation including cloud effects are possible (using e.g. the large-eddy simulation method). Do you know of anybody who’s actually done a study of the CO2 sensitivity for such models, and if so what is the climate sensitivity of CO2 in such models?
Thanks.

gary gulrud
February 26, 2009 3:13 pm

““The surface of the Earth is warmer than it would be in the absence of an atmosphere because it receives energy from two sources: the Sun and the atmosphere.””
Boo, Joel!

Steven Goddard
February 26, 2009 3:20 pm

Gerald,
I’m guessing you haven’t spent a lot of time at the beach in Northern California in the summer. Convective clouds are non-existent. Fog forms over the water and coast as soon as the air cools slightly, and only burns off for a few hours when the air temperature rises a few degrees. Some days the fog never lifts. I have a friend who paid his way through college at Berkeley selling sweaters to unprepared San Francisco tourists.

James Griffiths
February 26, 2009 3:21 pm

Gerald Machnee:
Thanks, your explanation is very much appreciated, but I understand that already.
My point, however clumsily expressed was that, making the most simplistic of comparisons with the moon, the surface temperature appears to be significantly cooled (rather then heated) by the atmosphere (and/or oceans) while in direct sunlight, and much warmer during the night hours.
My question is that, as peak surface temperature is much lower than it would be without an atmosphere, the mean average must surely be maintained by losing as little heat as possible during the period when the surface is not receiving SW warming.
Logically, I would assume that the bulk of this duty would be performed by the part of the system that is most efficient at storing heat, which is the oceans not the atmosphere.
If you, or anyone else can help me understand better whether my logic has any credible basis or not, that would be great!

Steven Goddard
February 26, 2009 3:29 pm

Roger Sowell,
You are correct. The equation
outgoing radiation = incoming radiation – changes in oceanic heat content
shows that if “changes in heat content of the oceans” is positive, the amount of outgoing radiation is less than incoming.

February 26, 2009 3:30 pm

lgl (14:17:20) :
Where do people get these ideas from? Googling the internet?
No ,this is textbook knowledge, which you usually embrace, why not this time?

Here is why [in answer to Phil]:
Phil. (09:56:18) :
The heat content of the air is 7/2kT per diatomic molecule (N2 and O2, which is essentially all there is). For each CO2 molecule there are about 3000 N2 and O2 molecules. About 4% of the CO2 molecules will be thermally excited at 300 K. The vibrational frequency of the bend in the molecule is 667 cm-1, kT at 300 K is about 200 cm-1, so the relative population will be exp(-667/200). Therefore, for each excited CO2 molecule there will be 75000 O2 and N2 molecules. The total heat content of those molecules will be 7/2 kT x 75000 ~ 5*10^6 cm-1 [The unit cm-1 as an energy unit is standard for spectroscopy]. The relative amount of deposited energy per excited CO2 is thus about 0.00001of the total energy. The temperature rise per CO2 excited is therefore negligible.

SteveSadlov
February 26, 2009 3:30 pm

What an outstanding tutorial. Good concise use of Physics. Bravo!

jae
February 26, 2009 3:31 pm

John Galt, 114:45:30
Tonopah isn’t very far from Death Valley. Both are 100 miles from the nearest tree, 200 miles from the nearest water, and 6 inches to hell!

Alex` Heyworth
February 26, 2009 3:48 pm

James Griffiths:
re the atmosphere cooling the surface while the sun is shining. This is incorrect. The atmosphere, like all things at non-zero (K) temperature, radiates heat. During the day, it still radiates heat. In fact the amount of heat the earth’s surface receives from the atmosphere is about twice the amount it receives from the sun. Although the sun is much hotter, it covers only a very small portion of the sky (90,000 suns would be required to cover the whole sky). A high intensity of radiation from the direction of the sun does not outweigh a small amount of radiation coming from the whole sky.
Re the article: “About 30% of the incoming shortwave radiation (SW) is reflected by clouds and from the earth’s surface. 20% is absorbed by clouds and re-emitted back into space as longwave (LW) radiation.” Strictly speaking this is incorrect. Radiation is never re-emitted. The photons it comprises are absorbed (by clouds, CO2 or whatever) and their energy converted into kinetic and potential energy. Because the temperature of the cloud or whatever has then increased, it radiates more, so that new photons (at different wavelengths) are emitted.
Simon Evans: re the rapidity of previous temperature changes. The drop in temperature at the beginning of the Younger Dryas took place over only about a decade (see http://en.wikipedia.org/wiki/Younger_Dryas). How cold was it? Approx. 15C colder than today at the summit of Greenland. Mean temperature in the UK of around 5C. Different data sources suggest that the warming that ended the Younger Dryas (of around 10C +/- 4C) took either 40-50 years, in three discrete steps, or only a few years in total. Of course, there may have been species extinctions associated with these events. But there still seems to be a great deal of biodiversity.

stas peterson
February 26, 2009 3:53 pm

Kevin dick,,
I am a Engineer with a degree in mathematics as well. I went over the Miskolczi’s differential equations, and I agree with his mathematics.
But the mathematics is dependent on the physical models that he postulates.
He is an empiricist as a Scientist, and simply looked at the satellite data reporting on the atmosphere, in his position at NASA. The data did not agree with the current GHG basis. That is based on radiation that Milne’s theory predicts and calculates.
His physical theory makes much more sense as a basis of planetary radiation theory. If Milne’s semi-infinite atmosphere model wasn’t the accepted and current theory it would be laughed off the stage as a ludicrous joke.
Just for kicks see if you can find the patch of Outer Space between your shoe-tops and the soles of your feet. If you can’t seem to find it, then Milne’s theory MUST be wrong. It is only that chunk of Space That MUST be there, that has prevented the Earth from descending into a global hell of catastrophic global warming eons ago, supplied with an infinite pool of GHG hydrogen dioxide. That is what the current theory predicts.
Miskolczi’s theory may not be totally correct, but it is a lot more rational than the current nonsense that passes as our accepted planetary atmosphere and its radiation balance. As an analogous example, Copernicus’ sun centered model was not fully correct. Kepler had to demonstrate that the planets were in elliptical not circular orbits, as Copernicus thought. But Copernicus’ model was much closer to reality than Ptolemy’s epicycles, with in epicyles, as a model of planetary movement about not the Sun, but the Earth.

Robert Wood
February 26, 2009 3:57 pm

O/T sorry:
I notice Spaceweather.com is still counting the sunspot #1013 which is now invisible.
How can an ex-micro-spec be counted as an SSN of 14.
Is this being counted by the official “spec”ulators?

Simon Evans
February 26, 2009 4:02 pm

Reed Coray (14:56:21) :
Since you ask questions I will respond to your post, despite your consistent use of the pejorative label ‘alarmist’ (I do not understand why this term is considered acceptable here. I do not label you or others whose views I do not share with pejorative terms).
the AGW alarmists had to adjust their “proof” when it was leanred that Antarctic temperature changes preceded Antarctic CO2 changes by approximately 1,000 years.
You don’t know your history. Milankovitch theory was well-supported by the 1970s ( the Vema 28-238 core and others having provided good verification), and the search then was for an explanation of how small solar variations could explain the extent of the glacial cycles. Scientists were considering positive feedbacks as an explanation in advance of discovering the relationship between temperature and CO2. When the Vostok cores were pulled up, by 1985, the correlation was evident, but it was always seen in terms of feedback. Your suggestion of an ‘adjustment’ in the argument is simply an invention.
So the AGW alarmists have increasing CO2 levels acting as both an originating and a feedback phenomenon. I’d like to know which if either is correct.
Both.
if both, then what stopped runaway global warming in the past?
The temperature feedback is less than the input.
If the larger temperature deltas at the system output are “fed back” into the system input, why won’t they cause even larger temperature deltas at the system output
They are not – at least, not fully. I don’t like the use of the term ‘feedbacks’ in climate science since it gives rise to this confusion, but I guess we’re stuck with it. Yes, CO2 induced warming may give rise to further increase in CO2 concentration, but is not likely to equal the ‘input’ (although some do consider the danger of methane release from the permafrost/sea bed, which I expect is what Hansen has in mind when he considers the possibility of a runaway effect if we were to burn all fossil fuels).
If it exists, I’d like to see a “circuit diagram” of the AGW alarmist’s temperature model where (a) the model inputs and outputs are defined, (b) all feed-forward effects (delays and multiplication factors) are shown, and (c) all feedback effects (delays and multiplication factors) are shown.
I’d recommend Chapter 8 of the AR4. It may not be quite in the form you want, but you can figure out answers to your questions:
http://www.ipcc.ch/pdf/assessment-report/ar4/wg1/ar4-wg1-chapter8.pdf

Fernando
February 26, 2009 4:03 pm

Steven: Good point.
No system is in thermodynamic equilibrium.
Time is not variable thermodynamics.
The Earth is an open thermodynamic system. (Possible decrease of entropy localized…ICE , urban heat island, life).

Gerald Machnee
February 26, 2009 4:05 pm

Steven Goddard (15:20:37) :
Gerald,
“I’m guessing you haven’t spent a lot of time at the beach in Northern California in the summer.”
Truthfully, I have been to LA but not San Francisco. However I do have pictures of the fog I use in presentations. Then I have to name the type of fog. I do not resort to Wikipedia much but I will quote the following:
The famous fog of San Francisco, California is a specific case of sea fog. Sea fog is a specific type of advection fog, which is characterized by the lateral transfer of temperature by wind blowing over cooler water. The water is often cool enough to lower the temperature of the air to the dewpoint, causing fog generation. Coastal areas in Mediterranean climates, such as San Francisco, have especially high occurrences of sea fog blowing off the ocean to just a few miles inland.
In San Francisco, the fog is created when warm, moist air blows from the central Pacific Ocean across the cold water of the California Current, which flows just off the coast. The fog can be very breathtaking if you are on the Golden Gate Bridge.
Temperatures can become very cold when the fog comes rolling in. Many people suggest you wear layers. Or, many fisherman sell thin, sweaters to people during the fog.

pft
February 26, 2009 4:09 pm

Correct me if I am wrong, but the deserts which are pretty cold in the evening have plenty of CO2. They are cold because of the absence of water vapor. What happened to the CO2 blanket?
The main cooling from the surface is actually weather: convection, cloud formation and precipitation. These are all poorly understood and there is little historical data to feed the models. In fact, if our atmosphere were static w/o weather and with the same amount of GHG as today, we would have 140 deg average surface temperatures. As for the Venus example, atmospheric pressure on Venus is enormous and it is much closer to the sun. Even w/o CO2 it would be blistering hot.
Since H20 is not present in significant amounts in the upper troposphere, where CO2 is presuambly well mixed, it would seem to me that atmospheric warming as a result of CO2 would show significant warming in the upper troposphere, yet we do not see this, do we? We do see stratospheric cooling as a result of higher CO2 levels. Could CO2 also be an important player in the cooling of the Earth (a negative feedback)?
In any event, nobody seriously argues that CO2 does not cause warming, just that it is not known how much, and the more CO2 there is, the less effect it has.
Your statement that “It is widely agreed that a doubling of CO2 will increase atmospheric temperatures by about 1.2C, before feedbacks.” is rather disturbing, as is your neglect to mention negative feedbacks. Widely agreed is just another term for consensus.
As for the stoplight analogy, you forget convection that allows the radiation to hop on the back of a CO2 molecule that runs through some or all of the red lights, although certainly higher CO2 levels mean that the heat removal takes place a bit slower, so perhaps a yellow liht anaology is more appropriate.
And this one: “increases in atmospheric temperature cause the ocean to warm up. Thus changes the oceanic heat content become the short term imbalance in the incoming/outgoing equilibrium equation”
If atmospheric temperatures rise first as a result of GHG, as they should, then the transfer of heat to cooler surface waters removes heat from the atmosphere. The ocean circulation allows the heat to be absorbed from the surface waters by the cold deep ocean. The heat capacity of the oceans simply dwarfs that of the atmosphere, so it is a great heat sink. But isn’t Hansen saying the heat is disappearing into the ocean, and then this heat will return somehow and heat the atmosphere?
A lot of my arguments came from Junk Science, so I really do not know what to make of this article.

pft
February 26, 2009 4:12 pm

“and the more CO2 there is, the less effect it has. ” should have said “and the more CO2 there is, the less affect more CO2 has”

Eric Anderson
February 26, 2009 4:23 pm

pkatt wrote:
“Dave L I have a few questions for you:) (never met anyone with a greenhouse) In your greenhouse when you raise Co2 do you have to turn on the airconditioner? From day to night at an increased level of Co2 does your daytime temp spiral out of control unless you adjust it manually?.. And Im guessing you have to constantly replenish the Co2.. it just doesnt get there and stay does it?”
I realize pkatt is having a bit of fun here, but I think this is actually a very useful thought experiment, and, frankly, would be a useful actual experiment to perform. Construct a large greenhouse with significant vegetation, some arid areas and a large open water reservoir. Pump in a significant amount of CO2 and see if the temperature spirals out of control. If not, why not?
Now, this doesn’t mean that all the alleged horrors of AGW are false, after all, one could certainly posit all kinds of extreme weather events even without a runaway greenhouse effect. However, it should at least provide evidence that the runaway greenhouse effect, the infamous “tipping point” is a non-issue (as if any further evidence were needed).

February 26, 2009 4:23 pm

Roger Knights (14:05:43) :
“Cap and trade” is a perfect example of a stealth tax on Americans. It is being sold as an ‘investment’; the gov’t would “invest” the “funds.”
In reality, cap and trade is a new way for the federal government to separate citizens from their money. And it is a lot of money — $645 Billion, that they admit to.
There would be a firestorm of protest if the government proposed a new $645 Billion tax increase. But when that tax increase is based on a completely non-existent problem — atmospheric carbon dioxide levels — then taking $645 Billion from people is A-OK.
0bama is slick. But he lacks honesty.

Simon Evans
February 26, 2009 4:23 pm

Alex` Heyworth (15:48:45) :
The drop in temperature at the beginning of the Younger Dryas took place over only about a decade (see http://en.wikipedia.org/wiki/Younger_Dryas). How cold was it? Approx. 15C colder than today at the summit of Greenland. Mean temperature in the UK of around 5C. Different data sources suggest that the warming that ended the Younger Dryas (of around 10C +/- 4C) took either 40-50 years, in three discrete steps, or only a few years in total. Of course, there may have been species extinctions associated with these events. But there still seems to be a great deal of biodiversity.
Indeed, but we’re not sure how global the effects were or, if they were, the pace of global change. There certainly is evidence for rapid regional changes in climate, as with Dansgaard-Oeschger events. In our own time regional temperature changes have far exceeded the global mean, of course. But basically I agree, that current climate change is not going to threaten the extinction of life on earth! Life will continue, but not, IMV, as we have known it. Some forms of life may flourish in warmer conditions and be able to take advantage of the pace of change. As for human society, I don’t fancy the implications of social disruption and conflict.

James Griffiths
February 26, 2009 4:23 pm

Alex Hayworth says:
“James Griffiths:
re the atmosphere cooling the surface while the sun is shining. This is incorrect. The atmosphere, like all things at non-zero (K) temperature, radiates heat. During the day, it still radiates heat. In fact the amount of heat the earth’s surface receives from the atmosphere is about twice the amount it receives from the sun. Although the sun is much hotter, it covers only a very small portion of the sky (90,000 suns would be required to cover the whole sky). A high intensity of radiation from the direction of the sun does not outweigh a small amount of radiation coming from the whole sky.”
Sure, but without an atmosphere, the surface temperature would be much higher during the day, right?
Anyway, clearly a high intensity of radiation form the sun does outweigh a small radiation from the whole sky, because the vast majority of the sky is only radiating because it has been heated by processes resulting from the heating of the surface by the sun. Am I correct?

rob
February 26, 2009 4:27 pm

I’m just an average joe here, but am I the only one that thinks that throughout the history of this planet, what we are doing now in this little tick of the geological clock is going to bring us all to ruination?
I do know that the climate of the Earth is changing constantly, and that it runs in cycles. Isn’t it the height of human hubris to think that we are changing the weather on this planet by accident while governments, countless scientists, and thinkers have been trying for years to change it on purpose?
From what I’ve seen, CO2 FOLLOWS climate warming; it isn’t the cause of it.
One element to the man-made global warming argument that gets no play whatsoever is the truth that man is a part of nature. The current argument seems to state that man is an alien invader to nature. In fact, we are PART of nature, not the controller of nature. Who is to say that our existence here isn’t as it is intended? Why is this a bad thing?

MikeE
February 26, 2009 4:29 pm

stas peterson (15:53:36) :
Heres a link too Miskolczi’s paper, http://met.hu/doc/idojaras/vol111001_01.pdf
For anyone who cares for a read, heres an article i also found under google on it http://constitutionallyright.com/2008/03/11/basic-greenhouse-equations-totally-wrong/
Im not qualified too comment 😉

Steven Goddard
February 26, 2009 4:40 pm

I don’t remember using the word “catastrophe” or “catastrophic” in the article. I thought I was just writing a high level description of how the greenhouse effect works.
“Nobody expects the Spanish Inquisition”
– Michael Palin

EJ
February 26, 2009 4:46 pm

This may have already been addressed. I haven’t read all 270 posts, but; The article states “… it is generally agreed that…” a doubling of CO2 will cause a 1.2 C rise in temps.
As far as I know, there is no engineering style study with empirical data verifying this hypothesis. Simply vague references to obscure and ancient textbooks are where you wind up when you follow the sources for this claim.
This radiative science boils down to the quantum physicists. Until one tells me that this 1.2 C hypothesis is indeed correct, I will wait for some rigorous science.

February 26, 2009 4:58 pm

Leif Svalgaard (11:58:35) :
Phil. (09:56:18) :
near the surface any energy is exchanged via collisions with neighboring molecules almost immediately (timescale less than a nanosecond).
Where do people get these ideas from? Googling the internet? How about some thinking?

Try teaching ‘Physics of Gases’ at the graduate level for over 10 years!
Any CO2 molecule which absorbs an IR photon will be rotationally/vibrationally excited, the radiation lifetime of that excited state is order microseconds or greater whereas the average time between collisions is less than 1 nanosec. Consequently the excitation energy will be preferentially lost via collisions (quenching). At higher altitude (lower pressure) the lower collision frequency means that the possibility of emission of a photon increases, hence fig 3 above.

foinavon
February 26, 2009 5:01 pm

David Porter (14:17:32) :
foinavon (10:14:41) :

As usual you disparage all the previous theories and replace them with the newly concocted papers from the early 2000 to 2008. That makes me very suspicious and it should make you suspicious also. The fact that it doesn’t, speaks volumes to me. You should ask yourself how the font of all knowledge appears only in the last eight to nine years.

Not really David. It’s not really about “theories”. It’s about the evidence, and the evidence certainly doesn’t come from just the past 8 years. The evidence for a climate sensitivity around 3 oC (temp rise per doubling of atmospheric CO2) goes back a long way (at least 20 years in the “modern” era, and even in the 1960’s the climate sensitivities were estimated to be in the range 1 – 4.5 oC). Callendar determined a value of 2 oC in 1938. The large majority of measures of climate sensitivity determined/estimated in the last 40 years have been in the range 1.5 – 6 oC (the top limit has always been difficult to pin down), and the more recent analyses just help to confirm the nature of the evidence and to narrow down the range to a higher likelihood that it’s between 2 – 4.5 oC at equilibrium.
So if we’re going to consider the climate sensitivity we really should consider the evidence as it stands. It doesn’t seem very professional to use as an illustration of climate sensitivity, a plot from a website called “junk science” which shows some rather odd data (the Charnock-Shine data seems to be incorrect as assessed by what Charnock-Shine actually reported….the Kondratjew and Moskalenko data is from an obscure 25 year old Russian report and the Lindzen data uses asumptions that we know are incorrect).
I can’t understand how you can be suspicious of newish science. Especially in reviews that address a compilation and synthesis of all the evidence, one expects that the most recent work is likely to be at least as good as earlier work and likely to conform to an increase in our understanding. Of course very recent single papers that might be provocative or especially novel, might well be looked at with reasonable skepticism until their essential points are confirmed. But that’s certainly not the case with the climate sensitivity.
…and as you can see I haven’t chosen to include Hansen’s interpretation of a rather high climate sensitivity (around 6 oC) that he and colleagues consider likely in relation to very long scale feedbacks involving the oceans and ice sheet dynamics…

Steven Goddard
February 26, 2009 5:03 pm

EJ,
I believe that the IPCC says ~1.2C for non-feedback CO2 doubling, and that John Christy uses that number as well.

Bill Illis
February 26, 2009 5:07 pm

I think everyone could benefit from understanding the greenhouse effect operates at a timescale of hours. It operates at the speed of light and at the speed of quantum physics.
It is just a delay of several hours in the time in takes for the Sun’s energy from the current day and the previous day to escape into space.
Last night, the temperature in my backyard dropped 12C in less than 12 hours – a third of the greenhouse effect. If the Sun stopped for three days, it would be -100C in my backyard. The greenhouse effect is extremely limited in timescale.
There are some other lags in the climate system. The peak of the summer is 30 days after the summer solstice. The atmosphere and land accumulates / loses about 0.5 W / m2 per day as the season’s change. The peak of surface ocean temperatures is 80 days after the solstice. The surface ocean accumulates / loses about 0.2 W / m2 as the season’s change. The deep ocean can accumulate 0.1 W / m2 over long timescales lasting 800 to 1,500 years.
But the greenhouse effect is just a 12 hour delay in how long it takes for a photon of energy received from the Sun to escape into space.
A visible light spectrum photon comes in from the Sun at noon and hits a rock on the beach. The rock warms by one photon.
Overnight, that rock cools off and gives off that photon in the IR spectrum. A new CO2 molecule in the atmosphere produced the day before by a near-by coal-fired power plant now intercepts that photon whereas before that photon would have just gone right through the atmosphere and hit Venus a few minutes later.
An electron in that CO2 molecule then moves to a higher energy state for a picosecond and decides it doesn’t like the new higher energy state and it gives up that photon – one-quarter of the time downward, one-quarter of the time upward and one-half of the time sideways. For the picosecond of time that this CO2 molecule is at a higher energy state, the measured temperature of the atmosphere is now higher – one photon’s worth that is.
That photon then happily skips around from N2 molecule to O2 molecule to another CO2 molecule and so on. The measured temperature is one photon higher but it only spends a few hours in the atmosphere or back on the ground before it is sent flying off to Mars or Jupiter or to the rest of the Milky Way.
This happens on the scale of trillions and trillions of photons in various different wavelengths and through trillions and trillions of molecules in the air and on the ground which intercept photons of light at different wavelengths. But it happens at the speed of light and at the speed of quantum physics.
It is so complicated that only a very detailed climate model can hope to possibly simulate it. Even then, it might be impossible. If it impossible, then we need to rely on empirical data instead which should be able to tell us what really happens (versus just guessing at what really happens). This is my main focus.
What does the empiricial data really say since the entire system is just too complicated to model or to describe in formulae.

Joel Shore
February 26, 2009 5:08 pm

gary gulrud says:

““The surface of the Earth is warmer than it would be in the absence of an atmosphere because it receives energy from two sources: the Sun and the atmosphere.””
Boo, Joel!

Gary, as I noted this statement comes from the website that I linked to, which is by a retired meteorology professor who is sort of a militant in terms of correct pedagogy. He likes the statement because it avoids saying anything technically incorrect. (For example, he doesn’t like saying the atmosphere re-radiates the IR it absorbs because he argues the act of radiating is independent of the act of absorbing. It radiates simply because it has a non-zero temperature.)
I have mixed feelings about his statement myself though because the price that you pay for avoiding saying anything that is at all a shorthand and thus technically incorrect is that the statement does not give a very holistic view. (E.g., it begs the question of where the energy came from in the first place that raised the atmosphere up to a finite temperature.)
At any rate, these are all really pedagogical issues. I have no disagreements with him on the actual physics of what is involved.

foinavon
February 26, 2009 5:10 pm

maksimovich (12:22:58) :

foinavon (10:14:41) :
Kondratyev and Moskalenko 1983, 1984 These are in Russian,though translated extracts are in Houghton 1985

Many thanks for that maksimovich!

Joel Shore
February 26, 2009 5:20 pm

Richard says:

Can anyone explain how the man-made portion of atmospheric CO2 is responsible for GLOBAL temperature change? The UN estimate of man-made CO2 is: 0.000348% by volume: “Mass of Global Atmospheric Gases” chart.
http://lce.folc.ca/2008/08/03/man-made-global-co2-emissions/
Then, factoring CO2 absorbtion of IR of about 8% spectrum yields the volume of atmosphere with sensitivity to thermal absorbtion is 0.00002785. Or, what have I done wrong?

Well, I count a few things. First of all, your numbers are confused. The fractional amount of CO2 in atmosphere by volume is ~0.000385, which is 0.0385%. Since it was ~0.0280% in pre-industrial times, we have raised the level by ~0.010%.
Now, that may not sound like much to you, but your intuition on this is bad for a couple of reasons: One is that 99% of the atmosphere consists of diatomic molecules (N2 and O2) that are transparent to infrared radiation. So, the remaining 1% play a disproportionate role in the climate. A second is that over a large range of concentrations, the radiative effect of CO2 is logarithmic in concentration, which again means that small amounts can play a disproportionate role.

Tom
February 26, 2009 5:21 pm

Joel Shore,
At the beginning of my comments, I gave what I believe to be the correct definition of positive feedback, namely an internal amplification factor greater than 1. If you plug this number into any equation describing a feedback system, it will give you a runaway system. You say that my definition is not correct, but you fail to give an alternative definition and use a hand waiving argument about the positive feedback being “weak” or ” strong”. I am willing to listen to your argument, but only if you give me an objective and consistent definition of positive feedback, which will give you an enhanced response without runaway. I do not think that such system exists, in the climate, or anywhere else.

February 26, 2009 5:23 pm

foinavon opines that “…the Lindzen data uses asumptions that we know are incorrect.”
Well now. Who are we to believe? foinavon? Or the head of M.I.T.’s Atmospheric Sciences department?
And Bill Illis, thanks for that explanation, which shows that there is simply a short delay when a CO2 molecule absorbs and re-emits a photon, rather than a runaway buildup of global heat.

Jeff Alberts
February 26, 2009 5:26 pm

rob (16:27:20) :
I’m just an average joe here, but am I the only one that thinks that throughout the history of this planet, what we are doing now in this little tick of the geological clock is going to bring us all to ruination?

Yes, you are.

rickM
February 26, 2009 5:26 pm

“As Dr. Hansen has correctly argued, increases in atmospheric temperature cause the ocean to warm up”
I suppose I should throw out all the oceanographic courses, knowledge and the limited experience I have. With this statement, everything I’ve observed has been wrong….
I just don’t ever recall the water column, to any significant depth, to be affected in the way Dr Hansen portrays.
Larry – nice link. Dr Stevenson was a giant in the field. The ocean systems and mitigation effects on the the underpinnings of the theory of AGW have been vastly underestimated. The effects of the atmosphere, and by CO2 in particular, on the ocean sysytems has been vastly overestimated.

Alex` Heyworth
February 26, 2009 5:26 pm

James Griffiths
“Sure, but without an atmosphere, the surface temperature would be much higher during the day, right?
Anyway, clearly a high intensity of radiation form the sun does outweigh a small radiation from the whole sky, because the vast majority of the sky is only radiating because it has been heated by processes resulting from the heating of the surface by the sun. Am I correct?”
In a word, no. The earth’s mean temperature without an atmosphere would be -18C. The sky radiates because it has been heated by both the sun and the earth’s surface.

Bart Nielsen
February 26, 2009 5:28 pm

Great article. Small quibble: in your list of important green house gases you list N2O. Should this be NO2?

Jeff Alberts
February 26, 2009 5:29 pm

Steven Goddard (16:40:25) :
I don’t remember using the word “catastrophe” or “catastrophic” in the article. I thought I was just writing a high level description of how the greenhouse effect works.

And that’s how this whole thing got started…

February 26, 2009 5:31 pm

Phil. (16:58:21) :
Any CO2 molecule which absorbs an IR photon will be rotationally/vibrationally excited, the radiation lifetime of that excited state is order microseconds or greater whereas the average time between collisions is less than 1 nanosec.
Even going along with that and that the N2 and O2 take up the heat, they in turn must re-radiate that heat [because they are now warmer], half up and half down [which then eventually heats the surface. Nothing changes with that, you still get a greenhouse effect.

Alex` Heyworth
February 26, 2009 5:34 pm

Simon Evans, I agree with what you say. However, to put it in context, the whole history of life on earth is constant change. The whole history of mankind is conflict and adaptation to change. Plus ça change, plus c’est la meme chose.

Joel Shore
February 26, 2009 5:36 pm

DR says:

Joel Shore,
Please explain this statement via Gavin Schmidt et al 2005:
http://www.osti.gov/energycitations/servlets/purl/881407-xk2Sdg/881407.PDF
“Tropospheric warming is a robust feature of climate model simulations driven by historical increases in greenhouse gases (1–3). Maximum warming is predicted to occur in the middle and upper tropical troposphere.”

Well, his quote is not incorrect but it is subject to misinterpretation. It is indeed a robust feature of climate model simulations. However, it is true that such warming is a robust feature of those simulations whether the warming is driven by increases in greenhouse gases or something else.
Indeed, if you actually read that whole paper rather than just quoting a single sentence, you would learn that one of their major points is that the amplification applies not only to the long-term temperature trends due to rising greenhouse gases but even to shorter-term temperature FLUCTUATIONS (such as those that occur due to ENSO). And, they point out in that paper, that this amplification of the fluctuations is seen observationally in all the data sets, which tends to confirm that the models are handling the basic features that produce this (convection) at least approximately correctly. It is only when one looks at the multidecadal trends that one sees significant discrepancy between the models and (many of) the observational data sets. Since these trend measurements are very sensitive to artifacts that are known to exist in the data, this is why they suspect that the problem is most likely to lie with the data.
To summarize it more succintly: The models and observations agree over the timescales for which the data is reliable but the observations disagree both amongst themselves and (in many cases) with the models over the timescales for which the data is known to have serious issues.

Richard
February 26, 2009 5:44 pm

Roger Knights (14:05:43) :
OT: Bloomberg story: “Obama Plan Has $79 Billion From Cap-and-Trade in 2010″
(Moderator: Snip if too off.) One reason for this dialog is to avoid a REAL catastrophe from fear-based labeling of CO2 as a “pollutant.” With piles upon piles of evidence that the biosphere has survived and excelled in higher CO2 environs and FACE studies showing unarguable evidence for CO2 fertilization – taxing this trace gas that enhances plant growth is tantamount to calling a dyslexic the spawn of the devil.

DR
February 26, 2009 5:45 pm

Joel Shore,
You did not reply to my post on Gavin Schmidt et al 2005 quoted thusly:
“Tropospheric warming is a robust feature of climate model simulations driven
by historical increases in greenhouse gases (1–3). Maximum warming is predicted
to occur in the middle and upper tropical troposphere.”
http://www.osti.gov/energycitations/servlets/purl/881407-xk2Sdg/881407.PDF
Your statement, and those being bandied about the various pro-AGW blogs are now back pedaling. With that, you are saying GCM predictions are mistaken.
If your version is correct, why then have Schmidt, Santer and the usual suspects been attempting to a) discredit satellite data and b) attempt to show models in agreement with observations? Surely you are aware of the serious “problems” with Santer et al 2008?
You can’t have it both ways. Are GCM predictions, which are driven by increases in GHG emissions, correct or not?

Paddy
February 26, 2009 5:45 pm

Assuming that the AGW hypothesis is correct, should not Mars (atmosphere 90% CO2) be a tropical paradise?

Larry Kirk
February 26, 2009 6:01 pm

Re: Steven Goddard (05:51:38) :
“Ric,
I did a canoe trip down the Green River in Utah about 25 years ago in July. It was too hot to sleep when you went to bed, and too cold to sleep when you woke up!”
Sorry to go a bit OT Steven, but your account of this just fascinated me. I spent 6 months in a camp on the banks of Mahat River in Central Sumatra ten years ago: in a mosquito-infested jungle just north of the equator, 100 Km inland and only 150m above sea level. It was almost too hot and humid to survive by day, and almost impossible to sleep at night, but then every morning at about 2.00am a freezing wind came though the camp and we shivered until sunrise. After about a week, I finally realised what was happening at night: a river of freezing cold air was flowing down the topography from the 6,000ft mountains to the west of us, eventually converging into the main valley that we were camped on and eventually hitting us like a flash flood about eight hours after sunset. Obviously the same thing happens on the Green River in Utah, and from considerably colder highlands at its source.
Do people usually take these topographic effects into account when discussing night-time temperatures? If they don’t, they should.

Elvis M
February 26, 2009 6:13 pm

Joel Shore (15:08:14) :
“The global climate models are programmed with the best understanding of the actual physical processes involved. ”
Since much of AGW theory is promulgated via these GC models – do we have independent analysis of the programming procedures and details? I understand that many of these details are unpublished or redacted under claim of proprietary interest. Considering that much climate science is conducted under grants of public funds – the code and algorithms utilized should be transparent to scrutiny by third parties.

Alex` Heyworth
February 26, 2009 6:16 pm

Paddy (17:45:06) :
Assuming that the AGW hypothesis is correct, should not Mars (atmosphere 90% CO2) be a tropical paradise?
No. The atmosphere on Mars is too thin for any significant “greenhouse effect”.

Willis Eschenbach
February 26, 2009 6:32 pm

Steve Goddard, an interesting explanation. However, you go a bridge too far when you say:

It has been known for a long time that even a short column of air contains enough CO2 to saturate LW absorption. This has been misinterpreted by some skeptics to mean that adding more CO2 will not increase the temperature. That is simply not true, as higher GHG densities force the temperature up. There is no dispute about this in the scientific community.

Absolutely not. There is agreement in the scientific community that increasing GHGs will increase the net downwelling radiation. However, whether and how this will affect the temperature is an open question.
An example I have used before is this: suppose we take a 75kg block of say copper, and stick one end into hot water. After a while, the heat is transferred to the other end of the copper block. I propose a theory that if you stick one end of something into a block of hot water, the other end will heat up at a certain rate. Simple physics, no problem. I try a block of wood. I notice that it works just the same, except it heats up more slowly. I try a steel block, same thing, it heats up, just takes a different time.
Finally, having proven my theory, I decide to test it on myself. I put my feet into the hot water, and I wait for my head to heat up … and wait … and wait.
The moral is, complex systems don’t obey simple physics.
The same is true about the climate. Everyone agrees that more GHGs will lead to more downwelling radiation, just as putting something in hot water leads to increased heat transfer to the object.
But as my example shows, this may have a) a larger effect than predicted, or b) a smaller effect than predicted, or c) no effect at all.
Nor is this a simple matter of getting the feedback right. In my opinion, the earth has an active system to keep it from getting too hot or too cold (thunderstorms, for those interested). This is why as far as we know, for the last half billion years or so, the earth’s temperature has only changed about ±2% …
As a result, changing the CO2 might lead to no change at all, not because of feedback, but for the same reason that putting my feet in hot water doesn’t heat up my head. It may not change because in both cases there is an active system which works to keep the temperature stable.
So in fact there is little scientific agreement on whether, and by how much, a change in CO2 will change the temperature of the earth, even in the absence of feedbacks.
w.

vivendi
February 26, 2009 6:35 pm

Simon Evans (16:02:27) :
Scientists were considering positive feedbacks as an explanation in advance of discovering the relationship between temperature and CO2. When the Vostok cores were pulled up, by 1985, the correlation was evident..

Care to explain a little more? I still don’t see how this explains the lag between temperature change and CO2 change. Is CO2 the driver or the follower?

Jerry Lee Davis
February 26, 2009 6:36 pm

Steve and Anthony
I am enjoying Steve’s post as well as the comments I’ve been able to read so far. I apologize if the question I’m about to ask has already been posed and discussed. The question is:
Why is there NOT a bona-fide greenhouse effect in the Earth’s atmosphere?
The main functions of a real greenhouse are to ensure that the air inside gets warmed by the sun and then kept warm by preventing its escape or mixing with outside air. I believe that the non-greenhouse gases in Earth’s atmosphere perform these functions as well. They are warmed by coming in contact with the materials at the Earth’s surface (independently of radiation), participate in convection (independently of radiation), and tend to stay warm because they cannot cool via radiation. Atmospheric air does not escape or mix with alien air. Gravity plays the role of glass panes.
I have tried to estimate the internal energy contained in atmospheric nitrogen, oxygen, and argon (actually argon is lost in round-off errors). Assuming an atmosphere at 0 degrees C, and using the relation U = 1.5 nRT, where U is internal energy, n is the number of moles, R is the universal gas constant, and T is temperature in Kelvin, then the atmosphere contains about 1.7E+17 kW hrs of internal energy.
That figure could be wrong, but if it’s close then it’s roughly the same as:
1) The amount of energy that would be consumed by the human race in 1250 years, using the 2006 worldwide energy consumption rate given by the EIA at http://www.eia.doe.gov/pub/international/iealf/tablee1.xls, or
2) The amount of solar energy intercepted (and not reflected) by the Earth in about two months.
So here we are back to my question: How can it be that all that energy does NOT participate in keeping us warm at least a little bit via a good old-fashioned greenhouse effect like Grandma used to use?

Steven Goddard
February 26, 2009 6:41 pm

Larry Kirk,
Interesting memories of Sumatra. Normally downslope winds are quite warm, due to adiabatic heating, so there must have been something else going on.
In the case of Canyonlands, the air was very still and most of the heat loss was due to heat stored in the rock radiating rapidly out into the dry night air. Of course it was colder at the bottom of the canyon because cold air sinks, but even up 20 or 30 meters it was still quite cold at night, and over 100F during the day.

Steven Goddard
February 26, 2009 6:41 pm

Paddy,
The Martian atmosphere has very little water vapor, which is the most important greenhouse gas.

E.M.Smith
Editor
February 26, 2009 6:43 pm

I find that spectral cooling chart fascinating. Is there a similar one for polar latitudes mid winter? The ozone, in particular. H2O is already going low in its impact at the ozone frequencies. In a cold polar winter, water ought to be even less of an impact.
So maybe this gives a mechanism for the sun modulating heat loss. Solar activity drives ozone (both up via UV and maybe charged particles (EFT); and down via GCR modulation). So there is this 10nm or so window controlled by the sun that waffles back and forth by 50% or so. And here we are stressing over a window at 7.5 that’s already well covered by water…
I know, it’s pure speculation. But just take a minute to look at that chart and ask yourself what happens as ozone wanders back and forth by 50 percent…
http://exp-studies.tor.ec.gc.ca/e/ozone/Curr_allmap_g.htm
doing a visual integration of the deviation chart further down the page I get about a 10% to 15% negative anomaly for the globe right now. It was even lower a few weeks ago when, surprise surprise, it was colder and snowing all over the place…
like this from January 3, 2009:
http://exp-studies.tor.ec.gc.ca/cgi-bin/selectMap?lang=e&type1=de&day1=03&month1=01&year1=2009&howmany1=1&interval1=1&intervalunit1=d&hem1=g&type2=no&day2=25&month2=02&year2=2009&howmany2=1&interval2=1&intervalunit2=d&hem2=n&mapsize=100
So I’m just left to wonder…

coaldust
February 26, 2009 6:44 pm

Joel Shore (11:12:12) :
Firstly, the sun has gradually brightened over time.
Yes. This is a system input. The fact that the system has not responded with runaway feedback over the variation of the sun’s input suggests the system is stable, not unstable.
Hansen argues that some negative feedbacks that operate at geologic timescales (mainly involving the GHGs themselves, I believe) could be inoperable here due to the very fast pace with which we are increasing the GHGs.
There are observations of past CO2 increases that were faster than todays. This line of reasoning is DOA.
But this begs the question, what is the water vapor/energy feedback relationship. Data analyzed by Roy Spencer indicates negative feedback. It suggests the system is not only stable, but resists change.
Furthermore, if water vapor feedback is negative, then the sensitivity is less than widely proclaimed 1 C per doubling CO2, and there is no cause for alarm. This is why I am a skeptic.

wirote
February 26, 2009 6:45 pm

Today,we can save the world by reduce used something its harmful such as co2 from our car,reduce used oil and gas and reduce everything that make gobal warming.

February 26, 2009 6:46 pm

Joel,
“The models and observations agree over the timescales for which the data is reliable but the observations disagree both amongst themselves and (in many cases) with the models over the timescales for which the data is known to have serious issues.”
So you’re saying models are in good agreement with, say, the last 10 years of temperature data?
I’m not sure that a statement such as yours, which indicates that when the models are wrong, it’s because the data is bad and when the models are right, it’s because the data is good, has much scientific credibility in the long run. (Although there are sometimes valid reasons why scientists try to use arguments like these, when looked back upon historically.)

AJ Abrams
February 26, 2009 6:56 pm

Something was mentioned twice here that was never addressed by the normal folk that inhabit this site –
Pressure.
Does the temperature of Venus, Mars Jupiter atmosphere at 1ATM correlate to earth’s average temperature after adjusting for distance from the sun?
If it does, and I’ve now read two websites not related to climate science that say that it does, then the big…..OH…moment for me is that pressure regulates temperature more than any thing and that the ocean accounts for the rest of earth’s temperature variations over time – noting that the earths overall temperature doesn ‘t actually fluctuate by that much).
Also it says that SG’s comments about Venus and Mars are really misleading to say the least, which as a long time reader here, is disappointing.
Can someone clarify? Leif would be a prime candidate to answer because what I said would go along perfectly with his tireless shouts that IT’S NOT THE SUN! Which I believe

E.M.Smith
Editor
February 26, 2009 6:57 pm

Bill Yarber (23:24:20) : Your use of Mars, Earth and Venus to prove that CO2 putis a major green house gas is totally flawed. You completely neglect their diameters and their distance from the Sun. Venus is 2/3s the distance from the Sun than earth and less than 1/2 that of Mars.
Don’t know what the diameter has to do with it (other than helping to retain more gas on heavier planets).
I’d have used moon / earth. Same distance from the sun is the same. It sure does get cold on the moon at ‘night’. Not sure how to handle the very hot ‘days’… Would be interesting to see what the ‘average’ does…
http://www.asi.org/adb/m/03/05/average-temperatures.html
says that it’s -35c below the surface. Probably a decent proxy for average…
That would give about a 45C heat gain for us from our atmosphere and vulcanism. And any differential albedo effects. Have no idea if that matches what any theory would predict. But I have trouble thinking that if the effect of the entire atmosphere is about 45C we could get more than 4C out of minor changes in one trace component…
Ergo, since the watts/sqm of the Sun’s out decreased with the square of the distance from the Sun, Venus gets over twice the energy that Earth gets and four times the energy Mars gets. Venus is warmer because it has a denser atmosphere and gets twice the energy Earth gets. This would be true if Venus had no CO2 in its atmosphere. Each planet has a different diameter but the difference is insignificant when compared to the impact of their individual orbit radii.
Bill

E.M.Smith
Editor
February 26, 2009 7:01 pm

That ought to be (without Bill’s stuff at the bottom):
Bill Yarber (23:24:20) : Your use of Mars, Earth and Venus to prove that CO2 putis a major green house gas is totally flawed. You completely neglect their diameters and their distance from the Sun. Venus is 2/3s the distance from the Sun than earth and less than 1/2 that of Mars.
Don’t know what the diameter has to do with it (other than helping to retain more gas on heavier planets).
I’d have used moon / earth. Same distance from the sun is the same. It sure does get cold on the moon at ‘night’. Not sure how to handle the very hot ‘days’… Would be interesting to see what the ‘average’ does…
http://www.asi.org/adb/m/03/05/average-temperatures.html
says that it’s -35c below the surface. Probably a decent proxy for average…
That would give about a 45C heat gain for us from our atmosphere and vulcanism. And any differential albedo effects. Have no idea if that matches what any theory would predict. But I have trouble thinking that if the effect of the entire atmosphere is about 45C we could get more than 4C out of minor changes in one trace component…

February 26, 2009 7:29 pm

Leif Svalgaard (17:31:15) :
Phil. (16:58:21) :
“Any CO2 molecule which absorbs an IR photon will be rotationally/vibrationally excited, the radiation lifetime of that excited state is order microseconds or greater whereas the average time between collisions is less than 1 nanosec. ”
Even going along with that and that the N2 and O2 take up the heat, they in turn must re-radiate that heat [because they are now warmer], half up and half down [which then eventually heats the surface. Nothing changes with that, you still get a greenhouse effect.
Sorry N2 and O2 are homonuclear diatomics and don’t radiate.

jae
February 26, 2009 7:29 pm

Willis, 18:32:47:
I salute you again! LOL. We don’t know as much as we think we do. I’m going to go stick my feet in some hot water and check your facts…. 🙂

E.M.Smith
Editor
February 26, 2009 7:46 pm

Fredrik Malmqvist (00:30:11) : Nils Bohr showed that gas molecules that absorb light are exited to a energy level and can only give off this energy with light at the same wavelength.
Um, I thought it was a bit more subtile than that. That two quanta could be absorbed, then emitted as one of higher energy, or that one could be absorbed, then emitted as two of lower energy, etc. Isn’t that how laser pumping works? Pump with UV, get IR laser light out, for example? And the exact quanta that can be absorbed are determined by the energy bands available for occupation in the electron shells of the atoms in question.
Or am I mixing up a long ago memory of physics that has moved on since they first discovered lasers…
(I also don’t know if this nit is relevant in the atmosphere at normal sunshine levels… just trying to find out if I’ve still got a grip on the physics…)

Joel Shore
February 26, 2009 8:07 pm

Will Nitschke says:

So you’re saying models are in good agreement with, say, the last 10 years of temperature data?

First of all, you are taking a statement out-of-context that I was using to talk about one issue in particular, the issue of tropical tropospheric amplification of temperature trends and fluctuations. Second of all, 10 years of data is insufficient to determine agreement or disagreement with the models for the global temperature trends because the error bar in the trend over such a time period is huge. Runs of climate models indeed confirm that one can expect some non-negligible fraction of periods of this length to show negative trends.

I’m not sure that a statement such as yours, which indicates that when the models are wrong, it’s because the data is bad and when the models are right, it’s because the data is good, has much scientific credibility in the long run. (Although there are sometimes valid reasons why scientists try to use arguments like these, when looked back upon historically.)

As I explained, there are independent reasons to believe that the data is questionable for the trends on the multidecadal timescales, not the least of which is that the two analyses of the same satellite data by two different groups yield very different results, one of which is essentially in agreement with the models within error bars and one of which is not. In general, one there is disagreement between observational data and models or theory, there is no hard-and-fast rule about which is right and which is wrong. It indicates the need for more work to try to resolve the discrepancy.

Steven Goddard
February 26, 2009 8:08 pm

People are passing along all kinds of great ideas about feedbacks and second order effects which could reduce heating. Thanks for that.
But none of that changes the base properties of greenhouse gas warming.

Jack Linard
February 26, 2009 8:11 pm

Brief comments: from a hydro engineer involved in climate issues for 40 years:
– outstanding comments from peter taylor – I’m copying this for further consideration
– generally disappointing post with too many assertions and too little justification
– why is it hotter in the summer than it is in the countryP

juan
February 26, 2009 8:14 pm

re: John Peter
There is something I must be missing. The first page of your link indicates testimony before Sen. Boxer’s committee yesterday, February 26. But all the internal pages show a date of July 10, 2002.

Jack Linard
February 26, 2009 8:21 pm

Oh yeah, I forgot.
Every one of the analogies presented in this thread are pointless and irritating. Using analogies to discuss climate issues is like using a pincushion to deflate a pelican.
Personally, I’m getting very tired of the smug and sanctimonious comments from smart-ass know-it-all Lief

Joel Shore
February 26, 2009 8:22 pm

coaldust says:

There are observations of past CO2 increases that were faster than todays. This line of reasoning is DOA.

I don’t want to spend a lot of time defending Hansen’s claim since I myself am skeptical of it and want to see his reasoning spelled out in more detail. However, I am also very skeptical of your claim about the speed of past CO2 increases. I don’t think there are faster increases in the ice core record and I don’t think that we have data resolved well enough in time before that to make such a conclusion.

But this begs the question, what is the water vapor/energy feedback relationship. Data analyzed by Roy Spencer indicates negative feedback. It suggests the system is not only stable, but resists change.
Furthermore, if water vapor feedback is negative, then the sensitivity is less than widely proclaimed 1 C per doubling CO2, and there is no cause for alarm. This is why I am a skeptic.

First of all, let’s get our facts straight. Spencer is no longer questioning whether the water vapor feedback is negative. He now admits it is probably positive. What he is questioning is the cloud feedback. See his post here for explanation: http://www.drroyspencer.com/2009/02/what-about-the-clouds-andy/
Second of all, Spencer is just one scientist. You can always find a scientist to support just about any view if you look hard enough. (Actually, for the case of intelligent design, you need look no further than Spencer himself: http://www.tcsdaily.com/article.aspx?id=080805I ) And, Spencer has made some pretty bad errors in analysis when trying to get the answer that he seems to have a predisposition to want to believe (see here: http://tamino.wordpress.com/2009/01/19/a-bag-of-hammers/ ) Furthermore, there are lots of things that would need to be explained if Spencer is correct – i.e., all the observational evidence we have, e.g., paleoclimate data as well as the temperature response to the Mt Pinatubo eruption, that suggest that the climate sensitivity is in the range given by the IPCC. In fact, it becomes very hard to explain even the ice age – interglacial cycles.
If you were really a skeptic in the broader sense, you would be more skeptical of Spencer’s claims.

Pamela Gray
February 26, 2009 8:28 pm

I like the feet in hot water experiment. That makes sense. It could also explain why a plausible CO2 mechanism does not pan out as a predictive closed model residing in a computer when compared to in-situ conditions and observations. The Earth is not capped by a firmament. It is an open, interconnected chaotic system. CO2 and its affects on water vapor, thus temperature, may indeed work one way in a petre dish, so to speak, but may work quite differently in the womb of Earth.

Joel Shore
February 26, 2009 8:30 pm

Elvis M says:

Since much of AGW theory is promulgated via these GC models – do we have independent analysis of the programming procedures and details? I understand that many of these details are unpublished or redacted under claim of proprietary interest. Considering that much climate science is conducted under grants of public funds – the code and algorithms utilized should be transparent to scrutiny by third parties.

The best checks we have is the fact that there are some 10 – 20 different climate models floating around due to different groups of scientists. Many of them do not make their code publicly available. However, they do publish explanations of their algorithms and make data available for model intercomparisons. This is no different than any other field of the physical sciences that I’ve worked in. The NSF has been clear on the fact that use of their funds does not compel scientists to make their code freely available as they do retain their intellectual property rights over it.
This being said, the GISS Model E code is freely available and some others may be too.
It is also worth noting that the claim that AGW relies that heavily on the climate models, especially for the basic prediction of the magnitude of the warming expected, is overstated anyway. The IPCC’s estimate of the climate sensitivity is based mainly on observational data. In some cases, climate models are used to help interpret what sort of climate sensitivities that data implies…but they are not generally used directly to predict the climate sensitivity. (Although the range of climate sensitivities that exist in the current models do tend to mirror the range that is found to be most likely based on the observational data.)

Robert Bateman
February 26, 2009 8:38 pm

Sure am glad there is some sort of greenhouse gas mixture in our atmosphere.
Otherwise, we’d all be frozen solid at night and fried during the day.
Sure is nice to have that blanket to nightly retain at least some of that precious energy coming from the Sun to keep warm.

Robert Bateman
February 26, 2009 8:41 pm

Pamela: Have you ever considered taking rock temperatures 50 feet underground might tell us? They are known by miners (me included) to remain stable year round. If the Earth was warming or cooling over decades, this would surely show it.

AlexB
February 26, 2009 8:44 pm

RE: Steven Goddard (20:08:31) :
“People are passing along all kinds of great ideas about feedbacks and second order effects which could reduce heating. Thanks for that.
But none of that changes the base properties of greenhouse gas warming.”
Steve,
I posted earlier on vertical wind shear. Convective heat transfer is not a feedback and it is not a secondary order effect. It is a primary order effect and the dominant primary order effect at that. When things warm up they expand and what happens to warm air? Here’s another simpsons quote for you:
‘Heat makes metal expand? Now who’s talking mumbo jumbo?’
Bart Simpson

Just want truth...
February 26, 2009 8:47 pm

“If you want to make a knowledgeable argument, learn about the feedbacks.”
Two part YouTube video of Roy Spencer talking about feedbacks (Lindzen’s “Infrared Iris”) :
Why the IPCC models are wrong – Part 1
link

Why The IPCC models are wrong pt 2
link

February 26, 2009 9:04 pm

Hi Joel,
I wrote: “So you’re saying models are in good agreement with, say, the last 10 years of temperature data?”
You wrote: “First of all, you are taking a statement out-of-context that I was using to talk about one issue in particular, the issue of tropical tropospheric amplification of temperature trends and fluctuations.”
Why do you say my remark is out of context? You were talking about models being in agreement with troposopheric temperature trends and I asked why you thought they were in good agreement, and you then complained I’ve changed the subject…? Odd… Could you clarify please?
“Second of all, 10 years of data is insufficient to determine agreement or disagreement with the models for the global temperature trends because the error bar in the trend over such a time period is huge.”
But the Santer paper was accepted for publication so the data he included in his analysis is thought to be sufficient. If even more data is included (adding the last few years to his data set), using his own methods, the models are not shown to be in agreement with predictions. Sorry, but on what reasonable basis do support your claim that models are doing well and we should have confidence in them?
“Runs of climate models indeed confirm that one can expect some non-negligible fraction of periods of this length to show negative trends.”
Obviously, but the larger that non-negligible fraction gets, the bigger the probability of the discrepancy being attributable to things other than ‘random weather noise’.
“In general, one there is disagreement between observational data and models or theory, there is no hard-and-fast rule about which is right and which is wrong. It indicates the need for more work to try to resolve the discrepancy.”
Generally speaking, in other research fields, your predictions need to conform with the empirical data before they are taken seriously. String theory, models of planetary formation, etc., require empirical confirmation, not the other way around. Climatologists invoke special pleading here for their own research work it seems… But let’s agree to disagree on that point.
Joel, may I ask if you have any association with what certain sceptics have referred to as the ‘Hockey Team’ ? Just curious, thanks.

February 26, 2009 9:11 pm

My old friend Mr Shore said (09:01:02) :
“Diameter doesn’t matter.”
Please have a word with my doctor, Mr Shore, and stop him nagging me.

cableguy
February 26, 2009 9:11 pm

I am sorry if this has already been posted. http://arxiv.org/PS_cache/arxiv/pdf/0707/0707.1161v3.pdf

Jerry Lee Davis
February 26, 2009 9:14 pm

In re-reading my comments and question after a couple of hours, I concluded that I should clarify a bit. I did not mean to express doubt about the existence of the type of greenhouse effect Steve is discussing when I used the term “bona-fide greenhouse effect.” Rather, I meant to refer to the type of greenhouse effect referred to in IPCC’s AR4 Frequently Asked Question 1.3 in the first sentence of the following excerpt:
“The glass walls in a greenhouse reduce airflow and increase the temperature of the air inside. Analogously, but through a different physical process, the Earth’s greenhouse effect warms the surface of the planet.”
Also, I said “…then the atmosphere contains about 1.7E+17 kW hrs of internal energy.” but I should have said “…then atmospheric nitrogen, oxygen, and argon contain about 1.7E+17 kW hrs of internal energy.

len
February 26, 2009 9:15 pm

“Falsification of the Atmospheric CO2 Effect Within the Frame of Physics”
http://arxiv.org/PS_cache/arxiv/pdf/0707/0707.1161v3.pdf
Short Hand Supposition and Conjecture is rudimentary theorizing … which can lead to something but in itself is not science.

February 26, 2009 9:19 pm

Joel,
You wrote:
“It is also worth noting that the claim that AGW relies that heavily on the climate models, especially for the basic prediction of the magnitude of the warming expected, is overstated anyway.”
Sorry upon what other basis is AGW based, if not the models?
“The IPCC’s estimate of the climate sensitivity is based mainly on observational data.”
Oh please, be serious. 🙂
How does one disentangle solar activity, ocean heat oscilliations, GHG’s, lag times and latencies, feedbacks, etc., simply by reference to observational data without theories? And how can these theories tell us anything without instantiating them within computer models?
If the IPCC analysis was based on observational data, then the forecast would surely be closer to 1-1.5C as the previous century had a .7C temperature trend. The rest of the heat is theoretical or hypothetical. Nothing wrong with that. That’s how science is done. But please don’t pretend otherwise. 😉

Steven Goddard
February 26, 2009 9:56 pm

AlexB,
If you are suggesting that convection increases as a result of GHG warming, is that not a feedback?

E.M.Smith
Editor
February 26, 2009 9:57 pm

OT? Maybe not… gas related…
http://volcanism.wordpress.com/category/volcanoes/galeras/
has this quote:
The volcano’s SO2 output has been fluctuating, with an average of under 430 tonnes/day. This appears to be within the normal range for Galeras during an active phase: during a similar period of activity from December 2005 to January 2006 the sulphur dioxide flux varied from 300 to 1500 tonnes/day.
This is about the volcano in Columbia that’s gone active, not Chaiten in Chile (nor Redoubt in Alaska that keeps flirting but won’t … [oh self snip… but it would have been a great punchline…]=)
Is a kiloton a day a lot? It sure sounds big to me, especially given that these guys get going and stay going for months on end. It’s also looking a little like the long period of lower volcanic activity has headed into a more normal range in the last few years. To what extent have volcanic gas variations been figured into climate prediction, er, projection, er, model prognostication, um, computer musings?

February 26, 2009 9:58 pm

Robert Bateman (20:41:18) :
Have you ever considered taking rock temperatures 50 feet underground might tell us? [..] If the Earth was warming or cooling over decades, this would surely show it.
It does, here are the temperature anomalies over the past 2000 years deduced from such measurements: http://www.leif.org/research/T-Boreholes.png

February 26, 2009 10:00 pm

The facts that Al Gore’s carbon tax collection company doesn’t want you to see.
CO2 is roughly 3% of the total greenhouse effect. Around 0.2% is man-made CO2 greenhouse gas effect. CO2 is an insignificant greenhouse gas, especially in terms of man-made contribution. Water is the only significant greenhouse gas.
Furthermore, you leave out that oceans and volcanos release more CO2 than humans.
Furthermore, you leave out that CO2 is part of the life cycle.
Futhermore, you leave out that CO2 levels tend to follow warming rather than cause warming.
Furthermore, you leave out the impact of the sun spot activity, which is thoroughly studied, and it’s impact of warming/cooling on the solar system as a whole.
Furthermore, you leave out that the solar system as a whole had been warming.
Furthermore, you leave out that the solar system and earth has cooled down due to lack of sun spot activity this past year.
Furthermore, you leave out that carbon taxes will not solve the problem but will be used to fund global government and bombing of nations.
Furthermore, you leave out that big corporations, especially big oil, actually support the global warming hoax because it helps them eliminate their competition who can’t afford the taxes.
Furthermore, you leave out that carbon taxes will do nothing but hurt the poor.
Furthermore, you leave out that polar bears can easily swim over 100 miles and swim as much as 300 miles. They hunt for seals and buluga whales.
Furthermore, you leave out that the ice caps are always melting and refreezing with the seasons.
Furthermore, you leave out that the government was caught making up temperature data to make it seem the planet was warming rather than cooling this past year.
Furthermore, you leave out that scientistics receive tons of funding by promoting global warming scares while skeptics don’t receive funding.
Furthermore, you leave out that the Al Gore hockey stick graph was generated using a statistical analysis computer algorithm that weighted data towards the mean the further out in history you go.
Furthermore, you leave out all the various drastic cooling and warming trends throughout the history of the Earth.
Furthermore, you leave out that global warming is actually preferred over global cooling because food production increases, namely because CO2 helps plant growth, and since you’re more likely to freeze to death than die of heat stroke.
Furthermore, statistics are lies, lies, and damned lies. Scientific models don’t represent reality. Any decent college professor in scientific modeling and statistics classes will tell you these things on the first day of class.
In conclusion, you’re being scammed by the New World Order with this global warming hoax to pay carbon taxes to them, and you should be mad as hell.

Steven Goddard
February 26, 2009 10:04 pm

Jerry Lee Davis,
The burning of fossil fuels does provide direct greenhouse warmth. In buildings and in in people’s homes!

Richard111
February 26, 2009 10:07 pm

Coal, oil, gas, fossil fuels are running out, right?
They originate from CO2, right?
Then we better get busy producing more CO2 so future generations have some fuel for their economies.

Chico
February 26, 2009 10:08 pm

Finally a discussion on the physics of the Greenhouse Effect. I have a few observations and questions.
1) Steve, statements like: “These are undisputed facts” and “There is no dispute about this in the scientific community” does nothing to help me understand the physics of first order, non feedback Greenhouse effect. When a Physicist explains Relativity, they never leave it at “Scientific consensus”, they explain experiments that prove predictions of Relativity such as the optical displacement of Mercury when viewed next to the Sun. Where are the experiments that demonstrate the greenhouse effect?
2) Phil’s Makes a very powerful point that an excited CO2 molecule will almost certainly transfer its ‘excess’ energy to surrounding O2 and N2 molecules. This is *very* important because O2 and N2 don’t principally radiate at a frequency that is absorbed by traditional Greenhouse gases. Correct me if I am wrong but it is O2 microwave radiation that satellites observe to infer the temperature of the Earth’s atmosphere (i.e. RSS and UAH). Although Phil also suggests that O2 and N2 don’t radiate? I thought all mater above absolute zero temperature radiates?
3) If my interpretation of Phil’s comments are correct then radiative energy transfer in the lower atmosphere should be negligible next to convective energy transfer. So the Greenhouse effect should be working at an altitude where radiative transfer becomes substantial. What is the predicted altitude?
4) Heat flow from surface of the earth to space occurs by, radiation, conduction, convection and phase change of H20. Since we are unable to know perfectly the breakdown of heat transfer in the atmosphere by these 4 mechanisms, lets just consider heat flow in the aggregate. I remain unconvinced that an increase from 250ppm of CO2 to 500ppm could possibly lead to a measurable increase in temperature. If so why isn’t CO2 the greatest insulator know to man? why do they fill insulated windows with expensive Argon and not cheap CO2?
5) I remain unconvinced that the radiative absorption/emission properties of a gas are correlated with gas’s thermal insulative properties. At minimum the greenhouse effect, as popularly described, requires that spectral properties of a gas be considered in isolation and seperate from other properties that may be pertinent to energy flows through the gas such as molecular weight, specific heat capacity, thermal conductivity and others.
chico sajovic

tmtisfree
February 26, 2009 10:15 pm

Peter (12:30:54) :
1/ The re-radiation doesn’t additionally warm the surface, but rather slows down the rate at which the surface loses heat – thereby keeping it warmer for longer than it would otherwise have been.
Ok. That means, as J. Honda pointed out, that surface emits less radiation per unit of time.
2/ However, it only slows down the rate at which the surface loses heat by radiation
This implies that the total number of CO² molecules absorbing radiation is less per unit of time, as is the number of CO² molecules de-excited in kinetic energy/heat thus resulting in a cooler atmosphere per unit of time. And it is normal as it compensates for the energy of the LW photon just absorbed by the surface: the energy of the system cannot increase.
3/ – it doesn’t affect the rate of heat loss from conduction, convection or evaporation.
So if the surface is losing heat at the same rate, that implies that atmosphere is gaining heat at the same rate with these processes. No change here.
The total is that, when the system is stable, the atmosphere is cooler per unit of time due to the facts of:
a) LW photon absorbed by surface and less radiation emitted by surface
b) less absorption by CO², less kinetic energy/heat.
When solar irradiation ends, the atmosphere is still cooler (because of 2/) and as seen in 1/, the surface is still warmer and thus continues to lose its heat in the atmosphere (by processes in 3/).
But as the surface has absorbed LW photons and the energy of the system can not increase, the atmosphere will still be a little cooler (less kinetic energy) that it was at the beginning [the rate at which the surface lost heat by radiation (or energy as E=hv) had slowed down].
So finally, the surface has been a little longer warmer and the atmosphere a little longer cooler.
This is not intuitive.
A question: in which form of energy is transformed the LW photon absorbed by surface (not kinetic energy/heat apparently)?
Bye,
TMTisFree

Reed Coray
February 26, 2009 10:32 pm

I thank Simon Evans (16:02:27) for responding to my post (14:56:21). I would like to respond to Simon’s remarks.
Simon wrote:
“Since you ask questions I will respond to your post, despite
your consistent use of the pejorative label ‘alarmist’ (I do not
understand why this term is considered acceptable here. I do
not label you or others whose views I do not share with
perjorative terms).”
In his response to my post it’s true Simon personally didn’t use any pejorative terms, and I thank him for that–no sarcasm intended–I mean what I say. However, in my opinion the term “alarmist” is a mild description of someone who says:
“The climate is nearing tipping points. Changes are beginning to
appear and there is a potential for explosive changes, effects that
would be irreversible, if we do not rapidly slow fossil-fuel
emissions over the next few decades. ”
“As species are exterminated by shifting climate zones,
ecosystems can collapse, destroying more species.”
“The amount of carbon dioxide in the air has already risen to
a dangerous level. ”
“Arctic sea ice will melt away in the summer season within the
next few decades. Mountain glaciers, providing fresh water for
rivers that supply hundreds of millions of people, will disappear –
practically all of the glaciers could be gone within 50 years – if
carbon dioxide continues to increase at current rates.”
“The greatest danger hanging over our children and grandchildren
is initiation of changes that will be irreversible on any time scale
hat humans can imagine.”
All of the above are quotes from one person, Dr. James Hansen, in a single article he wrote–see URL
http://www.guardian.co.uk/commentisfree/2009/feb/15/james-hansen-power-plants-coal
I’d call some if not all of those statements “alarmist”. If they aren’t “alarmist”–that is, if there’s no alarm–what’s the rush to limit fossil fuel use?
Finally, as far as pejorative terms go, I’ll trade “alarmist” for “holocaust denier”, “flat earther”, “tobacco-lawyer”, “in the pocket of big oil”, “immoral”, and “criminal”, which are used by AGW supporters (but admittedly not Simon) to characterize AGW nonsupporters. The term “immoral” was used by Dr. Gro Harlem Brundtland when she told reporters “This discussion is behind us. It’s over. The diagnosis is clear, the science is unequivocal — it’s completely immoral, even to question now, on the basis of what we know, the reports that are out, to question the issue and to question whether we need to move forward at a much stronger pace as humankind to addres the issues. James Hansen labelled at least some AGW nonsupporters as “criminal” when he said:: “When you are in that kind of position, as the CEO of one the primary players who have been putting out misinformation even via organisations that affect what gets into school textbooks, then I think that’s a crime.”
Simon then criticized my knowledge of history. He wrote:
“You don’t know your history. Milankovitch theory was well-
supported by the 1970s ( the Vema 28-238 core and others
having provided good verification), and the search then was
for an explanation of how small solar variations could explain
the extent of the glacial cycles. Scientists were considering
positive feedbacks as an explanation in advance of
discovering the relationship between temperature and CO2.
When the Vostok cores were pulled up, by 1985, the correlation
was evident, but it was always seen in terms of feedback. Your
suggestion of an ‘adjustment’ in the argument is simply an
invention.
Simon is right, I don’t know the history of AGW theory very well; and undoubtedly, Simon’s description of some scientists was right on. But if Simon is trying to convince me that when Al Gore first presented his slide show, Al (and others like him) knew that the initial temperature rises were not CO2 induced and carefully mentioned as much every time he gave his presentation, I can’t, or more correctly, won’t believe it.
In response to my question: “So the AGW alarmists have increasing CO2 levels acting as both an originating and a feedback phenomenon. I’d like to know which if either is correct.” Sinom answered: “Both”. When I asked: “if both, then what stopped runaway global warming in the past?” Simon answered: “The temperature feedback is less than the input.”
That answer is confusing to me because I thought the AGW supporters agree that the temperature changes from a doubling of CO2 by itself is at most 1.2 degrees, but the “feedbacks” will amplify this change by at least three degrees and somewhere I’ve read by as much as seven degrees.” And since the temperature effect as a function of CO2 level is logarithmic, more of the temperature change will occur early in the doubling process rather than later. As such I don’t understand how Simon can answer: “The temperature feedback is less than the input.” If Simon wants to talk about absolute temperatures, not temperature change, then he has a point. But I thought AGW theory was best described in terms of changes to a nominal level. This is in part why I want to see a clear diagram of the system inputs, outputs, feed forward, and feedback mechanisms.
And for what it is worth, the stability of a linear, time-invariant system with feedbacks is defined by the location of the system’s “poles” not by the system’s input. [For discrete-time systems, any pole on or outside the unit circle means the system is unstable. For a continuous-time system, poles on the vertical axis or in the “right half-plane” make the system unstable.] In turn, the location of a system’s poles are a function of its feedback loops, not the system’s input. Even for a large input, the system output may initially be small compared to the input but over time the system output will grow without bound.
Then I asked “If the larger temperature deltas at the system output are “fed back” into the system input, why won’t they cause even larger temperature deltas at the system output.” Simon answered”
“They are not – at least, not fully. I don’t like the use of the
term ‘feedbacks’ in climate science since it gives rise to this
confusion, but I guess we’re stuck with it. Yes, CO2 induced
warming may give rise to further increase in CO2 concentration,
but is not likely to equal the ‘input’ (although some do consider
the danger of methane release from the permafrost/sea bed,
which I expect is what Hansen has in mind when he considers
the possibility of a runaway effect if we were to burn all fossil
fuels).
I agree with you that the use of the term “feedback” is confusing; but as far as I am aware (and I have already admitted that my history of AGW theory is weak) it was the AGW supporters that coined the term “feedback” for the phenomena that resulted in larger temperature increases that could be supported by the initial forcing. In my opinion, and I’m NOT an expert on AGW climatological models, the phenomena AGW supporters argue will lead to catastrophic global warming are more respresentative of “feed forward” effects than feedback effects. But even with that caveat, Simon’s answer confuses me. First he says “They are not – at least, not fully”–meaning the larger delta temperature changes at the system output are “not fully” fed back to the system input. By “not fully” does he mean the output temperatures are not fed back to the system input but to an intermediate point in the system; or does he mean only a portion of the output temperatures are fed back to the system input; or both? The lack of precision of his answer is one of the reasons I want to see of “circuit diagram” for climate model used by AGW supporters.
Then after answering “not fully”, Simon then mentions methane gas and opines that Dr. Hansen just might have had a “full feedback” in mind for methane gas. If someone can clarify what’s going on, I’d appreciated it.
Finally, when I mentioned that I’d like to see a circuit diagram of the AGW theory, Simon responded: “I’d recommend Chapter 8 of the AR4. It may not be quite in the form you want, but you can figure out answers to your questions” Simon was kind enough to give the URL
http://www.ipcc.ch/pdf/assessment-report/ar4/wg1/ar4-wg1-chapter8.pdf
for that document.
When I did a quick look (and I mean quick), although I saw a lot of “discussion”, I didn’t see anything that even remotely resembled a circuit diagram. I’ll take a closer look when I get the chance; but if someone could refer me to a page number or figure number where such a diagram can be found, I’d appreciate that, too.

E.M.Smith
Editor
February 26, 2009 10:36 pm

John Galt (09:27:27) :
“Steven Goddard (08:52:13) : I have no affiliation with the University of Nebraska, and in fact have never been to Nebraska.”
Nebraska is just like Kansas, but without all the glitz.

Nebraska is much more like South Dakota, but without the trees.
Nebraska corn is shorter than Iowa corn, but only by a few inches. Lincoln Nebraska was where the folks from Iowa would go ‘for a good time’ back when you could not buy beer in Iowa. Nebraska was much more “loose” than Iowa then. They even had dancing.
(Per my Dad the drinking age was lower in 1930s and kids would head for the border… For the humor challenged: There is a T shirt saying “South Dakota State Tree” that I saw for sale in South Dakota. Had a picture of a telephone pole on it. Oh, and Dads’ side of the family has the Amish thread, thus the ‘shock’ over dancing in public… And Iowa hubris always held that Nebraska corn was just not as tall…)
To keep this remotely on topic: Anyone ever calculate how much CO2 is sucked out of the air each year by Corn Country? It’s got to be some tons / acre. And how much CO2 is over each acre? It would be an interesting thing to know… Can Nebraska claim carbon credits for all that CO2 sequestration?

Mark T
February 26, 2009 10:37 pm

^Chico: I believe O2 and N2 radiate at UV wavelengths, not IR, i.e., they are called IR-inactive.
Mark

Mark T
February 26, 2009 10:44 pm

Reed Coray (22:32:19) :
And for what it is worth, the stability of a linear, time-invariant system with feedbacks is defined by the location of the system’s “poles” not by the system’s input. [For discrete-time systems, any pole on or outside the unit circle means the system is unstable. For a continuous-time system, poles on the vertical axis or in the “right half-plane” make the system unstable.] In turn, the location of a system’s poles are a function of its feedback loops, not the system’s input. Even for a large input, the system output may initially be small compared to the input but over time the system output will grow without bound.
Finally… someone else with sufficient background to understand this.
Mark

Mark T
February 26, 2009 10:56 pm

Btw, Reed, there is at least one legitimate feedback that I can reasonably identify, though the tortured language used to define it, as well as the properties it is assigned, are a bit hard to swallow for anyone that has ever stepped foot in a system or control theory class.
Namely, raising the temperature of the oceans causes them to release more CO2 (or it decreases the net flux), which in turn adds to the greenhouse effect which raises the temperature of the oceans, etc. It seems somewhat like a single pole system with some delay (I don’t know what it is). By “partial feedback,” I would surmise that perhaps they assume some decay in the effect, which would imply a less than unity feedback path (after all the conversions), and hence a stable, lowpass system (there’s no apparent sign inversion, either, so the feedback term is positive). Since most of potential input frequencies likely have much greater periods than the delay element (just a guess), there’s some net gain associated with the process.
Some of the descriptions of how this behaves (the feedback “takes over” after some point, for example) have me about as confused as you seem to be.
Mark

pft
February 26, 2009 10:56 pm

On doubling CO2 .
http://entropy.brneurosci.org/co2.html
“Ted Ladewski suggested deriving an exponential curve from Beer’s Law. Although there are obvious problems involved in applying Beer’s Law quantitatively to a transparent medium as complex as the atmosphere (as he discusses in greater detail on his website, http://mysite.du.edu/~etuttle/weather/atmrad.htm#Spec),
(This is also discussed in http://www.sjsu.edu/faculty/watkins/GWnonlinear.htm )
Fitting the data to this equation, gives the much lower value of 287.62±0.07 K (±1 SD), or 0.46±0.08 °C increase above the 1980-2000 mean for a doubling of CO2 from current values. ”
and
“At the current rate of increase, CO2 will not double its current level until 2255.”
Joel Shore (17:20:42) :
“Now, that may not sound like much to you, but your intuition on this is bad for a couple of reasons: One is that 99% of the atmosphere consists of diatomic molecules (N2 and O2) that are transparent to infrared radiation. So, the remaining 1% play a disproportionate role in the climate”
I don’t disagree except you are treating H20 and CO2 as equals in the 1% and understimating N2 and O2’s role in climate by over emphasizing IR.
Looking at it another way. About 20% of the solar energy that enters the atmosphere, warms the atmosphere directly via absorption. Gamma rays, X-rays, and ultraviolet radiation and near-Infrared are primarily absorbed by oxygen, nitrogen and H20. Less than 1% of incoming sunlight is infrared that is absorbed by CO2. Also, clouds made of H20 also reflect 20% of the incoming radiation. The diagram on this article conveys the false impression that clouds are responsible for all the absorption of incoming solar radiation in the atmosphere.
Of the 50% of sunlight that actually reaches the surface and is absorbed, only 40% is released as LWR, or about what the same amount of radiation as the 99% of O2/N2/H20 absorbed from the incoming radiation.
H20 is about 1-4% of the atmosphere and is the most important GHG, depending on location and weather, and absorbs most of the LWR. CO2 is responsible for about 4-9% of the GHG effect, so it seems to me that the 99% O2/N2/H20 combo is a much bigger player than the 0.036% CO2, although CO2 packs a wallop on a molecule per molecule basis.
H20 clearly plays the major role in climate, and CO2 is a minor player. CO2 may be responsible for 1-2 degrees of our 57 degrees average temperature, more so in areas without much H20, but it does not drive climate. H20 and the sun do that, and N2 and O2 play a major role in providing the medium for convection which is so important to cool the surface, otherwise even with CO2 and GHG being at todays levels, we would be at 140 deg F (weather cools)

Paul
February 26, 2009 11:12 pm

In newspeak I write far from airstrip one on the outer borders of Oceania. With a bellyfeel for the blackwhite I must use crimestop and agree with BB that Co2 is an important GHG. There is no use in crimethink against a doublespeak concensus of science. We must be goodthinkers use duckspeek and appear fullwise and unbad to please the inner party.
Oceania ‘Tis for Thee.

J. Peden
February 26, 2009 11:15 pm

Stephen Wilde (11:06:27) :
I tackled this general subject a while ago and it can be found here:
http://co2sceptics.com/news.php?id=1562
Thanks, Stephen. I recommend it, no doubt partly because it asks/asserts what I’ve thought about forcings and runaways/tipping points. Here are some excerpts, which really don’t do Stephen’s analysis much justice. The in line brackets enclose my additions:

The total heat store available in the oceans [compared to that stored in land and the atmosphere, mainly resulting from SW solar input to oceans and their greater ability to capture and store it] is so large that it is capable of rendering changes in any Greenhouse Effect an irrelevance for all practical purposes.
[…]
Oceanic oscillations are sufficient to cancel out or enhance the effects of natural variations in solar irradiance or other forms of solar input to the heat budget of the Earth for variable periods of time. A range of a mere 4 Watts per square metre or less in Total Solar Irradiance is sufficient to explain changes in Earth’s atmospheric temperature for the past 400 years. Outside that narrow band of apparent solar normality we would have more to worry about than any Greenhouse Effect.
[…]
Much of the [atmospheric, vs oceanic] warming feared by the alarmists relies upon a positive feedback involving increased water vapour exaggerating any CO2 warming effect. However that process is unconvincing to my mind because it ignores the water vapour reducing processes of evaporation, convection, condensation and rainfall which are all substantial, but so far unquantified, moderating effects as far as atmospheric warming from water vapour is concerned. In the past no tipping point has ever been known to have occurred as a result of runaway warming from extra water vapour so how have we been persuaded to fear it so much?
[…]
Even the water vapour effect [varying water vapor density with varying atmospheric temperatures] has never provoked any tipping point in the face of the primary solar/oceanic driver so CO2 could never do so.

E.M.Smith
Editor
February 26, 2009 11:35 pm

Tom (11:58:29) :
Can somebody tell me what’s wrong with this critical examination of the traffic light analogy:
In the traffic light analogy, the energy is driving from point A (the outside extreme of the atmosphere) to point B (the surface). The author’s point is that the commute takes longer, and since the driver spends more time driving, this equates to an increase in heat.

I can’t stop thinking about thermalizing. You have this CO2 channel all plugged up, and long distances between CO2 molecules. Seems to me that the air must thermalize and then convection takes over. Kind of like you have more stoplights, but too many of them and they make the road start moving faster toward your destination.
Now at some point the hot air will get high enough in the sky for radiation to space to matter and let some energy go, but the whole greenhouse (and traffic light) analogy just seems broken to me due to the facts of thermalized energy at high densities in the lower atmosphere.
Basically, I think there are at least two regimes needed to properly model this:
1) Low altitude high pressure with thermalization and convection. (and a substantially IR opaque behaviour due to GHG density)
2) High altitude with radiation to space and poor thermalization due to thin density (low collision rate) and IR transmissive / radiative due to low collisions.
And I don’t even want to think about the transition zone right now…
The model of the AGW folks (and described in this post) looks at the world from the perspective of low thermalization. The skeptics complaint is about high thermalization. The reality is that “it all depends” on where you are in the atmosphere.
Then you get to add in water… 😉

Rik Gheysens
February 27, 2009 12:46 am

Steven Goddard,
Only to make things more complicated…: The viewpoint of IPCC concerning greenhouse gases can be found on
http://www.ipcc.ch/pdf/assessment-report/ar4/wg1/ar4-wg1-chapter1.pdf
I am not sure this will help.

Claude Harvey
February 27, 2009 12:55 am

After reading comments to the effect that “hot air can’t heat cold water”, garbled definitions of “relative humidity”, etc. I’m of the opinion that Al Gore’s Nobel Prize has festered some otherwise perfectly good brains.

David Porter
February 27, 2009 1:27 am

foinavon (17:01:51) :
Thank you for your reply.
One of the best ways of understanding a technical explanation is via a graph, (assuming its graphical of course), and since you find the three curves on Stevens graph to be unrepresentative of increasing CO2 levels versus temperature, I would appreciate seeing one from yourself. I make the assumption that one must exist. I have read from either yourself or Joel Shore that you accept that the curve is logarithmic.

February 27, 2009 1:42 am

Alex` Heyworth and others including Steve Goddard:
Mars has something like 30 times the mass of CO2 over each square meter than has Earth.
Mars ought to be a pretty good planetary model to figure out just how much “greenhouse” effect there is from CO2 alone. Mars has almost the same rotation period as Earth, about the same axial tilt , a year almost twice as long, the amount of solar radiation arriving at the top of the martian atmosphere is known and the amount of CO2 is known. There is no great amount of water vapor and no oceans to complicate matters. Yet the numbers I’ve seen for the Martian surface temperature ascribe a “greenhouse” effect temperature increment of between 5 and 10 deg C.
When someone comes up with a number for Mars with a much lower range I’ll listen to their estimate for Earth.

cal
February 27, 2009 2:34 am

I have finally realised why I have found this particular discussion so confusing. Steven began by trying to explain a simplified model of the badly named greenhouse effect. I generally agree with his description but he was not entirely explicit about the simplifying assumptions that he was making. This left him open to the “ah but what about..” comments which then tended to presage highly simplistic arguments about a far more complex model.
So at the risk of being boring I will be explicit.
We can consider the greenhouse effect in isolation if we assume that all other variables which affect climate are constant. These are:-
Cosmic variables – sun’s output, earth’s distance from and inclination to the sun, Solar wind, cosmic rays etc.
Geophysical variables – earth’s albedo (% of sun’s energy reflected by surface) , volcanic and geothermal activity etc.
Climate variables – energy stored in oceans land and atmosphere, vertical and horizontal non radiative heat transport (winds, sea currents, convection, evaporatation etc.) amount and location of clouds, aerosols humidity etc.
Any other variables you can think of other than the concentration of CO2.
This may seem ridiculous but only if all these variables are considered constant can you consider the affect of CO2 in isolation. I have effectively created a “Groundhog day” where every day is identical. Instead of using this to seduce my colleague I am going to change the CO2 concentration to see what would happen.
Steve gave a good description of the basic physics but I will give a slightly different slant on it which might be helpful.
In this scenario where everything is constant the temperature will also be constant. What would this temperature be?
For the temperature to be constant the energy absorbed by the earth has to be equal to the energy radiated into space.
The energy absorbed is mainly short wave radiation characteristic of the sun’s high temperature.
The energy radiated is long wave radiation characteristic of the earth’s low temperature.
The amount of energy radiated at any particular wavelength will depend on the emissivity and temperature of the surface which emits the photon into space.
For about half the spectrum these photons are emitted from the earth and therefore the temperature can be considered to be about 300K.
The other half of the spectrum is absorbed by the atmosphere and then reradiated into space from high altitude.
Water is the main factor here. Because its concentrations are high near the surface and considerably less at high altitude the effective height at which radiation into space takes place is only a few kilometres above the surface. Temperature reduces with altitude so the effective temperature is about 270K.
CO2 is more evenly distributed and is a powerful absorber albeit in a small frequency band between 14 and 18 micron compared with the total spectrum which is mainly between 6 and 50 micron. The effective height of this radiation layer is about 8km according to Hadley but the satellite data I have places it much higher. The effective temperature might be as low as 225K.
Because radiation is proportional to the 4th power of the absolute temperature the 10% reduction in the temperature of the water molecules radiating into space (compared with the surface) results in a reduction in energy loss reduction of about 1/3 for those wavelengths.
For CO2 the reduction in temperature is 25% so the energy loss reduction is more like 60%.
These reductions in energy losses at the characteristic absorption frequencies of water and CO2 (and to a lesser extent ozone and methane etc.) means that the energy losses at the other wavelengths have to increase to compensate. Again if all other things are constant the only way this can happen is if the temperature of the surface rises. This is the so called greenhouse effect. However silly the name, the effect is real.
So, back to our “Groundhog” experiment. What would happen if we increased the amount of CO2? The number of absorptions and re-radiations would increase as Steve describes but ultimately the key issue is that, at a given height, the probability that a photon would be directly radiated into space reduces. This means that the effective radiating surface must move to a greater (less dense) altitude. This implies lower temperatures and therefore further reduced losses. Therefore the surface has to increase in temperature to compensate. Thus the greenhouse effect still impacts even when the CO2 absorption bands are saturated.
This is the “incontrovertible” evidence that the AGWs propound. However it does not take a genius to see that all the other variables are not constant. It is also obvious that 60% of the total effect that CO2 can have has already happened. More importantly the level at which CO2 radiates is approaching the tropopause where further increases in height would not lead to reductions in temperature. Some research would suggest that the height of the tropopause is increasing to accommodate a temperature drop but I am yet to be convinced by the data.
So I agree with Steve. Don’t try and argue that the greenhouse effect is not real because you are on a loser. The real argument is that history demonstrates that the effect is swamped by all the other variables. The AGWs are then left with one argument which has some validity. Whereas all the other variables are cyclical and would average to zero over time the effect of CO2 is permanent.
My feeling about this is that we are living in a golden age. Temperatures are high compared with the average over the past million years although it has been warmer. If we are doing something that increases the average temperature by about 1 or 2 degreesC (which is what the science really suggests might happen over the next centuary) then all to the good. It might just keep us in the manner to which we have become accustomed.

Allan M
February 27, 2009 3:12 am

Steven Goddard (09:20:12) :
Gary Gulrud,
You wrote: Back-radiation has no practical consequence of heating the surface whatever.
I bicycle year round day and night, and am quite certain that my hands stay warmer on cloudy nights in the winter – due to back radiation from the clouds.
jae (09:34:15) :
Maybe it’s because water vapor has four times the thermal capacity of the rest of the air?

Also, as the air is more humid on a cloudy night, the perspiration generated by the effort of cycling evapoates slower, and hence reduces skin temperature rather less, and many many more factors –
NOT JUST THE OBSESSION WITH A RADIATIVE EXPLANATION OF LIFE, THE UNIVERSE AND EVERYTHING! SURELY THERE IS MORE TO LIFE THAN THIS!
As has been pointed out before on this site, the sun’s emissions in the IR band are as large as in the visible band, so it seems the sun is not too hot (proud?) to emit low frequencies.
AM

JamesG
February 27, 2009 3:51 am

One annoying thing in debates like this is the continual appearance of few wild guesses that keep being presented as facts or “evidence” in debates like this:
1. The continual assertion about “observational evidence” for the effect of increased CO2. All such observations in fact are ambiguous, with cause and effect difficult to determine, yet a lot of scientists have nevertheless given an opinion that certain effects must have been due to CO2. They managed this feat by excluding or ignoring all other possibilities. Yet their knowledge about these other possibilities is ludicrously minimal (which is even admitted by the IPCC). The arguments rest entirely on “we can’t think of anything else”. This flawed opinion then somehow becomes stated as “evidence”. Well it just isn’t! Should we accept it as expert opinion anyway? Only if some real evidence appears!
2. The idea that ice age cycling can be explained by CO2 amplification but by no other means. Obviously there must be some kind of amplification involved but whether this is CO2 is merely guesswork. In fact CO2 can explain the heating quite well but it can’t explain the cooling at all. And that is the illogical part of the feedback theory that AGWers just ritually ignore. In fact datawise that idea that CO2 acts alone has been debunked by Lowell Stott’s real evidence. Severinghaus on RC guesstimated a mere 30% effect from CO2, without of course telling us too much where the rest came from. Of course it could have conceivable been a negligible effect. When you don’t bother to look for any other factors then you surely don’t find them.
3. The idea that any demonstration of increased water vapour in the atmosphere – either the postulated 2% by Trenberth or that ridiculously short trend analysis by Dessler – means that the high CO2 amplification factor has been proven. Yet as Lindzen argues, higher water vapour can mean cooling as well as heating so you need to look at the actual temperatures for the evidence. Is it there in the temperatures? Well no it isn’t. And of course then the various hand-waves come into play – ozone depletion (lately Steig), brown clouds (Ramanathan), weather noise (everyone), the sun (Hansen), dubious statistical error overlaps and observational errors (Santer et al.). It’s all a big guessing game where the conclusion is pre-determined by bias. These expert scientists know the excess warming should be there so it must surely be masked by something else. Whereas the more logical explanation is that the basic theory of CO2 amplification is likely wrong.
I’d agree with Steven that the 1.2C degrees for no-feedback doubling, has to be conceded even if that is dubious too (due to the gross simplifications involved). But without focusing specifically on the much weaker feedback issue you can be picked off too easily as a knee-jerk contrarian. Whereas all we want is the real truth – not biased opinions presented as truth.

E.M.Smith
Editor
February 27, 2009 3:52 am

Potential double accounting?
This: http://www.globalwarmingskeptics.info/phpbb3/viewtopic.php?f=5&t=585&st=0&sk=t&sd=a
Claims that the CO2 measurements are done with IR that can not distinguish th CO2 absorption from other GHGs absorption signatures. IF true, this would assert that we actually measure “IR absorbent gas” rather than CO2, then we separately measure methane et. al. and add those GHGs to the CO2 measure to get total GHGs.
This looks to me like a double accounting. Does anyone know how they get the CO2 measure to only measure CO2, given that it’s really an IR measure?

DR
February 27, 2009 4:14 am

AD NAUSEUM to Joel Shore,
Joel Shore,
You did not reply to my post on Gavin Schmidt et al 2005 quoted thusly:
“Tropospheric warming is a robust feature of climate model simulations driven
by historical increases in greenhouse gases (1–3). Maximum warming is predicted
to occur in the middle and upper tropical troposphere.”
http://www.osti.gov/energycitations/servlets/purl/881407-xk2Sdg/881407.PDF
Your statement, and those being bandied about the various pro-AGW blogs are now back pedaling. With that, you are saying GCM predictions are mistaken?
If your version is correct, why then have Schmidt, Santer and the usual suspects been attempting to a) discredit satellite data and b) show models in agreement with observations? Surely you are aware of the serious “problems” with Santer et al 2008?
You can’t have it both ways. Are GCM predictions, which are driven by increases in GHG emissions, correct or not?

tmtisfree
February 27, 2009 4:14 am

Chico (22:08:27) :
Finally a discussion on the physics of the Greenhouse Effect. I have a few observations and questions.
I agree. It is interesting to see all the different approaches by people here. I am personally more interested by the instantaneous physical effect.
2) Phil’s Makes a very powerful point that an excited CO2 molecule will almost certainly transfer its ‘excess’ energy to surrounding O2 and N2 molecules. This is *very* important because O2 and N2 don’t principally radiate at a frequency that is absorbed by traditional Greenhouse gases. Correct me if I am wrong but it is O2 microwave radiation that satellites observe to infer the temperature of the Earth’s atmosphere (i.e. RSS and UAH). Although Phil also suggests that O2 and N2 don’t radiate? I thought all mater above absolute zero temperature radiates?
Is it possible that H²O and N² molecules (given their number) could also re-transfert some energy to CO² molecules by collision?
To Phil,
you apparently has some knowledge in physic (at the molecular level). Could you provide a (or point to a detailed) heat/energy analysis of the influence of the re-radiated LW photon by CO² molecules on both the surface and the atmosphere right above the surface? My own analysis above is meaningless and I think it is the central problem of the GH effect (if any). Thanks in advance.
Bye,
TMTisFree

foinavon
February 27, 2009 4:34 am

David Porter (01:27:35)
David you can construct a graph for yourself that is similar in form to the graphs in the top post, but is more representative of the climate sensitivity in relation to a very large amount of scientific evidence.
This puts the likely middle of the range value of the climate sensitivity (the temperature rise at equilibrium upon doubling atmospheric CO2) near 3 oC.
A suitable equation is:
T = (3.0/log(2))*(log(C))-9.39
This gives the temperature at equilibrium within a climate sensitivity of 3.0, normalised to a temperature of 15 oC at an atmospheric CO2 concentration (pre-industrial) of 280 ppm. The “log(2)” relates to the fact that the 3.0 is the temperature rise upon doubling, “C” is the atmospheric CO2 concentration and “log(C)” defines the logarithmic dependence of the temp rise on increased atmospheric CO2.
You can use a different value for the climate sensitivity but you’ll then need to re-normalize the 280 ppm value to 15 oC (by adding/subtracting the appropriate offset – we could do a better equation to normalize this automatically!). Use the appropriate tool in your graphing package to construct a set of data for “C”.
Note that the graph in the top post is additionally misleading in two further ways:
ONE: The Y-axis label (“Estimated temperature”) isn’t estimated temperature at all, but is the estimated temperature rise at equilibrium. For example, when the Earth’s temperature was “nearish equilibrium” at the preindustrial level of around 280 ppm, the temperature couldn’t have been 5 oC, 7 oC and 12 oC all at the same time! I’ve normalized my data to a pre-industrial (280 ppm) “temperature” of 15 oC, but you could normalize to a different pre-industrial temperature if you felt like it.
TWO: by plotting the data starting at a completely unrealistic value for [CO2] near zero the graphs appear to have virtually flattened from around 250 ppm [CO2] upwards, especially for the vastly unrealistically low values for the climate sensitivity. To determine what the equilibrium temperature response looks like within a 3 oC climate sensitivity under conditions that are meaningful for consideration of current and future emissions, plot the data for a [CO2] range from 250 ppm (say) to 1200 ppm (say) to encompass the [CO2] range of real world relevance.

AJ Abrams
February 27, 2009 5:06 am

I asked a question above to those that are here often and was ignored. Please someone take my question up. Educate me.
I said:
Something was mentioned twice here that was never addressed by the normal folk that inhabit this site –
Pressure.
Does the temperature of Venus, Mars Jupiter atmosphere at 1ATM correlate to earth’s average temperature after adjusting for distance from the sun?
If it does, and I’ve now read two websites not related to climate science that say that it does, then the big…..OH…moment for me is that pressure regulates temperature more than any thing and that the ocean accounts for the rest of earth’s temperature variations over time – noting that the earths overall temperature doesn ‘t actually fluctuate by that much).
Also it says that SG’s comments about Venus and Mars are really misleading to say the least, which as a long time reader here, is disappointing.
Can someone clarify? Leif would be a prime candidate to answer because what I said would go along perfectly with his tireless shouts that IT’S NOT THE SUN! Which I believe

Cassandra King
February 27, 2009 5:12 am

With regard to the supposed greenhouse effect, does rainfall affect the balance?
How much atmospheric carbon dioxide and atmospheric heat is removed by precipitation?
The old desert cooler(before aircon) was quite good at taking away atmospheric heat.
Does this constant removal affect the calculations in any meaningful way and has it been inculuded in the calculations?
Perhaps a quarter of the earths surface is being rained on any given day so this must take a considerable amount of CO2 and heat from the atmosphere and dump it either into the ocean cycle or the biomass cycle.

Roger Clague
February 27, 2009 5:25 am

Steve Goddard writes ‘higher GHG densities force the temperature up’. He goes on to say that this is not disputed.
This is wrong. He has played a linguistic trick.
The statement, that for planetary atmospheres ‘higher densities force temperatures up’ is based on well established physical theories.
By adding GHG, standing for ‘greenhouse gases’, he introduces a new, different and greatly disputed theory.
There is no greenhouse effect. It is no use saying, ‘OK, CO2 does cause warming, lets get paid for discussing how much’.
It is good to be sceptical especially of a warmist guest posting

Steven Goddard
February 27, 2009 5:37 am

EM Smith,
Doesn’t most of the corn grown in Nebraska get fed to cows and belched out as GHG?

Simon Evans
February 27, 2009 6:05 am

Reed Coray (22:32:19) :
Reed,
Thank you for your response. I’ll try to address your points as best I may.
Firstly, on the matter of labels (‘alarmist’, ‘denier’, etc.), I agree with you that this is an issue on both sides. I think there are some alarmists about (people deliberately misrepresenting or exaggerating the likelihood of calamitous outcomes) and also think there are some denialists around (people deliberately misrepresenting or exaggerating to the opposite effect).
Let’s just presume that we fall into neither camp :-). Though I would certainly accept that I am alarmed by the risks, my intention is not to exaggerate.
Simon is right, I don’t know the history of AGW theory very well; and undoubtedly, Simon’s description of some scientists was right on. But if Simon is trying to convince me that when Al Gore first presented his slide show, Al (and others like him) knew that the initial temperature rises were not CO2 induced and carefully mentioned as much every time he gave his presentation, I can’t, or more correctly, won’t believe it.
I’d be very surprised if he didn’t know, but I guess you’re right that he didn’t ‘carefully mention’ it (I’ve only watched it once, a while ago. What I remember as being ‘alarmist’, IMV, was the implication that multi-metre sea level rises might be imminent which, at the time at least, was not a fair view). But, as is oft repeated here, Gore is not a scientist. I’m not an apologist for him anyway, but I could pick out just as many ‘glosses’ from the public presentations of people like Bob Carter, for example.
In response to my question: “So the AGW alarmists have increasing CO2 levels acting as both an originating and a feedback phenomenon. I’d like to know which if either is correct.” Sinom answered: “Both”. When I asked: “if both, then what stopped runaway global warming in the past?” Simon answered: “The temperature feedback is less than the input.”
That answer is confusing to me because I thought the AGW supporters agree that the temperature changes from a doubling of CO2 by itself is at most 1.2 degrees, but the “feedbacks” will amplify this change by at least three degrees and somewhere I’ve read by as much as seven degrees.” And since the temperature effect as a function of CO2 level is logarithmic, more of the temperature change will occur early in the doubling process rather than later. As such I don’t understand how Simon can answer: “The temperature feedback is less than the input.” If Simon wants to talk about absolute temperatures, not temperature change, then he has a point. But I thought AGW theory was best described in terms of changes to a nominal level. This is in part why I want to see a clear diagram of the system inputs, outputs, feed forward, and feedback mechanisms.

It wasn’t my clearest ever answer, so I’ll try again (I should have maybe written “the temperature change from one circuit of the feedbacks is less than the temperature change from the input alone”). In the case of increased CO2 concentration giving rise to increased IR absorption giving rise to temperature gain (allow the premise for now!) giving rise to increased CO2 concentration as the ocean buffer changes, then that is a simple feedback loop. In this case, I understand that the GHG feedback will be markedly less than the GHG input, except perhaps in the case of methane clathrate release. In the case of CO2 ~ temperature increase ~ water vapour increase ~ temperature increase then there is, so far, a feedback to the system, though not to the original input. There will also be some feedback to the original input from further warming of the ocean buffer, but again it will be smaller than the original. For a given input, the system adjusts to a new state of unstable equilibrium. To simplify, an input of 1 which has a total feedback rate of 75% would give rise to an equilibrium output of 0.75/(1-0.75), = 3.
And for what it is worth, the stability of a linear, time-invariant system with feedbacks is defined by the location of the system’s “poles” not by the system’s input. [For discrete-time systems, any pole on or outside the unit circle means the system is unstable. For a continuous-time system, poles on the vertical axis or in the “right half-plane” make the system unstable.] In turn, the location of a system’s poles are a function of its feedback loops, not the system’s input. Even for a large input, the system output may initially be small compared to the input but over time the system output will grow without bound.
Well, I don’ think the climate is a linear, time-invariant system, and nor is it simply deterministic! But anyway, I think you are considering a system above where the input is maintained. In the climate case, where the input of increased IR absorption is considered, we may also conclude that if that input were to be maintained (and eventually, of course, the planet would run out of capacity to ‘feed’ it) then the output would continue to accumulate. The calculation of the outcome from doubling CO2 is simply that, though – an assessment of equilibrium temperature response to that doubling alone.
Simon’s answer confuses me. First he says “They are not – at least, not fully”–meaning the larger delta temperature changes at the system output are “not fully” fed back to the system input. By “not fully” does he mean the output temperatures are not fed back to the system input but to an intermediate point in the system; or does he mean only a portion of the output temperatures are fed back to the system input; or both? The lack of precision of his answer is one of the reasons I want to see of “circuit diagram” for climate model used by AGW supporters.
Ok, try again. There is no ‘larger temperature delta’ consequent upon a temperature input for one ‘circuit’ of any feedback loop. If there were then, of course, the system would be subject to explosive feedback. The larger temperature delta is the addition of all feedback over time and is not fed back in total, since it is only the change in temperature that feeds back and not the absolute temperature. By ‘not fully’ I meant simply that the feedback rate, in response to any temperature change, is less than 1.
Then after answering “not fully”, Simon then mentions methane gas and opines that Dr. Hansen just might have had a “full feedback” in mind for methane gas. If someone can clarify what’s going on, I’d appreciated it.
I had in mind the ‘clathrate gun hypothesis’ –
http://en.wikipedia.org/wiki/Clathrate_gun_hypothesis
Finally, when I mentioned that I’d like to see a circuit diagram of the AGW theory, Simon responded: “I’d recommend Chapter 8 of the AR4. It may not be quite in the form you want, but you can figure out answers to your questions” ….I didn’t see anything that even remotely resembled a circuit diagram.
No, as I wanted to indicate, it’s not in the form you want, but it does include the figures for various feedback assessments (p. 629 on) along with discussion. I’m not aware of a ‘circuit diagram’ presentation as such, so I was suggesting that the best one might do is to figure it out from the discussion of the modelling.
I hope I’ve clarified my views further.

foinavon
February 27, 2009 6:12 am

Smokey (17:23:56) :

foinavon opines that “…the Lindzen data uses asumptions that we know are incorrect.”
Well now. Who are we to believe? foinavon? Or the head of M.I.T.’s Atmospheric Sciences department?

Smokey, a useful aspect of science is that one can relieve oneself from tedious arguments (“my Pofessor is more important than yours!) and address the evidence.
It’s not clear where the data for Lindzen’s climate sensitivity in the graph in the top post comes from (someone apparently created the graph on a site called “junk science” apparently). However we know from Lindzen’s earlier papers that his interpretations of low climate sensitivity values arose partly from his model in which moist (cumulus) convection that should increase with atmospheric warming under enhanced CO2 concentrations, might cause a drying of the upper troposphere[***]. This proposed drying would be a negative feedback that would partly offset the positive feedback from enhanced water vapour at lower altitudes.
Now we know that this interpretation was incorrect. We can measure the water content of the troposphere (upper and all) and determine that in the real world the upper troposphere is moistening (as models predict in fact)[*****]. So we reject Lindzen’s failed hypothesis in this particular respect, since it is shown not to be correct. There’s no problem with that. It’s part of science. However what we shouldn’t do is to retain analyses that we know to be incorrect because we happen to like their implications.
[***]e.g. Lindzen RS (1990) Some Coolness Concerning Global Warming Bull. Am. Meteorol Soc. 71, 288-299
[*****]e.g. Soden BJ, et al (2005) The radiative signature of upper tropospheric moistening Science 310, 841-844
Santer BD et al. (2007) Identification of human-induced changes in atmospheric moisture content. Proc. Natl. Acad. Sci. USA 104, 15248-15253
Brogniez H and Pierrehumbert RT (2007) Intercomparison of tropical tropospheric humidity in GCMs with AMSU-B water vapor data. Geophys. Res. Lett. 34, art #L17912
Buehler SA (2008) An upper tropospheric humidity data set from operational satellite microwave data. J. Geophys. Res. 113, art #D14110
Gettelman A and Fu, Q. (2008) Observed and simulated upper-tropospheric water vapor feedback . J. Climate 21, 3282-3289

Roger Knights
February 27, 2009 7:11 am

pft (16:12:01) :
“and the more CO2 there is, the less effect it has. ” should have said “and the more CO2 there is, the less affect more CO2 has”

Not exactly (saith the grammar policeman).

February 27, 2009 7:14 am

Excellent discussion. I am wondering if you have a way to explain this to people in a general way that would attract a more serious public discussion than whether or not global warming is a reality.

Steven Goddard
February 27, 2009 7:16 am

Phil,
Your explanations are appreciated, but appear to be missing something important. If the CO2 is losing all of it’s excitation energy in nanoseconds near the surface, how can there be excited CO2 at 1mb altitudes?
According to this chart, the 15um absorption spectrum is saturated.
http://www.globalwarmingart.com/wiki/Image:Atmospheric_Transmission_png

HasItBeen4YearsYet?
February 27, 2009 7:21 am

@ Leif Svalgaard (08:15:03) :
“…any energy CO2 absorbs [most of it coming from below] is immediately re-emitted, half up into space and lost and half downwards back to the surface. The surface thus heats up and warms the air by conduction and convection.”
Wouldn’t it be more correct to say that the ground heats up faster, and/or cools more slowly?
(I know you know all this stuff, but I find it easier to understand if I look at it this way)
If we were at steady state, with energy emitted=energy absorbed, the ground and air would be maxed out and no change would occur. So, every day from sunrise to sometime in the afternoon the earth warms up, and from then till the next sunrise the earth cools. We never, fortunately, achieve ‘steady state’.
So, the ground warms faster because of GHG’s, and it does that because when it re-radiates the sun’s energy back into space, some of that energy is absorbed by the atmosphere and half is re-radiated back to earth. BUT that is only half of what the earth radiated out, so it can’t “warm” the earth.
If you give me two dollars, and I give you back one, you aren’t richer than you were before you gave me the 2, though you are less poor. The same with the energy arriving from the sun. In a fixed time, the earth receives, say 10, of which it gives 2 to the air, and the air gives one back 1, so the earth now has 9, as opposed to only 8 if the air didn’t give any back.
The involvement of the air, then, isn’t to warm the ground, but to slow it’s cooling, which will in turn keep the air warmer for a longer time. The question then is, how much longer and what difference does it make? I.e., how much does increased [CO2] slow the heat loss – and if water vapor does the same thing, and does it so much more effectively than CO2, why are we worried about CO2?
ASIDE – the plot of earth’s climate history is often used as a demonstration of how CO2 has been higher in the past, but what strikes me about that figure is how there seems to be a maximum temperature of 22DegC which has NEVER (except for a single blip) been exceeded, and how we are currently no where near that max. If that means the earth CAN’T get any hotter than 22DegC, no matter how much CO2

Joel Shore
February 27, 2009 7:22 am

JamesG says:

1. The continual assertion about “observational evidence” for the effect of increased CO2. All such observations in fact are ambiguous, with cause and effect difficult to determine, yet a lot of scientists have nevertheless given an opinion that certain effects must have been due to CO2.

Welcome to the world of science. Of course, all observational data has problems and is subject to different interpretations, which is why it is important to have a broad array of different data and a lot of different analyses, which is what we have.

2. The idea that ice age cycling can be explained by CO2 amplification but by no other means. … Severinghaus on RC guesstimated a mere 30% effect from CO2, without of course telling us too much where the rest came from. Of course it could have conceivable been a negligible effect. When you don’t bother to look for any other factors then you surely don’t find them.

First of all, people have been studying the ice ages for a long time and have looked at a wide variety of explanations. In fact, your own statement about Severinghaus shows that there are other factors that have not only be considered but in fact are understood to contribute significantly.
Second of all, in answer to your implied question, most of the rest of the forcing comes from the change in albedo due to the ice sheet advances and recessions. There is also a little bit of contribution from changes in aerosol levels in the atmosphere although I can’t remember which way that goes. The orbital Milankovitch oscillations that trigger these cycles contribute only a very small amount to global mean forcing although they play a vital role in causing the advance and recession in the ice sheets that lead to the considerable forcing due to the albedo change.
Third of all, your question about how we know that CO2 contributes about 1/3 of the change rather than being negligible is a good one (and something I once wondered to). The answer is that you are missing at least one important piece of the puzzle: As Steve Goddard’s discussion explains, we know very well the radiative forcing due to the change in CO2. We also have reasonable estimates of the radiative forcing due to the albedo change, the small forcing due to aerosol changes, and the very small forcing due to the orbital oscillations. From this, we have a good estimate of the total forcing in W/m^2. We also have reasonably good estimates for the global mean temperature change. By dividing this global temperature change by the forcing, we get an estimate of the climate sensitivity: ~0.75 C / [W/m^2]. Since doubling CO2 gives a forcing of 4 W/m^2, this implies a climate sensitivity of about 3 C per doubling.
Note that if we simply did what you suggest and assumed that CO2 had a negligible effect and excluded its contribution to the forcings, we would get an even higher estimate for the climate sensitivity than ~0.75 C / [W/m^2] because it would have then taken less W/m^2 to produce the observed temperature change. Of course, our calculation would also not be self-consistent since this result for the sensitivity together with the known radiative forcing due to CO2 would imply that in fact CO2 does play a non-negligible role, contrary to our original assumption.
The only conceivable way to pull a much lower climate sensitivity out of the ice age – interglacial cycles is to come up with some huge forcing that we are missing (or claim that we are vastly underestimating the albedo forcing)…Or claim that the whole concept of radiative forcings breaks down in a very dramatic way. And, of course, this still leaves us with difficulties in explaining other paleoclimate events, the eruption of Mt Pinatubo, etc.
As a final note, I have presented one piece of the puzzle here. But, there are other pieces. For example, I believe that the role of greenhouse gases is necessary in order to explain the synchronicity of the ice age – interglacial cycles in the two hemispheres. I.e., if you have only the other forcings, climate models tend to have difficulty producing the approximately synchronous rise in temperature seen in both hemispheres. This is additional evidence for the role of the greenhouse gases in the ice age – interglacial cycles.

February 27, 2009 7:26 am

When I see some evidence that the atmosphere has actually warmed above the level where most of the latent heat of evaporation is released I will start to take some interest in the possibility of a ‘greenhouse effect’. Till then, I will attribute the warming of the near surface atmosphere (the only part that has actually warmed) to the increase in ocean temperature and evaporation.
The land does not store heat and as Bill Illis very sensibly remarked:
“I think everyone could benefit from understanding the greenhouse effect operates at a timescale of hours. It operates at the speed of light and at the speed of quantum physics.
It is just a delay of several hours in the time in takes for the Sun’s energy from the current day and the previous day to escape into space.”
Now, that is good common sense. But is it hours or just minutes? And if any net addition to the surface temperature is lost before sun-up the next day, who cares.
In any case I know in my bones that this summer is just not warm enough.

Steven Goddard
February 27, 2009 7:46 am

Allan M,
At 20F, humidity is very low, and perspiration is essentially non-existent. Also, high clouds are not necessarily associated with low altitude humidity but they do produce a lot of back radiation.

HasItBeen4YearsYet?
February 27, 2009 8:02 am

@ Leif Svalgaard (08:15:03) :
“…any energy CO2 absorbs [most of it coming from below] is immediately re-emitted, half up into space and lost and half downwards back to the surface. The surface thus heats up and warms the air by conduction and convection.”
Wouldn’t it be more correct to say that the ground heats up faster, and/or cools more slowly?
(I know you know all this stuff, but I find it easier to understand if I look at it this way)
If we were at steady state, with energy emitted=energy absorbed, the ground and air would be maxed out and no change would occur. So, every day from sunrise to sometime in the afternoon the earth warms up, and from then till the next sunrise the earth cools. It seems we never achieve ‘steady state’?
So, because we aren’t at steady state, the ground warms. And, it does so faster than it would in a vacuum, or with a radiation transparent gas, presumably because of GHG’s, and it does that because when it re-radiates the sun’s energy back into space, some of that energy is absorbed by the atmosphere and half is re-radiated back to earth. BUT what is re-radiated back is only half of what the earth radiated out, so that energy can’t “warm” the earth.
If you give me two dollars, and I give you back one, you aren’t richer than you were before you gave me the 2, though you are less poor.
http://www.dailymotion.com/video/x44j0l_abbott-costello-the-loan_shortfilms
The same with the energy arriving from the sun. In a fixed time, the earth receives, say 10, of which it gives 2 to the air, and the air gives back 1, so the earth now has 9, as opposed to only 8 if the air didn’t give any back.
The involvement of the air, then, isn’t to warm the ground, but to slow it’s cooling, which will in turn keep the air warmer for a longer time. The question then is, how much longer and what difference does it make? I.e., how much does increased [CO2] slow the heat loss? – And if water vapor does the same thing, and does it so much more effectively than CO2, why are we worried about CO2?
ASIDE – this plot of earth’s climate history…
http://net33.com/images/earth_co2_temp_history_millions.gif
…is often used as a demonstration of how CO2 has been higher in the past, but what strikes me about that figure is how (1) there seems to be a maximum temperature of 22DegC which (2)has NEVER (except for a single blip) been exceeded, and how (3) we are currently nowhere near that max. If that means the earth CAN’T get any hotter than 22DegC, no matter how much CO2 is added to the atmosphere, and (4) the earth has survived that temperature throughout MOST of it’s history, we must question the motives (and/or intelligence?) of those who tell us it will be a disaster.

John Galt
February 27, 2009 8:08 am

It is also worth noting that the claim that AGW relies that heavily on the climate models, especially for the basic prediction of the magnitude of the warming expected, is overstated anyway. The IPCC’s estimate of the climate sensitivity is based mainly on observational data. In some cases, climate models are used to help interpret what sort of climate sensitivities that data implies…but they are not generally used directly to predict the climate sensitivity. (Although the range of climate sensitivities that exist in the current models do tend to mirror the range that is found to be most likely based on the observational data.)

I’ll grant that perhaps the scientific community does not put as much emphasis in the climate models as do the laymen, including the press and the politicians, do.
Many people feel the term ‘alarmist’ is insulting, but it’s a very proper description of Gore, Lauri David, Sharon Begley and Seth Borenstein. Unfortunately, I’ll have to put that label on Dr. Hansen as well.
Most of these people aren’t scientists, but Hansen is (or at least used to be) as scientist who relies heavily upon his models as the basis for his prophecies of doom.
Laymen also tend to label Mann’s fictitious hockey stick as climate model, which it is not.
Regardless, the evidentiary support for the models is quite weak. I’ll also repeat my assertion that the GC Models forecast global warming because that’s what they are programmed to do.

MartinGAtkins
February 27, 2009 8:23 am

Mike Borgelt (01:42:52) :

Mars has something like 30 times the mass of CO2 over each square meter than has Earth.

I’m not sure of the mass of CO2 psm compared with Earth but that is more or less irrelevant. The surface gas pressure is more important for the atmospheric energy content psm. Surface pressure on Mars varies wildly but it’s thought to be about 7.0 to 8.0 millibars average. Compare this to 1000 millibars for Earth. The Mars atmosphere has considerably less capacity for energy retention than Earth.

Mars ought to be a pretty good planetary model to figure out just how much “greenhouse” effect there is from CO2 alone.

Provided the planetary model is just for Mars then yes we should be able to model what role CO2 plays in it’s climate.

Yet the numbers I’ve seen for the Martian surface temperature ascribe a “greenhouse” effect temperature increment of between 5 and 10
deg C.

Surface temperature and atmospheric temperature are two different things. According to the source I’m using The Mean surface temperature is -63°C and the Maximum surface temperature 20°C. I find it difficult to imagine that CO2 would add 50 to 25 percent to the surface temps unless we invoke the IPCCs magic molecule hypothesis.
Source for my reply here.
http://www.solarviews.com/eng/mars.htm

February 27, 2009 8:30 am

AJ Abrams (05:06:05) :
Does the temperature of Venus, Mars Jupiter atmosphere at 1ATM correlate to earth’s average temperature after adjusting for distance from the sun?
We simply do not have the data yet for this. In a couple hundred years when we have temperature records for the other planets the answer will be obvious.

gary gulrud
February 27, 2009 8:31 am

E.M.Smith:
“Does anyone know how they get the CO2 measure to only measure CO2, given that it’s really an IR measure?”
They alternate ‘known samples’ with their in situ readings. They attempt to rid the sample of H2O with an ‘cold trap’.
Moreover, their ‘peak emissivities’ over the narrow absorption bands co-opt the term, derived to express the relation of the ideal black body with real materials, renaming this empirical constant “total emissivity”. The original relation is not maintained in any fashion.

jae
February 27, 2009 8:41 am

foinavon:
“Now we know that this interpretation was incorrect. We can measure the water content of the troposphere (upper and all) and determine that in the real world the upper troposphere is moistening (as models predict in fact)[*****]. So we reject Lindzen’s failed hypothesis in this particular respect, since it is shown not to be correct. There’s no problem with that. It’s part of science. However what we shouldn’t do is to retain analyses that we know to be incorrect because we happen to like their implications.”
Oh, yeah? http://www.drroyspencer.com/
And why is it not warming?

February 27, 2009 8:41 am

John Galt (08:08:54) :
I’ll also repeat my assertion that the GC Models forecast global warming because that’s what they are programmed to do.
just like the computers producing the accurate astronomical ephemeris used for navigation on Earth and in Space, they just do what they are programmed to do.

bsneath
February 27, 2009 8:46 am

“An earth with no CO2 would be very cold. The first few tens of PPM produce a strong warming effect, and increases after that are incremental.”
I need help understanding this.
10 ppm = 1 part per 100,000 = one molecule of CO2 in every 100,000 molecules of atmosphere.
1 degree Celsius = a change of about 1 per 300, given that absolute zero in about -272 degrees. This ratio is 3,333 times greater than above.
Therefore how is it that just one molecule of CO2, when added to 100,000 molecules of an atmosphere containing various other gases, will have the power to raise the temperate by a factor of 3,000?
I suspect I am violating a few thermodynamic laws in my questioning, but if someone could explain this in a common sense fashion, then perhaps I would be less of a doubter.

JamesG
February 27, 2009 8:46 am

Joel
Thank you. I’m certain that there’s some good science in all of this and the truth will out someday but many of the papers I’ve read seem to base their case on an inadequately proven assumption and just let that assumption lead them to a conclusion. More often than not, the poor quality data doesn’t support either the assumption or the conclusion. Yet this ragtag bunch of reasonable estimates is often misrepresented as being “overwhelming evidence” based on observations. In fact though, the majority is of these papers are chock full of caveats. And when the data doesn’t fit the theory then sometimes an accommodation is made by adding some unknowable factor which is then also estimated – aerosols for example. Or sometimes the data is processed by a statistical filter which gives the desired correlation. Yes I well know this is how science advances but does this conglomeration of unproven, faltering science really constitute sufficiently adequate knowledge to make one 90% certain of anything?
I’ve heard the argument before about a lower actual CO2 effect giving a higher CO2 sensitivity and I took the trouble of asking Wm Connelley how this seemingly perverse notion was arrived at. He produced a simple equation based on an electric circuit feedback view of the world’s climate. I argued that if the climate was actually slightly more complex (as indeed several independent peer-reviewed papers listed on one of Roy Spencers presentations had argued) then this claim of higher sensitivity for a lower CO2 effect was then wrong. He didn’t respond. Again it’s about the assumptions leading the conclusions. If you can’t think of another amplifier then let’s say it’s CO2. I find it to be a pretty lame argument.

MartinGAtkins
February 27, 2009 8:54 am

Fools rush in where angels fear to tread.
My last message suggested that the 20C maximum temp of the surface of Mars would have 0C as the base line. Therefore CO2 adding 5 or 10 deg C would be an increase of 25 to 50 percent.
This is wrong as the notional base temp would be 0 Kelvin and the max surface temp 293 K. Therefore if +10 deg C was the total added to the surface temp it would be closer to 3.5%.

Robert Wood
February 27, 2009 8:57 am

CO2 and other “GHGs” don’t produce warming; they prevent cooling, which is a very good thing.

Richard
February 27, 2009 8:58 am

Joel Shore (17:20:42) :
“Well, I count a few things. First of all, your numbers are confused. The fractional amount of CO2 in atmosphere by volume is ~0.000385, which is 0.0385%. Since it was ~0.0280% in pre-industrial times, we have raised the level by ~0.010%.”
Before getting to the radiative capacities of CO2 – can you explain what part of the chart created from IPCC data I’ve misinterpreted? The data clearly show that “Global annual man-made CO2 emissions” are .00053% by mass, or .000348% by volume.
Of course Beck and Slocum both dispute your claim that pre-industrial was ~0.0280%. Beck averaging 90,000 accurate chemical analyses of CO2 levels in the air recorded from 1812 through 1961, shows a peak of .0440% in 1820-1825, long before industry made its contribution, and another .0440% peak in 1945.
My calculation of radiative energy man-made CO2 (.000348) will absorb and radiate (in either direction) should coincide with observed global temperature rise – .000348 * .08 (percentage IR that CO2 absorbs) or 0.00002785. My understanding is that thermodynamics of a number this small has no statistical or measurable effect on global climate.

gary gulrud
February 27, 2009 9:08 am

“I need help understanding this.”
Actually, we all do. The original model for this problem using the Stefan-Boltzmann constant comes from Arrhenius, a chemist of note at the end of the 19th century.
His calculations put the temperature of the earth without GHGs, particularly CO2, at 255 degrees C, where the real average temp was thought 283 degrees for a 30 degree difference. The result seemed to fit reality.
His model however did not include the oceans, and his calculations included 30% of TSI reflected back into space. Beyond 7th grade Science the model is worthless. We don’t really have a good model to date.

Joel Shore
February 27, 2009 9:14 am

AJ Abrams says:

Something was mentioned twice here that was never addressed by the normal folk that inhabit this site –
Pressure.
Does the temperature of Venus, Mars Jupiter atmosphere at 1ATM correlate to earth’s average temperature after adjusting for distance from the sun?

Having a certain pressure does not get you around having to obey radiative balance. That is, you have a certain amount of energy coming from the sun to the planet and a certain amount escaping from the planet into space and (barring there being some extreme source of heat in the planet itself) the two have to balance. The amount escaping from the planet into space is determined by the Stefan-Boltzmann Law, i.e., that the power radiated is proportional to the 4th power of the temperature If the temperature of the planet’s surface is higher than what that Law implies it must be to maintain radiative equilibrium, that must mean that the effective location from which the emission into space is occuring is not the surface but rather some other altitude where the temperature is cool enough that the radiative equilibrium holds. This in turn requires that the atmosphere must be IR-active, i.e., that it must be absorbing and re-emitting electromagnetic radiation, that is, essentially that there must be a greenhouse effect.

Bill M
February 27, 2009 9:20 am

I’m a science writer and a fan of science. I’ve read all the posts to this point and I’d like to say I appreciate the conversation immensely.
I’d also like to say, to anyone wandering through the mix, to search for a post by ‘cal’ which very clearly states the issue in a basic fashion.
Oh, and thanks for all the material, I have bills to pay.
Bill

February 27, 2009 9:27 am

gary gulrud (09:08:28) :
His model however did not include the oceans, and his calculations included 30% of TSI reflected back into space.
This is an enduring myth. The 255K comes about this way:
He equates incoming and outgoing energy. Incoming energy at the surface [incl. the oceans] is Fi = S/4 (1 – A) where S is TSI [1361 W/m2] and A is the albedo [=0.3]. For an average blackbody outgoing energy is Fo = aT^4. Equating Fi and Fo gives for temperature T: T=((1-A)S/(4a))^(1/4). Inserting values, gives T = 255K.

February 27, 2009 9:42 am

Substandard… That’s the description on Steve’s article. The article is full of assumptions which don’t draw a parallel with reality. For example, experimental results show that increases of the mass of CO2 in the atmosphere lead to decreases of ΔT. The algorithm is not complicated and it is intelligible:
ΔT = Δq / m (Cp)
If we increase mass, without increasing q, i.e. incident Solar Irradiance on the Earth’s surface, delta T would be lower. For example, if the concentration of atmospheric CO2 increases x2, the change of temperature would be:
ΔT = 0.0786 J / (0.00138 Kg) (841 J/Kg K) = 0.07 K
If we apply the modified Arrhenius’ Equation, the preferred algorithm of IPCC, the things don’t run better:
ΔT = α (ln 2) / 4 (σ) (K3)
ΔT = 0.423 (W/m2) (0.7) / 4 (5.6697 x 10-8 W/m2 K4) (300.15 K3) = 0.3 W/m2 / 6.132 W/m2 K = 0.05 K
These calculations clearly show that the assumption on that higher changes of temperature obey to higher concentrations of atmospheric CO2 is absolutely false. I didn’t include L and m in Arrhenius’ equation because I opted to compare my equation with the equation from AGW proponents. The difference is that I have used real data, parameters and constants.
Trust me, the physics on which AGW proponents base their “greenhouse” effect is odd; I would say it is out of this world and this Universe… 🙂

February 27, 2009 9:50 am

Correction: ΔT = 0.423 (W/m2) (0.7) / 4 (5.6697 x 10-8 W/m2 K4) (300.15 K)3 = 0.3 W/m2 / 6.132 W/m2 K = 0.05 K

tmtisfree
February 27, 2009 9:56 am

Concerning the downwards LW radiation and its interaction with surface. There are apparently 5 possible mechanisms:
1/ One mechanism is for the activated molecule to collide with another molecule, and to drop back into a lower energy state; the energy thus freed becomes kinetic energy of the molecules and corresponds to warming the gas. This is absorption. The photon is permanently lost or attenuated from the radiation field.
This is not the case as surface is warmer than atmosphere.
2/
A second mechanism for release of the energy increase is the spontaneous transition of the molecule [in the surface] (in about one nanosecond) into its original state by emitting a photon which is identical to the absorbed one except for its direction of propagation. This is scattering, where the photon remains part of the radiation field but the direct beam is attenuated.
This means the LW photon is absorbed and instantaneously re-emitted upwards. Clearly possible. The atmosphere is transparent to this LW radiation.
3/
A third mechanism is the activated molecule releases its energy spontaneously but in two stages. Two photons with different lower energies result; the sum of the energies of the two photons equals the energy of the absorbed photon. The direct beam is attenuated; the original photon has been replaced by two photons at longer wavelengths and is no longer part of the radiation field. This is Raman-scattering.
Don’t know if this occurs, but seems identical to 2/. The 2 re-emitted photons are LW radiation and again atmosphere is transparent.
4/
Other mechanisms for energy release are fluorescence and phosphorescence. These occur when the energy is not released spontaneously, but after relaxation times of nanoseconds to hours.
Is it possible? Additional informations needed.
5/
Another basic type of interaction involves the conversion of molecular kinetic energy (thermal energy) into electromagnetic energy (photons). This occurs when molecules are activated by collisions with each other and the activation energy is emitted as photons. This is emission.
This is the mechanism of radiation emission by surface after solar irradiation. It is not possible to occur for the downwards LW photon as it has first to be absorbed by surface and 1/ prevents this absorption.
It seems that the only possible mechanism of interaction of the downwards LW photon with surface is the instantaneous re-emission of an identical LW photon. The atmosphere is transparent for this LW photon meaning that it escapes directly in space.
Feel free to add information or correct.
Bye,
TMTisFree

AJ Abrams
February 27, 2009 9:56 am

Joel,
“If the temperature of the planet’s surface is higher than what that Law implies”
You skipped my question. I know that the greenhouse theory is about time lag for escaping energy, that wasn’t my question.
I asked this in essence: What is the expected temperature of earth given it’s 14.7 psi at sea level? Your comment about altitude temperature is a bit of a red herring here because in atmosphere there are other mechanisms to explain that easily.
However….if, as I have read that the earths, venus’, mars and even jupiter’s temperatures at 1 atm are within expectation given the pressure it would mean that pressure is the regulator. You could have some variance due to other mechanisms like TSI, latent ocean heat + circulations etc just as we see on earth, but there would a cap of the lower and upper boundaries of those boundaries because of PV = nRT . To get above or below expected temperatures you’d have to add or reduce drastically the mount of gas on earth which isn’t going to happen internally. It would have to be from the sun via a large change in solar output or from a collision with a forgein body that either added matter, or blew off atmosphere.
Since that isn’t likely, you are constrained from any sort of run away temperature change on earth.
People often give Venus as some example of “run away” greenhouse, but as I see it Venus suffered from run away increase in pressure. The overall temperate from what I’ve read over the last few days seems to be almost completely explained by it’s pressure at surface, and internal heating from a thin crust as well as some other minor influences. The “greenhouse” effect might be an influence, but not, by far, a dominant one.
And before someone says it was the CO2 increase that increased temperatures that in return increased pressure, I highly doubt that. For one Venus never had a fast rotation given it didn’t have a giant collision with a planet sized object like earth did -i.e the moon. Given it’s slow rotation added to it’s closer proximity to the sun, it was only a matter of time before H2O disappeared. Once that happened CO2 pressures would go up (because it would have no sink). Once CO2 started gathering in numbers the pressure went up creating more heat which caused more CO2 to gas out.
Poor Mercury never had the size or magnetic shielding to gather an atmosphere.
This scenario isn’t possible here on earth for numerous reason (rotational speed being one of them and total TSI being another) until our sun starts to grow some and seeing that is out of human control, I think I’m ok.
Am I missing something here people not named Joel?

tmtisfree
February 27, 2009 10:02 am

Arg, missed the end of tag. It would be cool to have a preview on this blog.
Concerning the downwards LW photon and its interaction with surface. There are apparently 5 possible mechanisms:
1/ One mechanism is for the activated molecule to collide with another molecule, and to drop back into a lower energy state; the energy thus freed becomes kinetic energy of the molecules and corresponds to warming the gas. This is absorption. The photon is permanently lost or attenuated from the radiation field.
This is not the case as surface is warmer than atmosphere.
2/A second mechanism for release of the energy increase is the spontaneous transition of the molecule [in the surface] (in about one nanosecond) into its original state by emitting a photon which is identical to the absorbed one except for its direction of propagation. This is scattering, where the photon remains part of the radiation field but the direct beam is attenuated.
This means the LW photon is absorbed and instantaneously re-emitted upwards. Clearly possible. The atmosphere is transparent to this LW radiation.
3/A third mechanism is the activated molecule releases its energy spontaneously but in two stages. Two photons with different lower energies result; the sum of the energies of the two photons equals the energy of the absorbed photon. The direct beam is attenuated; the original photon has been replaced by two photons at longer wavelengths and is no longer part of the radiation field. This is Raman-scattering.
Don’t know if this occurs, but seems identical to 2/. The 2 re-emitted photons are LW radiation and again atmosphere is transparent.
4/Other mechanisms for energy release are fluorescence and phosphorescence. These occur when the energy is not released spontaneously, but after relaxation times of nanoseconds to hours.
Is it possible? Additional informations needed.
5/Another basic type of interaction involves the conversion of molecular kinetic energy (thermal energy) into electromagnetic energy (photons). This occurs when molecules are activated by collisions with each other and the activation energy is emitted as photons. This is emission.
This is the mechanism of radiation emission by surface after solar irradiation. It is not possible to occur for the downwards LW photon as it has first to be absorbed by surface and 1/ prevents this absorption.
It seems that the only possible mechanism of interaction of the downwards LW photon with surface is the instantaneous re-emission of an identical LW photon. The atmosphere is transparent for this LW photon meaning that it escapes directly in space.
Feel free to add informations or correct.
Bye,
TMTisFree

Joel Shore
February 27, 2009 10:31 am

Richard says:

Before getting to the radiative capacities of CO2 – can you explain what part of the chart created from IPCC data I’ve misinterpreted? The data clearly show that “Global annual man-made CO2 emissions” are .00053% by mass, or .000348% by volume.

Oh…Okay. Now I understand. This is the number for the amount by which we are ANNUALLY adding to the level of CO2 in the atmosphere. Yes, CO2 levels are increasing by somewhere around 1.5 ppm per year. (The number you quote is about double this because only about half of our emissions are accumulating in the atmosphere, with the other half being absorbed by the oceans and biosphere.) However, note that the amounts accumulate over time and what I was giving you was the total amount by which CO2 levels have changed as a result.

Of course Beck and Slocum both dispute your claim that pre-industrial was ~0.0280%. Beck averaging 90,000 accurate chemical analyses of CO2 levels in the air recorded from 1812 through 1961, shows a peak of .0440% in 1820-1825, long before industry made its contribution, and another .0440% peak in 1945.

No serious scientist believes this garbage. It is based on measurements that are known to be bad…and involves believing that the carbon cycle behaved totally differently until Keeling started his measurements and then miraculously settled down. You are welcome to believe Beck if you want but you will not be taken seriously by anyone in the scientific community.

My calculation of radiative energy man-made CO2 (.000348) will absorb and radiate (in either direction) should coincide with observed global temperature rise – .000348 * .08 (percentage IR that CO2 absorbs) or 0.00002785.

I don’t know what to tell you except that your calculation is completely and utterly meaningless. It is just numerology with absolutely no physical basis behind it.

Stevo
February 27, 2009 10:39 am

Joel Shore,
“Having a certain pressure does not get you around having to obey radiative balance.”
Quite correct. The temperature at the top of the atmosphere is controlled by radiative balance, Stefan-Boltzmann, etc.
The difference between the top of the atmosphere and the surface temperature at the bottom of the atmosphere, however, is caused by pressure. It is the difference between the temperature predicted by pure radiative heat balance (about -18 C) and the actual observed temperature (about 14 C) that we refer to as “the greenhouse effect”. The emission to space is from the troposphere, and in the troposphere convection dominates, forcing the profile towards the moist adiabatic lapse rate. The lapse rate is because of pressure.
Without convection, and treating the atmosphere as a series of layers opaque to IR, then the business about each layer radiating at much up as down would mean that for each layer (~ 1 optical depth) you descend, the heat radiated would have to increase by an amount equal to the amount emitted to space at the top. With the increasing density making the layers thinner, this would rapidly lead to a ginormous temperature gradient. But in real air, the moment the gradient exceeds the adiabatic lapse rate, it becomes hydrostatically unstable, and turbulent convection quickly arises to correct the situation.
The absorption of IR and all the complicated MODTRAN quantum calculation of absorption/emission spectra is relevant to the question, because it determines where the IR radiates from. But because all this goes on in the convection-dominated troposphere, the mechanism by which a temperature difference arises is down to the pressure difference (modified by the heat carried by moisture) between the altitude at which IR radiates to space and the surface.
See here for Held and Soden 2000, look at figure 1 and the discussion just above and below it, to see the IPCC-cited version.
http://www.gfdl.noaa.gov/reference/bibliography/2000/annrev00.pdf
AJ Abrams,
Thanks for listening. It’s appreciated.

Joel Shore
February 27, 2009 10:55 am

AJ Abrams says:

You skipped my question. I know that the greenhouse theory is about time lag for escaping energy, that wasn’t my question.
I asked this in essence: What is the expected temperature of earth given it’s 14.7 psi at sea level?

I didn’t skip your question. I just answered it at a more fundamental level. The question that you now ask is not well-defined. PV = NRT has four variables in it. Just specifying P does not allow you to uniquely determine T. The more fundamental physics determining T is that the planet must be in radiative balance with its surroundings.
Your talk about the pressure somehow explaining the high temperature makes no sense at all unless you are trying to suggest that the temperature on Venus is not governed primarily from the energy it received from the sun but is instead governed by radioactivity or continual gravitational collapse or some other form of energy. Good luck with that hypothesis! Such a notion was already dismissed in the mid-70s; see this 1975 paper linkinghub.elsevier.com/retrieve/pii/0019103576901573 entitled “Can internal heat contribute to the high surface temperature of Venus” that concludes:

In summary, this exercise yields an approximate but quantitative demonstration of what everyone suspected anyway–that the maximum contribution to the surface energy budget from internal heat is down by at least one and probably two orders of magnitude from the solar contribution. This conclusion is independent of any model for the source of the internal energy. In the absence of any other plausible heat source, the one calorie per square meter provided each second by the Sun is all there is to maintain a 750°K surface temperature. Thus, we can assert with confidence that a very efficient atmospheric trapping mechanism (greenhouse effect) is operating, whether or not the details of this mechanism are fully understood.

gary gulrud
February 27, 2009 11:14 am

“Fi = S/4 (1 – A) where S is TSI [1361 W/m2] and A is the albedo [=0.3].”
I stand corrected on the detail of energy input to the calculation.

AJ Abrams
February 27, 2009 11:28 am

Joel
Now you are being deliberately obtuse and condescending. Nobody, in their right mind could read what I wrote/asked and think :
“Your talk about the pressure somehow explaining the high temperature makes no sense at all unless you are trying to suggest that the temperature on Venus is not governed primarily from the energy it received from the sun but is instead governed by radioactivity or continual gravitational collapse or some other form of energy.”
In fact…the person that wrote back right above you had no problem at all understanding my question.
Thanks for not answering my question Joel. Please refrain from responding to my posts in the future.
To Stevo – Thank you. What you’ve said about convection made perfect sense and I’ll read your link shortly.

Robert Wood
February 27, 2009 11:34 am

Leif Svalgaard @ 09:27:52
Leif, if he accounts for albedo in the IN direction, why not for the OUT direction. Yes, I realise this is opening up a can of worms.

maryp
February 27, 2009 11:37 am

Global Warming Is A Hoax – Quick and Dirty
February 26, 2009
The facts that Al Gore’s carbon tax collection company doesn’t want you to see.
CO2 is 3.168% of the total greenhouse effect. Around 0.117% is man-made CO2 greenhouse gas effect. CO2 is an insignificant greenhouse gas, especially in terms of man-made contribution. Water is the only significant greenhouse gas
Furthermore, you leave out that oceans and volcanos release more CO2 than humans.
Furthermore, you leave out that CO2 is part of the life cycle.
Futhermore, you leave out that CO2 levels tend to follow warming rather than cause warming.
Furthermore, you leave out the impact of the sun spot activity, which is thoroughly studied, and it’s impact of warming/cooling on the solar system as a whole.
Furthermore, you leave out that the solar system as a whole had been warming.
Furthermore, you leave out that the solar system and earth has cooled down due to lack of sun spot activity this past year.
Furthermore, you leave out that carbon taxes will not solve the problem but will be used to fund global government and bombing of nations.
Furthermore, you leave out that big corporations, especially big oil, actually support the global warming hoax because it helps them eliminate their competition who can’t afford the taxes while big oil gets the big grants from the tax payer to reduce and meet emission standards.
Furthermore, you leave out that carbon taxes will do nothing but hurt the poor.
Furthermore, you leave out that polar bears can easily swim over 100 miles and swim as much as 300 miles. They hunt for seals and buluga whales.
Furthermore, you leave out that the ice caps are always melting and refreezing with the seasons.
Furthermore, you leave out that the government was caught making up temperature data to make it seem the planet was warming rather than cooling this past year.
Furthermore, you leave out that scientistics receive tons of funding by promoting global warming scares while skeptics don’t receive funding.
Furthermore, you leave out that the Al Gore hockey stick graph was generated using a statistical analysis computer algorithm that weighted data towards the mean the further out in history you go.
Furthermore, you leave out all the various drastic cooling and warming trends throughout the history of the Earth.
Furthermore, you leave out that global warming is actually preferred over global cooling because food production increases, namely because CO2 helps plant growth, absorbing that 0.117% man-made CO2 like candy. Not to mention that you’re more likely to freeze to death than die of heat stroke.
Furthermore, statistics are lies, lies, and damned lies. Scientific models don’t represent reality. Any decent college professor in scientific modeling and statistics classes will tell you these things on the first day of class.
In conclusion, you’re being scammed by the New World Order with this global warming hoax to pay carbon taxes to them, and you should be mad as hell.

Robert Wood
February 27, 2009 11:37 am

Following up on my previous post, shouldn’t this 255K then represent the TOP of the atmosphere, not the surface, which would be warmer.

February 27, 2009 12:15 pm

Robert Wood (11:34:12) :
if he accounts for albedo in the IN direction, why not for the OUT direction.
Because what comes in must go out eventually, so the IN-direction sets the amount.
Robert Wood (11:37:37) :
Following up on my previous post, shouldn’t this 255K then represent the TOP of the atmosphere, not the surface, which would be warmer.
The TOP of the atmosphere is out in space somewhere, but if we stick to the lower 10 miles or so, they are warmed from the surface, because the air is largely transparent to incoming radiation. Stand on a [tall] mountain, you get the same amount of incoming [actually a bit more – you sunburn quicker], yet it is usually cold up there, perhaps there is even snow on that mountain, but not down in the valley.

jae
February 27, 2009 1:30 pm

“linkinghub.elsevier.com/retrieve/pii/0019103576901573 entitled “Can internal heat contribute to the high surface temperature of Venus” that concludes:
In summary, this exercise yields an approximate but quantitative demonstration of what everyone suspected anyway–that the maximum contribution to the surface energy budget from internal heat is down by at least one and probably two orders of magnitude from the solar contribution. This conclusion is independent of any model for the source of the internal energy. In the absence of any other plausible heat source, the one calorie per square meter provided each second by the Sun is all there is to maintain a 750°K surface temperature. Thus, we can assert with confidence that a very efficient atmospheric trapping mechanism (greenhouse effect) is operating, whether or not the details of this mechanism are fully understood.”
I would like to read that paper; is it available for free somewhere?
It takes a lot of energy to keep the atmosphere in the state that it’s in, and that energy must be measurable as temperature. Probably about 18 C more than sans atmosphere 🙂

jae
February 27, 2009 1:33 pm

Stevo:
“See here for Held and Soden 2000, look at figure 1 and the discussion just above and below it, to see the IPCC-cited version.
http://www.gfdl.noaa.gov/reference/bibliography/2000/annrev00.pdf
That little discussion has been used thousands of times. Unfortunately, it ignores details like convection.

Reed Coray
February 27, 2009 1:48 pm

Mark T (22:56:04)
I agree with you that the concept of increasing temperatures causing a release of CO2 which causes an additional increase in temperature seems like a legitimate single pole feedback. But like you, I don’t know either (a) what delay to assign to this phenomenon, or (b) what multiplicative factor to apply. I also agree that such a feedback loop would be stable if the absolute value of it’s multiplier was less than one. Finally, I agree that at certain frequencies, such a stable loop will likely have a finite gain larger than one. But just a comment. For any frequency other than zero, although the gain may be larger than one, the output must cycle through positive and negative values. If the system inputs/outputs are degrees Kelvin, negative temperatures are a physical impossibility. I have no idea how to overcome this apparaent contradiction other than to assume such a feedback system applies to “temperature changes” relative to a “nominal value”. Provided the gain at all non-zero frequencies is “small enough” that the sum of the “nominal value” and the most “magnitude” of the output at that frequency is positive, such a system wouldn’t contradict the fact that zero degrees Kelvin is as low as temperatures get.
Like you, I am confused. That is why I asked for a circuit diagram or a flow diagram or whatever someone wants to call it. Without such a diagram, I don’t know how to attack the problem. I have no, zero, nada plans to try to create such a diagram because (a) if I did, it would be incorrect, and (b) it would Agive GW supporters ammunition to argue that the AGW-denier Reed Coray doesn’t know what he is talking about–which is probably true anyway.
Finally, given how complex the environment is, my belief is that it can’t be represented by a linear, time-invariant system. It may be possible to “linearize” the environment over a small temperature range; but I even doubt that.
Best wishes, Reed Coray

Reed Coray
February 27, 2009 1:55 pm

Anthony,
It has been suggested that the term “AGW-alarmist” is pejorative. Although I like to use humor and sarcasm to express some of my thoughts, I don’t want to cross the bounds of decency. Since this is your blog, I’ll let you be the judge. Specifically, for posting on your blog, is the term “AGW-alarmist” unacceptable?
REPLY: Or simply “alarmist” either works, thanks for checking – Anthony

Joel Shore
February 27, 2009 1:55 pm

jae says:

That little discussion has been used thousands of times. Unfortunately, it ignores details like convection.

No…As Stevo nicely explained it is convection that is important in controlling the vertical thermal gradient that exists in the troposphere. Hence, it is not in fact being ignored at all.

Joel Shore
February 27, 2009 1:59 pm

jae says:

I would like to read that paper; is it available for free somewhere?

I believe that you can just set up a free account at that ScienceDirect website and then you’ll have access to it free…at least it seemed to work that way for me.

Simon Evans
February 27, 2009 2:42 pm

is the term “AGW-alarmist” unacceptable?
REPLY: Or simply “alarmist” either works, thanks for checking – Anthony

Can I check also, Anthony? Is the term ‘denialist’ acceptable? Personally I think good discussion will be undermined by assigning labels, but I’d like to know if you consider it ok on one side only or on both.
REPLY: Denialist is not, due to its connotations with “holocaust denier”. Alarmist has no such connotation. Similarly “septic”, sans the c or k, depending on regional use, is not acceptable either. The two preferred terms are “skeptic” and “alarmist” or alternately if you prefer “AGW’er” Simon, let me clear on this point, no further discussion of it since I see your intent. I’m not interested in a debate on semantics with you. My blog, my rules. Abide or abstain. – Anthony

RobJM
February 27, 2009 2:44 pm

Here is something else that doesn’t make sense
Plug 0 into the IPCC CO2 forcing calculation to determine the total forcing of 280ppm of C02 prior to 20th century (or any concentration for that mater).
Solution
CO2 forcing = infinity w/m2
just slightly fishy!
try it yourself
5.35ln 280/0 = infinity lol

Hank
February 27, 2009 2:46 pm

I don’t get this greenhouse metaphor. Couldn’t we just say that the atmosphere absorbs radiation from the sun as well as the earth. And additionally, since atmosphere is itself matter it emits a certain amount of radiation to itself. To the degree the atmosphere absorbs energy it inhibits radiative cooling of the earth into space.
No one has pointed out that CO2 is used to make infrared lasers because CO2 not only absorbs infrared but emits it as well.

Simon Evans
February 27, 2009 2:59 pm

My blog, my rules. Abide or abstain. – Anthony
Ok, Anthony, I’ll abstain. I’m not interested further in contributing to the sense of ‘authenticity’ of a blog where the proprietor sanctions name-calling. Your statement “The two preferred terms are “skeptic” and “alarmist”” is comically revealing of your intent. I don’t suppose you’ll post this, but you’ll read it, so cheerio. I really do dislike people pretending to be disinterested when their bias is so obvious. Have the balls to post this, why don’t you? It’s my last word anyway – I’ll look for places that have some objective standards to post my views. Not here – not after your active encouragement of name-calling, the base resort of those lacking any substance in their arguments.

February 27, 2009 3:09 pm

Hank: No one has pointed out that CO2 is used to make infrared lasers because CO2 not only absorbs infrared but emits it as well.
Yes, you’re right; CO2 absorbs almost the same amount of IR than it emits. The problem here is that the absorptivity-emissivity of CO2 have been exaggerated as if our atmosphere excerpted a pressure of hundreds atmosphere-meter. At its current Pp, i.e. 0.00034 atm m, the absorptivity of CO2 is 0.001. Just compare this cipher, taken from experimental work, whith the cipher applied by AGW proponents which is 0.75, which, in some cases has been exaggerated to 0.9.

February 27, 2009 3:13 pm

Some literature that will guide you to real parameters:
Bakken, G. S., Gates, D. M., Strunk, Thomas H. and Kleiber, Max. Linearized Heat Transfer Relations in Biology. Science. Vol. 183; pp. 976-978. 8 March 1974.
Manrique, José Ángel V. Transferencia de Calor. 2002. Oxford University Press. England.
McGrew, Jay L., Bamford, Frank L and Thomas R. Rehm. Marangoni Flow: An Additional Mechanism in Boiling Heat Transfer. Science. Vol. 153. No. 3740; pp. 1106 – 1107. 2 September 1966.
Pitts, Donald and Sissom, Leighton. Heat Transfer. 1998. McGraw-Hill.
Potter, Merle C. and Somerton, Craig W. Thermodynamics for Engineers. Mc Graw-Hill. 1993.
Schwartz, Stephen E. 2007. Heat Capacity, Time Constant, and Sensitivity of Earth’s Climate System. Journal of Geophysical Research. [Revised 2007-07-16]
Van Ness, H. C. Understanding Thermodynamics. 1969. McGraw-Hill, New York.
Wagner, F., Aaby, B., and Visscher, H. Rapid atmospheric CO2 changes associated with the 8,200-years-B.P. cooling event. Proceedings of the National Academy of Sciences. September 17, 2002; vol. 99; no. 19; pp. 12011-12014.
Wilson, Jerry D. College Physics-2nd Edition; Prentice Hall Inc. 1994.

February 27, 2009 3:28 pm

Steven Goddard (07:16:36) :
Phil,
Your explanations are appreciated, but appear to be missing something important. If the CO2 is losing all of it’s excitation energy in nanoseconds near the surface, how can there be excited CO2 at 1mb altitudes?

Because at 1/1000th of the pressure the mean time between collisions is much longer and the excited state stays that way for longer and therefore radiation takes place. (think of it as a Poisson process)
According to this chart, the 15um absorption spectrum is saturated.
http://www.globalwarmingart.com/wiki/Image:Atmospheric_Transmission_png

That’s little more than a cartoon at such low resolution that it doesn’t show the individual lines with the gaps between. See below for a portion of the CO2 spectrum, the top graph shows the spectrum at mars conditions and the lower at earth conditions, illustrating the effect of broadening.
http://i302.photobucket.com/albums/nn107/Sprintstar400/Mars-Earth.gif

Reed Coray
February 27, 2009 3:37 pm

Correction to my post (13:48:11). Instead of
Provided the gain at all non-zero frequencies is “small enough” that the sum of the “nominal value” and the most “magnitude” of the output at that frequency is positive, such a system wouldn’t contradict the fact that zero degrees Kelvin is as low as temperatures get.
I meant to say
Provided the gain at all non-zero frequencies is “small enough” that the difference between the “nominal value” and the “magnitude” of the sinewave at the output (“nominal value” minus “output magnitude”) at that frequency is positive, such a system wouldn’t contradict the fact that zero degrees Kelvin is as low as temperatures get.

George E. Smith
February 27, 2009 3:45 pm

“”” gary gulrud (11:14:37) :
“Fi = S/4 (1 – A) where S is TSI [1361 W/m2] and A is the albedo [=0.3].”
I stand corrected on the detail of energy input to the calculation. “””
Gary, applying this expression; which does seem to be standard practice; also is part of the problem.
The relation between temperature and radiant emittance (or absorbtance) is not a linear relation (it is 4th power of T) but other processes such as thermal conduction are linear with temperature (like Ohm’s Law)
So if you average the correct solar irradiance (1367 W/m^2) over 4 times the area, to get that fictitious 342 W/m^2, then you don’t get the correct distribution between radiation and conduction, so you get the wrong answer.
At 4 times the irradiance, the surface temperature gets much hotter, but the conductive flow only goes linearly with the temperature rise, whereas the radiative component goes as the 4th power of the absolute temperature. The two answers aren’t the same, and the hotter it gets the more unequal they are.
George
Which is why I don’t like the “forcings” model. Maybe it is no surprise, that the only standard “Climatology” lingo that I can find in my CRC Handbook of Chemistry and Physics, is “Albedo”.
No tables of forcings, or feedbacks, or climate sensitivities, or anomalies, or any of that; just ordinary conventional Science terms; like Temperature.
So when those Japanese Scientists referred to Pachauri’s UN IPCC stuff as “ancient Astrology”, they perhaps were insulting ancient astrolgy.

Sandw15
February 27, 2009 5:02 pm

There’s some impressive debate going on here. Steve, you did a good job of getting the pot stirred in your attempt to educate some of us who are less up to date. I appreciate it and hope you don’t feel like you’re getting beat up too much. Many of the posts involve issues which seem to be addressed by the link I gave on the previous article regarding CO2 and glacial cycles. I can’t vouch for Dr. Hug’s research but it certainly seems, at least, to have significance relating to the IPPC mention of…what are they called…wings(?) that are frequently invoked.
http://www.john-daly.com/artifact.htm

Joel Shore
February 27, 2009 5:19 pm

RobJM:

Here is something else that doesn’t make sense
Plug 0 into the IPCC CO2 forcing calculation to determine the total forcing of 280ppm of C02 prior to 20th century (or any concentration for that mater).

The formula stating that forcing is logarithmic in concentration is only approximate, although a quite good approximation over the concentrations of interest to us. At low enough concentrations, it is no longer logarithmic but rather linear.

February 27, 2009 5:23 pm

I notice that “Simon Evans” has once again promised to go away, because this site won’t give in to his demand that it should be OK for AGW skeptics to be equated with holocaust deniers.
Well, we’ll see. Last time he said he wouldn’t post here any more, it lasted about 36 hours.

February 27, 2009 5:29 pm

Joel Shore:

The formula stating that forcing is logarithmic in concentration is only approximate, although a quite good approximation over the concentrations of interest to us. At low enough concentrations, it is no longer logarithmic but rather linear.

I seem to recall that those ‘low enough’ concentrations are around 20 ppmv. Since most life on the planet would not survive very low CO2 levels, it’s a moot point, isn’t it?
I’m still waiting for real world evidence, which shows that higher CO2 levels are detrimental in any way.

February 27, 2009 5:30 pm

There is a considerable amount of misinformation propagated about the greenhouse effect by people from both sides of the debate. This is true, but all the graphs and data will not convince joe sixpack of anything. The pros and cons even among the comments here bring questions. Joe sixpack worries about more taxes, does not want to give up his hemi powered pickup, and has no idea what a carbon foot print is. While science provides us with the needed information, Rush, and those like him constantly regail against it. Joe has disregarded the science and believe that climate change is natural, from a wobble in Earths orbit, a change in the sun, or the end of the last ice age. He may even believe there is no climate change at all. Joe believes there is no reason to change anything, as natural, we did not cause it, we can not change it, therefore nothing can be done about it. Going green is foolishness. An electric or hydroelectric car is pointless. If joe sixpack was the republican hero, than Homer Simpson is his archtype. To evolve from the horse drawn wagon to the gas powered automobile was easy, to evolve beyond that is more than Joe can handle. Maybe he has a point, Consider this, In one mans life span we have gone from horse drawn wagon, to the automobile, to the jet airplane, then to walking on the moon. Throw in the atom bomb, Vietnam, and computers, maybe it is more than Joe can handle. With the last President, science was ignored. Perhaps now with President Obama things will change. If the U.S. continues its downward trend regarding it economic downturn, then the climate change problem will be put on the back burner. Europe and Asia could lead the way for change. Greed is greed though. What drove us drives them as well. To Joe anything that changes his lifestyle is a loss of freedom. A seatbelt use law is like a communist plot. Even though it is for everyones good, to mandate any change for going greener, would be viewed as some sort of government restriction on Joes freedom. His motto is family, land and his rifle. Any change to deal with climate change will have to be done with out him. Alkataba

February 27, 2009 6:08 pm

alkataba, whatever are you ranting about??

Joe has disregarded the science and believe that climate change is natural, from a wobble in Earths orbit, a change in the sun, or the end of the last ice age.

Contrary to your misguided understanding of the scientific method, it is not the accepted status quo that has the burden of defending the long held theory of natural climate variability; the burden is on the AGW/CO2 “tipping point” fanatics to explain how the climate is acting differently now than it has previously. So far, they have failed to show anything that is not explained by completely natural climate change.
Commenters on this “Best Science” site repeatedly ask why no one has ever been able to falsifiy the hypothesis that the observed temperature changes are the result of entirely natural climate variability.
But hey, maybe you can be the first to falsify that accepted theory.
See, until someone provides convincing evidence that the current climate is not fluctuating naturally, and well within normal parameters, then it’s all red-faced, AGW/CO2 flying-spittle “what if” arm-waving on one side of the argument, and ho-hum on the other.

Joel Shore
February 27, 2009 6:43 pm

Smokey says:

I seem to recall that those ‘low enough’ concentrations are around 20 ppmv. Since most life on the planet would not survive very low CO2 levels, it’s a moot point, isn’t it?

I suppose. I was just answering a question from RobJM who was wondering why you get an infinite result when you use the logarithmic formula to try to calculate how much forcing is occurring relative to the case of 0 ppm CO2.

Sandw15
February 27, 2009 6:59 pm

Alkataba
“Even though it is for everyones good, to mandate any change for going greener, would be viewed as some sort of government restriction on Joes freedom. His motto is family, land and his rifle.”
It’ll be ok Al. Drink some chamomile tea and have a good night’s sleep. Maybe tomorrow you’ll be able to find the blog you meant this post for.

Bill Illis
February 27, 2009 7:06 pm

Joel Shore says the logarithmic formula should not be used for low concentrations of CO2. (CO2 has never been below 180 ppm so why should the formula and a chart showing these very unlikely low values be used at all?)
Because showing the entire spectrum of possible CO2 values provides an important insight into what the warmers are assuming for the “more normal” values of CO2.
They are assuming of course that CO2 is responsible for nearly the entire greenhouse effect. As CO2 creates an increase in atmospheric temperatures, the atmosphere is “capable” of holding more water vapour and the increase in temperatures does, in fact, result in a proportional increase in the water vapour content of the atmosphere.
The increased water vapour content then results in an amplified greenhouse effect. Since water vapour is the most potent greenhouse gas.
So when this post and others cites 33C as “the greenhouse effect”, it is important to note that the current estimates of 3.0C per doubling of CO2 assumes CO2 is responsible for nearly the entire greenhouse effect because water vapour will increase in lock-step with any increase in CO2.
If CO2 is Zero, then water vapour will effectively be Zero as well and we don’t want a chart shown to the general public which demonstrates this impossibility.
I don’t like theory that only applies to a small slice of possible values because that could indicate the theory is not “robust” – to use a warmers term, and could easily be faulty. The math needs to work backwards and forwards or there is likely a problem in the math.
In the case of the IPCC theory of global warming theory, CO2 is THE ONLY GHG because it is controls water vapour as well.
The empirical evidence to date does not support this assertion of course.
Here is the chart Joel Shore doesn’t want you to see.
http://img216.imageshack.us/img216/9652/logwarmingillustratedkn7.png
And here is how the data to date stacks up on those two lines – Zoomed-in to the Joel-favoured limited range of CO2.
http://img254.imageshack.us/img254/2626/tempobsrvvsco2ct4.png
One could say at this point that the estimates should be reduced in half.

pyromancer76
February 27, 2009 7:10 pm

Steve Goddard, thanks for a post that has generated 447 comments at the time of mine. When I first read it, I disagreed with a number of issues you raised. (Actually, I felt annoyed.) However, after living through the many knowledgable “rebuttals” of aspects of your argument, I have been able to re-educate myself. Yes, the main science issue today for combatting the nonsense of AGW-Climate Change is agreeing on the “real warming” effect of CO2 and deciding if there are any “forcings” or “feedbacks” –such strange animals. I see these molecules dancing around and slinging their LWs back at earth ad infinitum. Warm until you tip!
This is a real stretch for me, but doesn’t the basic physics of all the atmospheric gases together solve the problem?
I still believe that your acceptance of the idea of CO2 as a “Greenhouse Gas” complicates the problem of communication to a large audience. Analogies are great, but this one is a real loser. It reinforces the idea of human/divine omnipotence for good or ill regarding the climate. The idea of a greenhouse will immediately call up an image of Someone building those glass walls and someone else adding the gas. Flash point. Tipping point. I think we need to return to clear scientific terms.
Thanks for being willing to put your ideas forward to all us rowdies.

Joel Shore
February 27, 2009 7:11 pm

DR says:

Joel Shore,
You did not reply to my post on Gavin Schmidt et al 2005 quoted thusly:
“Tropospheric warming is a robust feature of climate model simulations driven
by historical increases in greenhouse gases (1–3). Maximum warming is predicted
to occur in the middle and upper tropical troposphere.”
http://www.osti.gov/energycitations/servlets/purl/881407-xk2Sdg/881407.PDF

I replied in my post of 17:36:27 (26/02/2009).

You can’t have it both ways. Are GCM predictions, which are driven by increases in GHG emissions, correct or not?

The GCM predictions and observational data appear to agree well on tropical tropospheric amplification when one looks at fluctuations on timescales of several months to at least a few years. There appears to be some discrepancies between the models and many of the data sets for the multidecadal trends but the data sets have known problems in determining these trends and in fact do not even agree well with each other. Progress has been made on reconciling these discrepancies although the issue cannot as of yet be said to be resolved. This is not surprising as in any scientific field at any given time there are certain puzzles that scientists are working to understand better. If climate science weren’t that way, it would be pretty boring scientifically!
And, finally, while the question of how well the models handle tropospheric amplification is an important one in regards to how faithful the models are in reproducing the earth’s climate system, particularly convective processes in the tropics, any apparent discrepancy between data and models has essentially nothing to say in regards to the mechanism responsible for the observed warming since the prediction of tropospheric amplification appears to a feature of the models that is independent of the mechanism causing the warming.

J. Peden
February 27, 2009 7:17 pm

His [Joe 6-pack’s] motto is family, land and his rifle. Any change to deal with climate change will have to be done with out him.
Halleluja! At last, to be finally left alone – thankyou, thank you, thank you – while Alkataba [pbuh] bravely sallies forth without us alcoholic white trash, “[de]evolving” back to the era of Horse-drawn carriages, and, Yea, beyond that even to The Garden, noblely forsaking his family, land, and rifle to “save the World” and, yes, preventing “the destruction of Creation” by coal carrying railroad cars!
Alklataba [pbuh], may your Fairy Tale end as full flowered as it begins! Burp…Ma, git me another one o’ them thar carbonated beverages.

jae
February 27, 2009 7:17 pm

The AGW folks admit to a logarithmic (or something close) relationship between CO2 concentration and the mysterious greenhouse effect. But I cannot, for the life of me, find similar discussions about the relationship between water vapor levels and the mysterious greenhouse effect. Why? Water vapor is present in ppm levels around 25,000; whereas, CO2 is down around 385 ppm. [snip] Come on “climate scientists,” please explain the relationship between the most important “greenhouse gas” and temperature. This should be a priority in research in the new “transparent” administration, no? This whole “climate science” “science” ain’t “science” at all!

Steven Goddard
February 27, 2009 7:22 pm

Phil,
I don’t think you understood my question. How did the CO2 get excited at 1mb in the first place? What event caused it to get excited?

aurbo
February 27, 2009 9:09 pm

Re: AJ Abrams (09:56:51) :
You’ve described what I was thinking back when I posted at aurbo (11:42:45) which was Boyles Law:
PV= nRT
Once some sort of radiation balance is acheived, one should be able to discern T from simply knowing P and V. But In the planetary case when dealing with a single phase (gaseous) compound and in the absence of monopolar changes in the atmospheric mass of the compund over time, V should be essentially a constant. at any given altitude. Thus, if V=~ a constant, PV then =~ a constant and T can be obtained by simply knowing P.
Since no part of Mars’ atmoshere reaches 1 atm, and Mercury has no atmosphere, we are left with Venus as the only nearby planet on which to test this idea. Some of the data I’ve seen puts the temeprature at the 50km height, where the pressure is roughly 1 bar, at between 300°-400°K this is warmer than Earth’s ~288°K. However, no error bars are availble for the Venus estimate.
In a gaseous atmosphere, PV determines T. If either P or V can be held constant, then T is determined by the other…P or V.

DR
February 27, 2009 10:23 pm

Joel Shore,
I have read the Schmidt et al 2005 paper thoroughly, so drop the patronizing. Point me to the data and the models that correlate in support their claims. I don’t give a rat’s behind what authors of any paper say; they say a lot of things, and most end up being wrong. I want to see the numbers.
No amount of obfuscation on your part can change the facts. The “hot spot” has been thoroughly discussed over at CA.
http://www.climateaudit.org/?p=4731
The recent Dessler et al 2008 paper on water vapor feedback resides in the same vain. How does such garbage make it past reviewers? It is a polished turd like so many pro-AGW material being pushed through so-called “peer review” in recent years like crap through a goose. It seems to have started with the hockey stick.
I will also point, after failing to show the “hot spot” exists via UAH satellite and balloon data (which do correlate), it shifted to claiming thermal winds finding the non-existent “hot spot” which you say doesn’t matter. A recent submitted paper on the matter in direct conflict with the “high impact” journal ‘Nature’:
http://www.climatesci.org/publications/pdf/R-342.pdf
Herman et al UAH vs RSS
http://www.agu.org/pubs/crossref/2008/2007JD008864.shtml
BTW, you are aware Tamino conceded on this issue after a long UAH bashing thread when Herman and Douglass kindly put him in his place?
What’s next, Hansen’s “smoking gun” is also not a problem? How many mulligans does CO2 AGW get?
http://pubs.giss.nasa.gov/docs/2005/2005_Hansen_etal_1.pdf
A very simple yes or no answer will do; is the following statement true today? I submit is a failed hypothesis.
“Our climate model, driven mainly by increasing human-made greenhouse
gases and aerosols, among other forcings, calculates that Earth is now absorbing 0.85 +/- 0.15 watts per square meter more energy from the Sun than itis emitting to space. This imbalance is confirmed by precise measurements of increasing ocean heat content over the past 10 years. Implications include (i) theexpectation of additional global warming of about 0.6-C without further changeof atmospheric composition; (ii) the confirmation of the climate system’s lag in responding to forcings, implying the need for anticipatory actions to avoid any specified level of climate change; and (iii) the likelihood of acceleration ofice sheet disintegration and sea level rise.”
It was already known in 2002 the “hot spot” theory ist kaput. Since then the effort has been to discredit satellite/radiosonde data using very creative methods, with Santer 2008 being the most recent sloppy science promoted as “robust”.
http://www.co2science.org/articles/V5/N14/EDIT.php
As jae eludes to, only an unrealistic and unphysical runaway strong positive water vapor feedback can save the CO2 AGW fairytale, and it doesn’t exist. Of course, we must ignore the discrepancies in GCM’s for water vapor:
http://www.cdc.noaa.gov/people/dezheng.sun/dspapers/Sun-Yu-Zhang-JC-revised.pdf
In conclusion, do you have empirical evidence to support CO2 AGW, or will we be treated to ever increasing long winded speeches?

par5
February 27, 2009 11:02 pm

The earth is capable of storing more energy than it emits- and is just as capable of emitting more energy than it stores. This is not a violation of thermodynamics. Cloud coverage changes daily, weekly, monthly, seasonally, yearly, locally and globally. The first image in your primer suggests cloud cover as a constant, but for what time period? If cloud cover changes yearly (and it does), then there is no balance of radiative energy from year to year. I would call that normal, but I would not call it ‘radiative heat balance’- THAT is a violation of thermodynamics. You brought of Homer Simsons’ quote, so that’s fair game. Lisa was violating the laws of GRAVITY, not thermodynamics. At least, that is why I laughed.

par5
February 27, 2009 11:26 pm

My bad. Someone told me that she was playing her sax on the ceiling- that would be gravity. Someone else told me that she built a perp motion machine- that WOULD be thermodynamics. Anybody see the episode? Constant rain here in Atlanta, shot an 88 with a bad rotator cuff. Enjoy your day…

Brendan H
February 28, 2009 2:34 am

Anthony: “The two preferred terms are “skeptic” and “alarmist”…”
Anthony, this seems to imply that “skeptic” and “alarmist” convey similar connotations. If so:
1. Would you say that “skeptic” is an honourable term?
2. Would you say that “alarmist” is an honourable term?

foinavon
February 28, 2009 2:36 am

Bill Illis (19:06:59)

They are assuming of course that CO2 is responsible for nearly the entire greenhouse effect. As CO2 creates an increase in atmospheric temperatures, the atmosphere is “capable” of holding more water vapour and the increase in temperatures does, in fact, result in a proportional increase in the water vapour content of the atmosphere.
The increased water vapour content then results in an amplified greenhouse effect. Since water vapour is the most potent greenhouse gas.
So when this post and others cites 33C as “the greenhouse effect”, it is important to note that the current estimates of 3.0C per doubling of CO2 assumes CO2 is responsible for nearly the entire greenhouse effect because water vapour will increase in lock-step with any increase in CO2.
If CO2 is Zero, then water vapour will effectively be Zero as well and we don’t want a chart shown to the general public which demonstrates this impossibility.

That could hardly be more incorrect Bill. The determination of climate sensitivity makes no assumption of the proportion of the greenhouse effect by CO2. For example, Steve Goddard has (incorrectly as it happens) referred to the analysis of Charnock and Shine (see graphs in the top post). If you read what Charnock and Shine actually report (a fairly standard, if earlyish, analysis), it’s clear that the complete removal of CO2 might be expected to reduce temperatures by around 1/3d to 1/2 of the total greenhouse effect warming:
“Corresponding estimates of the effect on the mean surface temperature of the Earth are much more complicated, as both Campbell and Tomkin say. But using a simple radiative convective model,1 with no other change, one finds that doubling the CO2 produces a 1.5 °C warming and removing it a 12 °C cooling. Including a simple relative humidity feedback (but no ice-albedo feedback) changes these values to 2.4 °C warming and 17 °C cooling”
Charnock H, Shine KP (1993) CO2s Greenhouse Contribution Debated Physics Today 4646, 66-66
The simple climate sensitivity analysis can’t be used at very low [CO2], if we’re interested in the real world. It simply doesn’t apply in that form at very low concentrations

The empirical evidence to date does not support this assertion of course.
Here is the chart Joel Shore doesn’t want you to see.

Those charts don’t make sense. Where are they from? Whoever made them has made the pretty fundamental error of taking a model for an equilibrium response and plotting this with data defining a transient response. It’s wrong, and is bound to give an incorrect interpretation of the climate sensitivity.
One needs to be careful with models; firstly to understand what they mean, secondly to understand their range of applicability and thirdly to use them properly.

foinavon
February 28, 2009 2:54 am

jae (08:41:45) :

foinavon: “Now we know that this interpretation was incorrect. We can measure the water content of the troposphere (upper and all) and determine that in the real world the upper troposphere is moistening (as models predict in fact)[*****]. So we reject Lindzen’s failed hypothesis in this particular respect, since it is shown not to be correct. There’s no problem with that. It’s part of science. However what we shouldn’t do is to retain analyses that we know to be incorrect because we happen to like their implications.”
jae: Oh, yeah?

…and then you refer me to someone’s blog. However the blogger doesn’t address the point at all. Lindzen was proposing something specific that turned out to be incorrect. That particular point is settled in standard scientific practice through testing against real world data.

schnurrp
February 28, 2009 4:56 am

If the hypothesis is “runaway AGW”, I would suggest “believers” and “skeptics” to describe the two camps.

Steven Goddard
February 28, 2009 5:37 am

Phil,
Why does your photobucket link show 750/cm instead of 650/cm? The wikipedia link I provided (which you described as a cartoon) also shows transmission at 750. Bu the band is centered at 650. I don’t see any discrepancy between the graphs – you just missed the center in yours.
You haven’t answered either of my questions yet.

jae
February 28, 2009 6:31 am

foinavon:
Sorry. I pointed you to Spencer’s blog, because I thought he had a post that addressed that issue. But I don’t see it now…

tallbloke
February 28, 2009 7:03 am

Joel Shore (13:55:42) :
jae says:
That little discussion has been used thousands of times. Unfortunately, it ignores details like convection.
No…As Stevo nicely explained it is convection that is important in controlling the vertical thermal gradient that exists in the troposphere. Hence, it is not in fact being ignored at all.

Good to hear. So what the proportions of outgoing heat from the surface parameterised by the models?
Nothing like these I bet…
Sorokhtin, 2001a: Greenhouse effect: Myth and reality. Vestnik Russian Academy of Natural Sciences 1:8-21..
“According to our estimates, convection accounts for 67%, water vapor condensation in troposphere accounts for 25%, and radiation accounts for about 8% of the total heat transfer from the Earth’s surface to troposphere.”

tallbloke
February 28, 2009 7:11 am

foinavon (02:36:30) :
One needs to be careful with models; firstly to understand what they mean, secondly to understand their range of applicability and thirdly to use them properly.
And fourthly not to make rash alarmist assertions based on their output because they have never successfully predicted anything.

February 28, 2009 7:16 am

Steven Goddard (19:22:28) :
Phil,
I don’t think you understood my question. How did the CO2 get excited at 1mb in the first place? What event caused it to get excited?

Absorption of bb radiation and by collision with other molecules (Boltzmann distribution).

Editor
February 28, 2009 8:04 am

Brendan H (02:34:26) :
Anthony: “The two preferred terms are “skeptic” and “alarmist”…”
Anthony, this seems to imply that “skeptic” and “alarmist” convey similar connotations. If so:
1. Would you say that “skeptic” is an honourable term?
2. Would you say that “alarmist” is an honourable term?

Not stepping on Anthony’s turf just giving my take on the terms:
The issue isn’t honor. It’s accuracy. Skeptics are, in fact, skeptical about the quality of the science and thinking behind AGW hypothesis. They do not “deny”, they question.
The AGW hypothesis supporters have unfortunately earned the term alarmist. Look at anything from Hansen and AlGore in the last couple of years. End of the world as we know it, etc. Oh, and death of Gaia from Lovelock. So it is an acceptable term, to me, since it correctly describes behaviour and is devoid of pejorative nature. There are hundreds of examples of folks saying they need to raise the alarm level. They wear it as a badge of honor in many cases… It is not particularly a pejorative, especially if the alarm is valid.
Now me, I’ve moved from skeptic to a more direct denial that AGW is real. I’ve seen enough of the science (and lack of it on the part of AGW advocates) that I’m comfortable denying the correctness of their claims. (I do not deny that warming has happened for 30 years. It has; but was preceded by cooling for 30 years, and to be followed by cooling for 30 years…)
Unfortunately, the use of “denier” by the alarmists has been in a way charged with religious hate speech overtones, so they have contaminated the word with confounding hate. So I understand the ban on ‘denier’ as a pejorative. Just as I’m happy to call myself an “old fart” but would not expect the term to be acceptable in general public as a pejorative applied by others.
So I’m happy to call myself an old fart denier, but would not think it proper for others to do so. Hate speech is often like that, asymmetrical in it’s use. And I’d be happy to be called an “Anti-AGW alarmist” raising the alarm about all the harm it will do to our economy. By anyone. I’d wear it as a badge of honour…

February 28, 2009 8:27 am

tallbloke (07:03:30) :
Good to hear. So what the proportions of outgoing heat from the surface parameterised by the models?
Nothing like these I bet…
Sorokhtin, 2001a: Greenhouse effect: Myth and reality. Vestnik Russian Academy of Natural Sciences 1:8-21..
“According to our estimates, convection accounts for 67%, water vapor condensation in troposphere accounts for 25%, and radiation accounts for about 8% of the total heat transfer from the Earth’s surface to troposphere.”

Since those figures are total rubbish I’m sure they’re not! There’s no way that radiation is as low as 8%.

Hank
February 28, 2009 8:44 am


schnurrp (04:56:58) :
If the hypothesis is “runaway AGW”, I would suggest “believers” and “skeptics” to describe the two camps.

Or how about the skeptics and the unskeptical? “Proponents” is the word i tend to use when being polite.
And another thing. Proponents can be deniers too. Proponents deny my hypothesis that things will most likely continue on quite equably just as they have for centuries with humans adapting quite well to most anything short of another ice age.

February 28, 2009 9:00 am

Steven Goddard (05:37:01) :
Phil,
Why does your photobucket link show 750/cm instead of 650/cm?

Because I happened to have that one available, it shows the effect on the wing rather than the Q-branch.
The wikipedia link I provided (which you described as a cartoon) also shows transmission at 750. Bu the band is centered at 650. I don’t see any discrepancy between the graphs – you just missed the center in yours.
It’s difficult show the whole band at full resolution, the difference is that the cartoon by not showing the individual lines and therefore appearing to show a complete wide band completely misses the impact of line-broadening. Which is where the extra absorption with increased concentration comes from.
Here’s a whole band at a lower resolution which shows the effect of doubling [CO2] and also the edge effect.
http://i302.photobucket.com/albums/nn107/Sprintstar400/CO2spectra.gif
You haven’t answered either of my questions yet.
I did this morning, I guess they’re slow posting?

Bill Illis
February 28, 2009 9:12 am

foinavon,
I will happily redo the log warming chart with the proper qualifiers when someone nails down what timeline should be used for the transient equilibrium response.
A little over a decade ago, it was only several years, then it changed to 30 years and now it seems much longer timelines are being discussed.
In Hansen’s newest writings, he has redefined it to take 1,500 years and apparently the NOAA, NCAR and Hadley Centre models agree with that (although it not clear whether they mean +3.0C in 1,500 years or +6.0C in 1,500 years).
If it is +6.0C in 1,500 years, then the log warming formulae will have to be revised of course. If it is just +3.0C in 1,500 years, then the log warming formulae should again be revised but this time the important time delay factor will have to be included.
I would like to be able to test the predictions against the empirical data to date to find out if we are on the right track or not. If the timelines are far too long to be tested, then we will have to go back to arguing about the theory again (albeit far into the future theory).
http://pubs.giss.nasa.gov/docs/2008/2008_Hansen_etal.pdf

Joel Shore
February 28, 2009 9:36 am

DR says:

Joel Shore,
I have read the Schmidt et al 2005 paper thoroughly, so drop the patronizing.

Well, if you read it so thoroughly, I don’t understand why you quoted one sentence from their introduction where they were basically setting up the background of what was understood before they embarked on their study. You would have given a much better overview of their conclusions if you had quoted their abstract.
As for the general question of tropical tropospheric amplification, I gave what I thought was a very fair summary of what is known and not known at the moment and I don’t have anything further to add to it.

As jae eludes to, only an unrealistic and unphysical runaway strong positive water vapor feedback can save the CO2 AGW fairytale, and it doesn’t exist. Of course, we must ignore the discrepancies in GCM’s for water vapor:
http://www.cdc.noaa.gov/people/dezheng.sun/dspapers/Sun-Yu-Zhang-JC-revised.pdf

Thanks for the link. I hadn’t seen that paper and it looks interesting…I’ll give it a read. Of course, there are also other papers (in addition to Dessler’s) that have looked into the water vapor feedback and reached the conclusion that it is being modeled quite well. See, for example, these two papers by Soden: http://www.sciencemag.org/cgi/content/abstract/sci;310/5749/841 and http://www.sciencemag.org/cgi/content/abstract/296/5568/727 [And, lest you try to claim that Soden is biased, I will note that he seems to call them as he sees them given that he is basically quite skeptical in regards to the effect of AGW on hurricanes as can be seen here http://www.sciencemag.org/cgi/content/summary/sci;322/5902/687 , which is one of the reasons why I emphasize that the science on this particular issue is still quite unsettled (although I was saying that even before this Soden paper appeared).]

In conclusion, do you have empirical evidence to support CO2 AGW, or will we be treated to ever increasing long winded speeches?

Well, to save you the long-windedness, I could just refer you to the IPCC report, as there is plenty of such evidence in there. If you want me to throw out just one thing off the top of my head: the cooling of the stratosphere for which the data (compared to the issue with the upper troposphere) has a much higher signal-to-noise ratio. And, unlike the tropospheric “hot spot” issue, the cooling of the stratosphere actually is a fingerprint of AGW that distinguishes it from other warming mechanisms such as an increase in solar irradiance.

Steven Goddard
February 28, 2009 11:59 am

Phil,
Ahh… earlier you were arguing that IR excited tropospheric CO2 immediately loses it’s energy through collisions with N2 and O2 molecules. But now you are saying that those processes also work in reverse.
Previously you said –
Leif, the bolded statement is only true high in the atmosphere, ~stratosphere, near the surface any energy is exchanged via collisions with neighboring molecules almost immediately (timescale less than a nanosecond).”
and then you said-
Sorry N2 and O2 are homonuclear diatomics and dont radiate.
You described it initially as a one-way phenomenon, when actually it is more of an equilibrium. The N2 and O2 can also pass energy back to CO2, which does radiate.

Steven Goddard
February 28, 2009 12:43 pm

Phil,
Thanks much for the new picture, which though more detailed, shows the same overall pattern as the Wikipedia 600-900 region.

Joel Shore
February 28, 2009 1:01 pm

Bill Illis says:

I will happily redo the log warming chart with the proper qualifiers when someone nails down what timeline should be used for the transient equilibrium response.
A little over a decade ago, it was only several years, then it changed to 30 years and now it seems much longer timelines are being discussed.

The question is not so much one of timeline. The response predicted seems to be highly non-exponential in time, much like the decay in atmospheric concentrations of CO2 is following the release of a slug of it into the atmosphere. Hence the differences in times is at least partly a matter of what sort of time you are talking about…time to get halfway to the equilibrium value, 75% of the way, 90% of the way.
Probably the more relevant way to treat this issue is not to ask about the time…which is not really the useful question anyway for comparing temperature trends between theory and observation. Rather, you would want to make a similar plot to the one that you showed but using an estimate of the transient climate response (TCR) rather than the equilibrium climate sensitivity (ECS).
Table 8.2 of the IPCC AR4 Working Group I report gives a chart of the transient climate response for different climate models and they range from 1.4 C to 2.6 C (although the 2.6 C is a bit of an outlier as the next highest is 2.2 C). The somewhat more conservative conclusion of the IPCC report itself on the TCR is (Section 9.6.2.3)

Choosing lower and upper limits that encompass the range of these results and deflating significance levels in order to account for structural uncertainty in the estimate leads to the conclusion that it is very unlikely that TCR is less than 1°C and very unlikely that TCR is greater than 3.5°C. Information based on the models discussed in Chapter 10 provides additional information that can help constrain TCR further (Section 10.5.4.5).

As a final note, the “transient climate response” technically depends on how fast one is increasing the forcing and the numbers quoted are apparently based on an increase in CO2 in the atmosphere at the rate of 1% / yr. I believe that the actual rate of increase in CO2 is closer to 0.5% / yr, so our current “experiment” lies between the extremes of the ECS and the TCR, but probably still significantly closer to the TCR value I think. (Maybe you can find some studies of how the TCR value obtained depends on rate but I don’t know of any.)
Having said all of this, I should also point out that because of the uncertainties in the aerosol forcing as well as the uncertainties in the level to which natural variability has contributed one way or the other to the temperature trend, it has generally been found that attempts to constrain the climate sensitivity based on the 20th century temperature trend are not very good…I.e., the data is compatible with quite a broad array of climate sensitivity given the uncertainties. Better constraints are obtained from other things (paleoclimate data, the Mt Pinatubo eruption) and, of course, the best are obtained by combining all of this together.

foinavon
February 28, 2009 2:09 pm

Bill Illis (09:12:30) :

foinavon,
I will happily redo the log warming chart with the proper qualifiers when someone nails down what timeline should be used for the transient equilibrium response.
A little over a decade ago, it was only several years, then it changed to 30 years and now it seems much longer timelines are being discussed.
In Hansen’s newest writings, he has redefined it to take 1,500 years and apparently the NOAA, NCAR and Hadley Centre models agree with that (although it not clear whether they mean +3.0C in 1,500 years or +6.0C in 1,500 years).

No, that’s incorrect. And it’s not a “transient equilibrium response”. There is an equilibrium response (that incorporates several time constants) which is the full response to forcing once all of the elements of the Earth system have come to equilibrium….and there is the transient response (which is the real time evolution of temperature in response to a change in forcing).
At no time was the Earth’s equiibrium temperature response to a change in forcing considered to be instantaneous. Yet that’s the implicit assumption in your plots. That’s clearly incorrect, and it’s pointless to make an analysis based on knowingly incorrect assumptions.
Hansen hasn’t redefined “it” to take 1500 years. Why not simply read Hansen’s papers if you’re going to comment on them? They’re all freely downloadable. Hansen assesses the fast feedback (direct greenhouse warming, water vapour and sea ice albedo feedbacks) to give a climate sensitivity near 3 oC with ocean thermal inertia, much in line with a large amount of other analyses. The time constants are of the order of several decades. They’re of interest to events occuring during the coming decades/century
This climate sensitivity corresponds (according to Hansen) to the situation where there are negligible changes in polar ice sheets and albedo effects from major vegetation changes. When the latter are included over long time scales (centennial to millenial timescale) the climate sensitivity becomes (according to Hansen) closer to 6 oC.
No one considers that the time constants are sufficently well defined that we can establish the short term climate sensitivity precisely by analysis of contemporary temperature evolution (e.g. last 100 years). The uncertainties in climate response times and aerosolic contributions means that the problem is under-determined. Thus most analyses of climate sensitivities make use of analyses involving paleotemperature evolution under conditions where the Earth might be expected to come near equilbrium with forcings, or analyse transient responses (to the solar cycle or vlocanic effects).
Despite all of this you’ve chosen to assume that every response element to enhanced forcing occurs instantaneously. You’ve plotted data corresponding to the transient response against a model corresponding to the equilibrium response. Whatever our uncertainties about the respective time constants, we certainly know that they’re not all zero!

Brendan H
February 28, 2009 3:17 pm

EM Smith: “The issue isn’t honor. It’s accuracy…They do not “deny”, they question.”
If the issue is simply accuracy, then ‘denier’ should be acceptable, since it is arguably even more accurate than ‘sceptic’ — many AGW sceptics are selective questioners, subjecting AGW claims to scrutiny while giving as free pass to sceptic claims.
“The AGW hypothesis supporters have unfortunately earned the term alarmist.”
Many AGW sceptics also paint alarming scenarios of mass misery and death from measures for mitigating AGW, so from the point of view of accuracy they could also earn the term ‘alarmist’. Some of these anti-AGW alarmists can also be accurately described as ‘paranoids’, with their fantasies of eco-terrorist plots to rule the world.
“And I’d be happy to be called an “Anti-AGW alarmist” raising the alarm about all the harm it will do to our economy. By anyone. I’d wear it as a badge of honour…”
So the issue isn’t just accuracy, but also honour? Do you regard ‘AGW alarmist’ as a badge of honour?
Reply: NO NO NO! The word “denier” is a specific attempt to draw skeptics with the same brush as holocaust deniers and for that reason and that reason alone is prohibited as a pejorative on this site. End of story. It’s Anthony’s blog. It’s his rules. ~ charles the moderator

Roger Knights
February 28, 2009 3:31 pm

If not for the Holocaust association, “denier” would be an acceptable term. It’s connotation would be of people “in denial” about the ill-effects of their short-sighted, ego-centric behavior. (Drunkenness, abusiveness, etc.) IMO, that is what 90% of the users of the term intend to convey. I wouldn’t mind the label.
Believer and Disbeliever would be a pair of matched terms, but they lack the necessary zing. And Believer carries the disparaging hint of True Believer.
What’s needed is a set of terms that are only mild “zingers.” (Two that I like for the Believers are Warmmongers and Hotheads.) Alarmist qualifies, but there needs to be a countervailing term for their side to use. (Cold fish? Snoozer? Scoffer? Slug-a-bed?) Is there some well-known character from film or fiction who ignored a warning with disastrous consequences? I bet there is, but I can’t call it to mind.

Roger Knights
February 28, 2009 3:33 pm

Oops: Second line, change to “Its connotation.”

tmtisfree
February 28, 2009 3:34 pm

Edward (06:39:24) :
tmtis free 00:39:45
You stated earlier: “As an analogy, if you use a blind over a window on a sunny day, adding more blinds will not make the room any darker”
I like your analogy, however I partially disagree and suggest an improvement . Think of the gases in the atmosphere as if the “shade” was cut into narrow strips. Pulling these strips down individually would block particular segments of the incoming light. The more strips you brought down the darker the room would become. Some strips would overlap while others would still allow light into the room through gaps between each other. To completely block light from coming into the room you would need to add multiple layers of shading to eliminate all of these “gaps”.

Sorry for the late response.
This is all good except that the number of strips is so high that adding some more will not allow the light to pass through any further because the light already does not pass through.
I suggest you to calculate (it is easy) the number of CO² molecules (only) present in a, say, 10km high square meter column of atmosphere. You can use then the simple Beer’s law to approximate the heigh of the atmosphere required all to absorb the LW radiated photons. Then you can reduce the estimated height by plotting the power absorbed by increased level of CO² (x2, x3, X4). Finally the calculation of this height by using the absorption cross section of CO² at 15µm (see HITRAN for that) will show you that the height is minimal. This means that there are many many strips (10km in fact) above the height at which LW radiations are absorbed by CO².
Bye,
TMTisFree

February 28, 2009 5:12 pm

Steven Goddard (11:59:22) :
Phil,
Ahh… earlier you were arguing that IR excited tropospheric CO2 immediately loses it’s energy through collisions with N2 and O2 molecules. But now you are saying that those processes also work in reverse.

Of course that’s the point.
Previously you said –
Leif, the bolded statement is only true high in the atmosphere, ~stratosphere, near the surface any energy is exchanged via collisions with neighboring molecules almost immediately (timescale less than a nanosecond).”
and then you said-
Sorry N2 and O2 are homonuclear diatomics and dont radiate.
You described it initially as a one-way phenomenon, when actually it is more of an equilibrium. The N2 and O2 can also pass energy back to CO2, which does radiate.

It’s one-way as far as the excited state is concerned. Yes over a large number of molecules it’s equilibrated but the lifetime of an individual excited state is very limited by those collisions, it doesn’t stay excited long enough to significantly radiate at low altitude. At high altitude the lifetime is longer because of reduced collision frequency and there is a bigger contribution from emission.

February 28, 2009 5:59 pm

tmtisfree (15:34:11) :
Edward (06:39:24) :
tmtis free 00:39:45
You stated earlier: “As an analogy, if you use a blind over a window on a sunny day, adding more blinds will not make the room any darker”
I like your analogy, however I partially disagree and suggest an improvement . Think of the gases in the atmosphere as if the “shade” was cut into narrow strips. Pulling these strips down individually would block particular segments of the incoming light. The more strips you brought down the darker the room would become. Some strips would overlap while others would still allow light into the room through gaps between each other. To completely block light from coming into the room you would need to add multiple layers of shading to eliminate all of these “gaps”.

A better analogy would be a set of strips hanging vertically which can be rotated to block more light. As they’re rotated it’s the equivalent of increasing the concentration of the absorber.

Hank
February 28, 2009 6:16 pm

Roger Knights –
Good post. Personally I don’t think of myself as denier, as you point out it links in to being in denial. It’s not like I’m not looking for answers. They just have to be believable answers. I get nowhere hearing that we know global warming is real because Jim Hansen say it’s real. Just call me a skeptic, a doubter and a disbeliever.

Brendan H
February 28, 2009 6:47 pm

“Reply: NO NO NO! The word “denier” is a specific attempt to draw skeptics with the same brush as holocaust deniers and for that reason and that reason alone is prohibited as a pejorative on this site. End of story. It’s Anthony’s blog. It’s his rules. ~ charles the moderator”
The issue here is EM Smith’s claim that he regards “alarmist” as an accurate description, so therefore it is “devoid of pejorative nature”. My contention was that if the only criterion is accuracy, “denier” is no more pejorative than “alarmist”.
However, it’s clear that the connotation of the term “denier” can be more pejorative than the connotation of the term “alarmist”, although a pejorative phrase such “in denial” has no connection to the holocaust as far as I can see. But that’s not the point at issue.
So my original question (2) stands: “Would you say that “alarmist” is an honourable term?”

Reply to  Brendan H
February 28, 2009 6:49 pm

Personally, I use the term AGW proponents, but given Anthony’s treatment by some of them, and some of their pronouncements, I understand his willingness to use the term alarmist. I think it’s too much of a harsh generalization myself.

Steven Goddard
February 28, 2009 8:17 pm

Phil,
Then it sounds like we are in agreement.
The more GHG in the atmosphere, the greater the number of collisions between photons/molecules and molecules/molecules, and the higher the temperature required to maintain equilibrium.

peter_ga
March 1, 2009 12:56 am

Steve,
I had the impression that the CO2 infra-red absorption bands were saturated at sea-level; rather the variation in CO2 caused changes in the amount of energy transmitted from the upper troposphere to outer space. Does this not mean that any increases in surface temperature would theoretically be caused by higher surface temperatures being necessary to force convective heat transfer, rather than direct radiative effects at the surface?
If so, would not the role of extra water vapor be to induce negative feedback, by decreasing lapse rates, rather than further plugging already saturated energy bands?

PLS
March 1, 2009 1:44 am

I don’t understand now CO2 in the atmosphere can change the temperature of the planet. Please help out a novice.
I recall a thought experiment from college physics. Takt an insulated box with two spheres inside and everything in thermal and radiative equilibrium. Make it a big box so the spheres have gravity and atomspheres. Both spheres start at the same temperature.
Now change the color of one sphere. Maybe add CO2 to the atmosphere, maybe add water vapor. Maybe paint it pink.
The assertion is that the change in color will affect both radiation absorption and emission in the same way so that the temperature of the two spheres will remain the same. If the don’t then you can hook a heat engine between the two spheres and extact heat from a system in thermal equilibrium in fine disregard for the second law of thermodynamics. You can also make a perpetual motion machine.
So in what sense can CO2 in the atmosphere change the temperature of a planet? Or has the second law been falsified while I wan’t looking?
++PLS

JamesG
March 1, 2009 2:49 am

I seem to recall the IPCC reports saying that the stratospheric cooling was largely from ozone depletion and then later that the true AGW signature was the combined tropospheric warming and stratospheric cooling. Nevertheless can someone tell me if it’s possible to get a feedback factor from just from the the non-ozone stratospheric cooling? If that is meant to be the gold standard signature of GHG warming then why don’t we treat it as such and ignore all the other guesstimations for sensitivity. I’m perplexed though because I’d have thought that if the strong postulated water vapour feedback wasn’t in the tropical troposphere then it wouldn’t be anywhere. Does water vapour feedback extend to the stratosphere?
As far as I can bear to read Hansens papers, he seems to get his main feedback calculations from Vostok – as if what happened there could be applied across the entire planet. His gross assumptions for those sensitivity calcs have been mocked even by AGW stalwarts like Annan and Connelley.
On alarmist, can we agree that the scientists and pundits who go beyond the findings of the IPCC reports are by definition alarmist? There are lots and lots of those types. One enviro journalist (M. Lynas I believe) actually replied once on R. Pielke Jr’s blog and confirmed he was an alarmist and proud of it. Schneider and Gore have said much the same thing in interviews, though in different words. But if alarmist offends then what’s the best way to describe someone who believes the gross media distortions that routinely go way beyond the IPCC reports and all other scientific consensus statements?

lgl
March 1, 2009 4:21 am

PLS,
Forget about second law. This is about radiation, not heat.
View CO2 as a semitransparent mirror, more CO2 give more reflection.
JamesG,
Ozone depletion is cooling the lower stratosphere, more CO2 is cooling higher up. I would also like to know whether more H2O in the stratosphere will give extra cooling and whether this has any impact on surface temperature.

peter_ga
March 1, 2009 5:05 am

pls,
You are neglecting the sun, which has a temperature of 10000 degrees K at its surface, sends high energy photons to the surface of the earth, which has to heat up at its surface to about 290K to export its energy so that it has an apparent temperature of 250 degrees K at mid-upper troposphere. So I think all the laws of thermodynamics are being obeyed.
So if one of the balls in your experiment is 10000K, I think you will find the other ball will probably be quite warm.

gary gulrud
March 1, 2009 5:12 am

“View CO2 as a semitransparent mirror, more CO2 give more reflection.”
Idiocy.

gary gulrud
March 1, 2009 5:20 am

“the lifetime of an individual excited state is very limited by those collisions, it doesn’t stay excited long enough to significantly radiate at low altitude.”
And this has been given congruent explanations in complete agreement above by George Smith and myself.

March 1, 2009 5:37 am

Brendan H.:

My contention was that if the only criterion is accuracy, “denier” is no more pejorative than “alarmist”.

Can’t agree with that, my friend.
As stated here repeatedly, the term ‘denier’ has an unsavory connotation with holocaust denier. That is exactly why it is used.
The term ‘alarmist’, on the other hand, is absolutely accurate. Here’s why:
Alarmism is the only reason that $Billions [so far] have been shoveled into global warming research and grants. More $Billions are in the pipeline. Why? Because AGW proponents use unsavory scare tactics to extract money from taxpayers.
If the truth were being told based on the Scientific Method, AGW proponents would be forced to acknowledge that the only basis for their scary scenarios comes from always-inaccurate computer models.
Solid, empirical [real world] evidence showing that CO2 causes global warming is non-existent. Therefore, if the truth were being told by the proponents of the AGW/CO2-global warming-tipping point hypothesis, they would be forced to admit that they are only putting forth a conjecture, based on speculation, which would normally merit only a footnote or two in a few minor scientific journals.
But minor journal footnotes do not result in $Billions in grant money to study a non-frightening problem. Thus, those working the system resort to alarmism over a harmless, beneficial rise in a very minor but essential trace gas.
Alarmism over CO2 showers the alarmists with money and status. Without alarming the public, the status would be missing and the money would go to much more deserving scientific research [which is currently being starved of funds because global warming alarmism sucks up most of the available money].
Scientists and journal editors are as human as the rest of the population, and they are just as susceptible to temptation. So some of them resort to alarmism to get what they crave.
The term “alarmist”, when applied to those promoting the AGW/CO2 climate catastrophe hypothesis, is entirely accurate. They are deliberately alarming the public, akin to crying “Wolf!!” when there is no wolf. They have no shame. But the concept of shame in the new millennium is as old fashioned as the concept of honor. Now, money and status trumps all, and alarmism is just another tool used by alarmists to achieve their desired ends.

pochas
March 1, 2009 7:28 am

PLS:
When the two spheres in the box are at the same temperature, then they are in thermal equilibrium and Kirchoff’s law applies, meaning that they will remain at the same temperature regardless of differences in color.
However, if one of the spheres has an internal energy source (such as the sun) then Kirchoff’s law no longer applies and the temperature of the spheres will depend on color. Adding the first whiff of CO2 to a transparent atmosphere changes its color, resulting in warming, although we can’t see the color change because it is in the infrared wavelengths. Adding more and more CO2 is akin to adding more and more coats of paint to your house. At some point the next coat of paint doesn’t matter.

Joel Shore
March 1, 2009 10:10 am

PLS: Just to add to what others have said, your thought experiment is in fact used to prove that the emissivity and absorptivity of a body must be equal to each other AT EACH WAVELENGTH. (Because, as you say, if this were not the case then if you have two bodies at the same temperature but different emissivity / absorptivity characteristics at some wavelengths, one would warm the other.)
However, with the sun – earth system, you have two bodies at very different temperatures. So, the radiation that the earth receives from the sun is primarily in the visible (and near-ultraviolet and near-infrared) whereas the radiation it emits is well into the infrared. By changing the emissivity and absorption in the infrared but not the visible, you can therefore raise the temperature.

Joel Shore
March 1, 2009 10:22 am

JamesG says:

Nevertheless can someone tell me if it’s possible to get a feedback factor from just from the the non-ozone stratospheric cooling? If that is meant to be the gold standard signature of GHG warming then why don’t we treat it as such and ignore all the other guesstimations for sensitivity.

I think you are mixing a couple things together. I think the argument has been that the statospheric cooling while the troposphere warms is a signature of GHGs being the cause. However, I don’t think the argument has been that you can determine the feedbacks from the stratospheric cooling. Feedbacks are estimated in different ways.

As far as I can bear to read Hansens papers, he seems to get his main feedback calculations from Vostok – as if what happened there could be applied across the entire planet.

That is not so. In estimating the equilibrium climate sensitivity (ECS) Hansen uses an estimate for the global temperature rise that is about 1/2 the rise seen at Vostok and I think most people agree that this is a pretty good estimate.

His gross assumptions for those sensitivity calcs have been mocked even by AGW stalwarts like Annan and Connelley.

I don’t think that Annan and Connelley have mocked the 3 C part of the ECS calculation, what Hansen calls the “Charney sensitivity”. I think that they have strongly doubted that there is enough ice left to be melted to raise the long-term sensitivity to 6 C. (They may also be somewhat skeptical of Hansen’s claims that what ice disintegration does occur is really likely to occur as fast as Hansen thinks it might.)

On alarmist, can we agree that the scientists and pundits who go beyond the findings of the IPCC reports are by definition alarmist?

Well, in that case, we would need to have a similarly-pejorative term to describe those who make predictions that disagree with the IPCC findings in the other direction too. There is a finite probably that the IPCC might be wrong on various things…but that probability extends in both directions.

JamesG
March 1, 2009 1:41 pm

Joel
“However, I don’t think the argument has been that you can determine the feedbacks from the stratospheric cooling.”
I know that’s not the argument, it was something I thought about on the spur of the moment and thought I’d ask. And I meant total sensitivity including all feedbacks. It just seems to me if that is the one unique attributable effect to an increase in GHG’s then we should try to use it to pin down the actual effect of the GHG increase and reduce the guesswork. Can we not determine anything from the stratospheric cooling numbers? eg. Joules retained in the system.
“we would need to have a similarly-pejorative term to describe those who make predictions that disagree with the IPCC findings in the other direction too”
You could use contrarian, since that seems to be the new preference at RC, perhaps used pejoratively. And while it is probably pejorative to say alarmist, it’s merited by a lot of people wouldn’t you say?
“Alarmist – a person who tends to raise alarms, esp. without sufficient reason, as by exaggerating dangers or prophesying calamities”.
Clearly that is a very good description of many of the actors in this drama, certainly most of the journalists and politicians, and a great deal of the scientists who court the press. I’ll be fair though and say I believe that the contrarians are often ridiculously alarmist about the costs involved in moving away from dirty to clean fuels. By dirty I mean of course all the other gunk that comes out of the tailpipe and chimneys, not the CO2 🙂

AlexB
March 1, 2009 2:01 pm

Steve Goddard,
To suggest that because convective heat transfer is a feedback because it only comes into play after absorption would be the same as suggesting that emission is a feedback. It is a primary heat transfer mechanism just like radiative emission and should be included in all clear sky simulations as it is the dominant effect.

JamesG
March 1, 2009 2:24 pm

Alex B
All emission except the Sun’s are indeed feedbacks. The sun is the only driver (excluding seismic effects, the odd meteor strike and cosmic rays). Even carbon-based fuels were created by the sun. Forcings are just artificialities to make climate models a lot simpler. It’s also routine in heat transfer models to have convective heat transfer as a feedback (or a secondary effect) from a radiative source eg a reactor – even when it dominates the heat transfer mechanism.

jae
March 1, 2009 8:52 pm
Brendan H
March 1, 2009 11:00 pm

Smokey: “As stated here repeatedly, the term ‘denier’ has an unsavory connotation with holocaust denier.”
And “alarmist” has unsavoury connotations of scientists lying in the pursuit of money and power.
REPLY: Better check the definitions there Brendan, because in references, there is no place where “alarmist” has “unsavoury connotations of scientists lying in the pursuit of money and power”.
from http://dictionary.reference.com/browse/alarmist
a⋅larm⋅ist
   /əˈlɑrmɪst/ Show Spelled Pronunciation [uh-lahr-mist] Show IPA
–noun
1. a person who tends to raise alarms, esp. without sufficient reason, as by exaggerating dangers or prophesying calamities.
–adjective
2. of or like an alarmist.
Origin:
1795–1805; alarm + -ist
see also http://en.wikipedia.org/wiki/Alarmism
Now let’s look at “denier”
http://en.wikipedia.org/wiki/Denier
one of the Wiki items is: Denial (state of), a person who denies something e.g.”He is a holocaust denier.”
See also: http://dictionary.reference.com/browse/denier “A person who denies.”
You lose on these points. Don’t bring up the issue again, because I have a low tolerance for discussion of it. – Anthony

Law of Nature
March 1, 2009 11:41 pm

Hi all,
and thanks to Steve for addressing another interesting topic.
However I want to ask about something which seems very interesting in the discussion, but completly ignored/downplayed in the original article.
The question, what exactly happens if the CO2-concentration is doubled (lets just say from 300 to 600ppm):
Apparently not much in the lower atmosphere as there is already a significant amount of CO2 there and the IR-radiation decays over a few 10m or so.
A doubling does affect the side bands but only changes little and the radiation at low heigths is then transferred into kinetic energy, which basically means once it is re-radiated most of it will be in a spectrum where water dominates the CO2.
So, I think the problem might be something like: At what height a doubling of CO2 would increase the absorbed energy (in a certain band) by 10%?
(or similar)
And the related question (from what I read here I guess this height is nowhere near the earth surface):
How much does this change affects the tempeature down here?
(And is this effect significant compared to others . . like clouds and so on?)
All the best
LoN

Gary
March 2, 2009 1:24 am

Maybe it’s the engineer in me but I do get irritated when I see the statement that the greenhouse effect is equivalent to 33C. It is not. When comparing current conditions to no atmosphere, you cannot separate the GHG and other atmos effects such as reflection, absorption etc. Because of these effects solar insolation at the surface averages about 232 w/m2. With no atmosphere the solar insolation rises to 315 w/m2. Without showing all the calcs this reduces the GHG effect from 33 C to 12 C. It is only 33C if the solar insolation is held constant. This is not physical. Whenever the atmosphere composition is changed you cannot hold insolation constant. Rising water content will reduce solar insolation due to not only reflection but also absorption. this is the negative feedback mechanism of water, the only GHG that partly absorbs solar radiation. It doesn’t take much increase in reflection/absorption to negate the CO2 effect.

PLS
March 2, 2009 3:12 am

Wrong two bodies.
The thought experiment envisions two bodies in a radiation field. Think of them as Earth with two different amounts of CO2 in the atmosphere. The argument is that adding CO2 cannot change the temperature of the body.

Joel Shore
March 2, 2009 8:29 am

PLS says:

Wrong two bodies.
The thought experiment envisions two bodies in a radiation field. Think of them as Earth with two different amounts of CO2 in the atmosphere. The argument is that adding CO2 cannot change the temperature of the body.

I don’t know who you were even responding to here with your first sentence but multiple responders including myself have explained to you in quite a bit of detail and from different angles why your argument is absolutely wrong.
Just to clarify what is wrong with this current incarnation: If you by “in the radiation field”, you mean a radiation field other than that produced by these two bodies themselves, then you do not have a closed system and the 2nd Law cannot be applied to it.
And, of course, anyone who has ever tried walking barefoot across an asphalt surface with some white paint on it (or across asphalt and cement surfaces) would immediately be able to tell you that you are wrong.
And, finally, if you mistakenly apply the 2nd Law to systems that are not closed, you can prove all sorts of things such as the notion that a refrigerator is a violation of the 2nd Law!

Joel Shore
March 2, 2009 8:39 am

Gary says:

Maybe it’s the engineer in me but I do get irritated when I see the statement that the greenhouse effect is equivalent to 33C. It is not.

I understand your point…but I think how this is best stated is that the presence of our atmosphere causes multiple effects, the “greenhouse effect” which causes warming and other effects (reflection or absorption of some of the incoming solar radiation) that cause cooling. So, I don’t think it is incorrect to say that the greenhouse effect itself causes ~33C of warming. But, yes, it is important to also understand that the presence of water vapor…and especially condensed water vapor (clouds)…in the atmosphere cause reflection and absorption of some of the incoming solar radiation.

gary gulrud
March 2, 2009 12:42 pm

“multiple responders including myself have explained to you in quite a bit of detail and from different angles why your argument is absolutely wrong.”
Turn about is fair play, Joel. Persisting in your metaphysics and never admiting to material error does not engender responsible argument from your opponent.

Joel Shore
March 2, 2009 2:39 pm

gary gulrud says:

Turn about is fair play, Joel. Persisting in your metaphysics and never admiting to material error does not engender responsible argument from your opponent.

Care to cite an example of exactly what it is that you are talking about?

George E. Smith
March 2, 2009 3:22 pm

Lottta water going under the bridge here and some of it seems to be getting quite muddy.
I read most of what has transpired since I left my last note; and I have some differences of opinion with some of it, but there’s too many posts to identify individually.
Somewhere I believe it was Joel Shore who talked about ordinary atmosphere gases transferring theremal energy back to GHGs in collisions; and he implied that the atmosphere gases like N2 and O2 can’t radiate; being symmetrical diatomic molecules.
That’s not true. Thermal radiation following a somewhat black body spectrum, is emitted from all real physical bodies simply as a function of their temperature. The mechanism of that radiation is regarded in Classical Physics as deriving from the acceleration of elctric charges, which all physics substances contain.
You may recall that the early nuclear atom, was thought to radiate energy at a frequency corresponding to the orbital rotation rate of the eouter electrons which somehow were in orbit about the nucleus. This idea quickly fell on the scrap heap, as a result of the realization that an orbiting charge is in constant acceleration, and therefore must be constantly radiating EM waves; so it muct be continuously losing energy.
For you non-physics folks out there; quantities like velocity and momentum, are “vector” quantities; whiich means they have both “speed” and “direction”, and change of direction is an acceleration just as vhange of speed is. The continuous energy loss of accelerated charges, is the reason for the existence of the Stanford 2-mile Linear Accelerator. Electrons lose too much energy going around race tracks in circular type accelerators like Bevatrons and the like; hence use a straight line instead to reduce the energy losses.
So along came the Bohr atom with its stable non-radiating electron orbitals; which became probability clouds of where the electron might be found at any time; and radiation only occurrred during certain transitions from one permissable electron level to another. The resulting wavelength and energy emitted was the difference betweeen the energies of the two levels.
But that gives us line spectra with emission only at certain avelengths, and led to the science of atomic spectroscopy.
A diatomic molecule like N2 or O2 has six degrees of freedom; three translations, three rotations and one vibration, and it is usually only the vibrational mode of diatomic molecules which is non absorbtive. Threst do result in charges accelerating, leading to a radiated continuum of thermal radiation which depends only on the temperature (E=kT) where (k) is Boltzmann’s constant. It is interesting that Boltzmann derived the black body radiation law (Stefan -Boltzmann law) R = sT^4, where s (sigma) is the Stefan Boltzmann constant.
This was before Max Plack derived the exact spectrum of black body radiation, and launched the quantum era.
So the ordinary atmospheric gases DO (sorry about the caps foinavon) radiate thermal radiation solely as a function of their temperature, and since they radiate a black body spectrum, and not a line spectrum, only a portion of that radiation is in the absorption band of say CO2 or other GHG such as water.
So the downward radiation from the warm atmosphere (as well as upward) is a black body spectrum (roughly) and not a re-emission of the CO2 15 micron resonance line which is a vibration molecular mode of an assymmetrical kind (bending). Because the temperatures are not uniform throughout, then there are many overlapping BB spectra at slightly different peak wavelengths which is one reason why the whole thermal spectrum of earth is not a simple BB spectrum. But remember that the Stefan-Boltzmann T^4 relationship still holds.
I don’t think I agree with Steve Goddard that we would get quite cold without CO2 in the atmosphere. Maybe later on I will describe a desert island sand and stick experiment to show why I believe that isn’t so. Hell ! buy a Mai-tai, and we’ll use the little umbrella to scratch in the sand.
George

pochas
March 2, 2009 7:14 pm

George Smith 15:22:16
“So the ordinary atmospheric gases DO (sorry about the caps foinavon) radiate thermal radiation solely as a function of their temperature, and since they radiate a black body spectrum, and not a line spectrum, only a portion of that radiation is in the absorption band of say CO2 or other GHG such as water.”
George, thank you for the refresher. However, gases don’t radiate a blackbody spectrum. Their spectrum is composed of thousands of discrete lines, each with its own emissivity. Here is a low resolution absorption spectrum for the atmospheric gasses. The upper panel indicates the blackbody spectrum at the indicated temperatures.
Solids and liquids may often be approximated as blackbodies, but obviously anything that has visual color cannot be a perfect blackbody.
http://upload.wikimedia.org/wikipedia/commons/7/7c/Atmospheric_Transmission.png

Joel Shore
March 2, 2009 7:50 pm

George E. Smith said:

Somewhere I believe it was Joel Shore who talked about ordinary atmosphere gases transferring theremal energy back to GHGs in collisions; and he implied that the atmosphere gases like N2 and O2 can’t radiate; being symmetrical diatomic molecules.

First, I think you may have mixed me up with someone else as I didn’t talk about the transferring of thermal energy in collisions as far as I can recall.
However, in talking about why small amounts of trace gases such as CO2 play a disproportionately-large role in our climate, I did say the N2 and O2 don’t absorb infrared radiation…and, barring definitive evidence to the contrary, I stand by that. For example in Fundamentals of Fourier Transform Infrared Spectroscopy by Brian C. Smith ( http://books.google.com/books?id=cL_cvNCoEmkC&pg=PA11 ), it says

Another class of substances that do not absorb infrared radiation are homonuclear diatomic molecules. These are molecules comprised of two identical atoms, such as N2 and O2. Homonuclear diatomic molecules do not possess infrared spectra due to their symmetry. N2 and O2 are very abundant in the atmosphere, and if they absorbed infrared radiation all the radiation in the spectrometer would be absorbed by the atmosphere before it got to the sample.

hotrod
March 2, 2009 8:20 pm

I did say the N2 and O2 don’t absorb infrared radiation…and, barring definitive evidence to the contrary, I stand by that.

It is my understanding that the radiate in the microwave not the IR spectrum. If that is true than it is accurate to say that they do re-radiate energy and that they do not radiate IR.
Perhaps that is the important point?
I do not recall where I stumbled across that reference so cannot give a link to a source.
Larry

Westhighlander
March 2, 2009 8:59 pm

Folks — Sorry — good effort — but as the huge number of posts in reply show — there can’t really be a useful primmer on the Green House Effect
This is not surprising as once one gets a Ph.D. hood for work in physics the Prof. whispers to you the “Secret of / to the Universe” — Unfortunately I can not divulge this to the uninitiated (under the pain that the Universe will just collapse back into the …… and disappear) and as for the rest of those of us who have been initiated — well we already know it.
What I can do however — is drop a few oblique hints that may be illuminating (pardon the pun?)
1) Let’s start with an everyday experience — Look at your self in a mirror — no physicist can explain to anyone else how you can see yourself — or any coherent image for that matter (except by reverting to classical Electromagnetism)
2) In point of fact while QED (Quantum Electrodynamics) is considered to be the most validated theory that we have ever constructed — no closed form Quantum equation can explain in detail the energy levels of any atom except hydrogen (even H requires some fudging let alone what happens when you shine a photon on a H2 molecule) — even Helium (monatomic gas) is a problem — let alone a triatomic molecule such as CO2 or H2O
3) We do even worse when we have to consider more than one molecule or atom of the same species in a very rarefied gas (with only elastic collisions) let alone when we mix gases
4) Now let the atoms have some inelastic collisions — things rapidly become a hand waving exercise — left to the reader
5) Now if we let the gas molecules have some center of mass motion (temperature) when absorbing and reradiating then the Doppler effect starts to change the effective energies of the transitions and of course the individual molecules are also exchanging CM energy in their collisions
6) Skipping ahead a couple of graduate courses — Some of the absorption and reradiation transitions can be “forbidden” in the case of an isolated atom or molecule due to the “wrong quantum numbers” — however, when you let atoms collide and the other “funny stuff” — the transitions just might become allowed
7) some of the atoms or molecules can carry some of the excitation energy (due to photon absorption) with them when they move a considerable distance (particularly at low densities where because of the low probability of collisions (long mean free path) and the low probability of the transition (long effective lifetime) of the state (so-called metastable) — now they can appear somewhere else a long way from where they absorbed and already ready to radiate
8) there is a lot more and we are just trying to explain how a monochromatic beam of photons can pass through an isolated column of gas
9) Note nothing has been said about oceans, ice caps, clouds, aerosols, cosmic rays, solar magnetic fields, breaking waves, or any of the other things which make this an interesting topic
PS: — everything we treat in physics is considered a “closed systems” — in reality — perhaps the Universe is a “closed System” — But nothing else is

George E. Smith
March 3, 2009 8:56 am

“”” pochas (19:14:23) :
George Smith 15:22:16
“So the ordinary atmospheric gases DO (sorry about the caps foinavon) radiate thermal radiation solely as a function of their temperature, and since they radiate a black body spectrum, and not a line spectrum, only a portion of that radiation is in the absorption band of say CO2 or other GHG such as water.”
George, thank you for the refresher. However, gases don’t radiate a blackbody spectrum. Their spectrum is composed of thousands of discrete lines, each with its own emissivity. Here is a low resolution absorption spectrum for the atmospheric gasses. The upper panel indicates the blackbody spectrum at the indicated temperatures.
Solids and liquids may often be approximated as blackbodies, but obviously anything that has visual color cannot be a perfect blackbody. “””
OK so perhaps I should have said “continuous spectrum” rather than “Black Body”. Yes I know about black grey, red, and blue bodies; and that no real body is a black body but some come close.
I believe I used the term thermal radiation.
The Wiki reference you gave (very nice plot thanks) is all about absorption spectra. It doesn’t say a thing about thermal radiation spectra.
The word “Thermal” generally implies thew concept of “Temperature”; and I believe I said that the thermal radiation was solely a function of temperature.
Now all of the ABSORPTION spectra in your Wiki post are either atomic or molecular (mostly) absorption spectra; they are a unique p[roperty of an individual molecule, or an individual atom.
No individual atom or molecule, has any idea what you are talking about when you mention the word TEMPERATURE; it is not a property of individual molecules or atoms.
Now I agree with Joel, that N2 and O2 don’t absorb in the long wave infrared. sorry for the mixed attribution there Joel.
Those absorption spectra are line spectra, and the intrinsic line specra are broadened by effects such as Doppler shifts, which are a result of the kinetic energy of the molecules themselves in a gas; which is a function of temperature (E=kT). The intermolecular collisions which are a function of total molecular density (pressure) also result in line broadening. The shorter the time interval between molecular collisions, the bigger is the energy shift. (Heisenberg ? maybe dE.dt > h/2pi )
So I don’t disagree that the absorption spectra of those molecular or atomic species in the atmosphere are line spectra which have to be explained in quantum mechanics, as westhighlander implies (or one might infer from his comments.
But I was not talking about those absorption spectra (other than that they exist); I was talking about the thermal emission spectrum of those gases; which depends roughly on the 4th power of temperature (yes I agree no real body is a true black body); but the BB spectrum bounds their possible thermal spectrum at the same temperature.
If we want to resort to quantum chromodynamics to explain global warming; we would need more space, and formatting and font capability than Anthony’s fireside here.
Then we might as well use Einstein’s general relativity, rather than Newton’s gravity or Keppler’s laws of orbital mechanics to explain the seasonal changes in climate.
We don’t need a 500# bomb to swat a fly. But Westhighlander’s comments do remind us that we can get a long way removed from “climate sensitivity”, or “forcings”, or any of the other trappings (like “anomalies”) that pass for science in the field of “Climatology”.
I believe that the Japanese recently described that as “ancient astrology”; and that might be unkind to the ancient astrologers.
But I shall have my own bookburning, and do some warming myself by setting fire to “The Infrared Handbook” of Wolfe and Zissis, and “Elements of Infrared Technology” by Kruse, McGlauchlin and McQuistan, along with all my optics and quantum electronics texts; and learn to rely on Google and Wikipedia for the gospel truths. And I need a new Handbook of Chemistry and Physics that contains all those new sciences like forcings.

lgl
March 3, 2009 9:40 am

George,
I’m afraid you will soon burn Google too. It will find this for you:
http://netfam.fmi.fi/PBL07/Savijarvi_Radiation-PBL.pdf
showing a LW spectrum not looking like a BB spectrum at all.

pochas
March 3, 2009 9:43 am

George E. Smith (08:56:56)
“But I was not talking about those absorption spectra (other than that they exist); I was talking about the thermal emission spectrum of those gases;…”
The two types of spectra are identical, as far as the frequencies of absorption/emission. I don’t want anybody to get the idea that gases absorb at discrete frequencies and emit as blackbodies. They don’t.

Earl E Winter
March 3, 2009 9:47 am

Referring to the first illustration:
Does this description represent the balance over just the land mass, or an average over the land mass and the oceans combined? If just the land mass, then it seems it would be very different over the oceans, and if this is just the land mass then can you post something explaining the balance over the ocean?
Your humble gardner.

foinavon
March 3, 2009 11:52 am

jae (20:52:21) :

Ahh, foinavon, here it is:
(a link to a blog page)
What say you?

Nope, jae, that doesn’t address the point.
Lindzen made a very specific prediction. At a time when tropsopheric moisture content variations were not easily assessed, he predicted that moist (cumulus) convection, which is expected to increase with atmospheric warming under enhanced CO2 concentrations, would result in a drying of the upper troposphere. This proposed drying would be a negative feedback that would partly offset the positive feedback from enhanced water vapour at lower altitudes.
When it became possible to determine tropospheric moisture content relaibly, and assess the response to greenhouse-induced warming, it became cleat that Lindzen’s hypothesis was incorrect. The upper troposphere desn’t “dry” in in response to tropospheric warming….in fact it “moistens” (much as models predicted, as it happens).
Your blogger doesn’t address this point at all. He speculates (rather discursively) about all sorts of other things. But these don’t relate to the Lindzen hypothesis which was sufficiently specific that it could be assessed categorically by real world measurements.

March 3, 2009 12:03 pm

The big picture is: global temperature and the amount of atmospherically CO2 through the centuries is cyclical. Sometimes somewhat capriciously but still unmistakable. Cyclic patterns in nature are always caused by a source with cyclical characteristics (e.g. global temperature by the sun cycles) and/or third order negative feedbacks.
Without negative feedback some values should grow continuously. So, if there has been a proven positive relationship between rising CO2 levels and rising temperature in the past, rising CO2 levels should have caused the increase of temperature “for ever”.
Nor temperature, nor CO2 levels have ever shown such characteristics.
And because nature cannot distinguish between “natural” CO2 and “manmade” CO2 (this dichotomization is of course rubbish), there will be always some negative feedbacks in place. So even the anthropogenic greenhouse gas levels and the global temperature will always decrease at a certain point as we actually have seen in nature. There for the IPCC 4th Evaluation Report’s conclusion that from now on atmospheric temperatures are likely to continuously increase, should be perceived as a falsified hypothesis.

George E. Smith
March 3, 2009 12:03 pm

“”” pochas (09:43:58) :
George E. Smith (08:56:56)
“But I was not talking about those absorption spectra (other than that they exist); I was talking about the thermal emission spectrum of those gases;…”
The two types of spectra are identical, as far as the frequencies of absorption/emission. I don’t want anybody to get the idea that gases absorb at discrete frequencies and emit as blackbodies. They don’t. “””
Well I think I’ve restated enough that nothing emits like a black body.
The paper cited by Igl above contains a whole lot of meteorological stuff that is outside of my knowledge base; so I’m not able to comment intelligently on that. I notice he does mention “continuum radiation” from water; as well as line spectra.
But for the moment I will just accept that gases don’t emit any thermal radiation; despite the fact that every text book I’ve ever seen particularly on infra-red technology have all stated that every single thing in the universe emits thermal radiation due to its temperature; and yes I accept that nothing in the universe emits black body radiation.
I would like to know just what that water “continuum radiation” is though; and what causes it.
By the way; Kirchoffs law only relates to a body that is in thermal equilibrium with its surroundings at a given temperature; well I suppose my textbooks are wrong on that too.
I wonder why the sun seems to emit a “continuum ” radiation, even though it isn’t solid or liquid; well I suppose plasmas do contain accelerating electric charges.
And for the record I’ll repeat; no not all thermal radiation spectra are blackbody spectra; but they are thermodynamically limited to radiate less than a BB (at any wavelength); well at least that’s what my textbooks say.

lgl
March 3, 2009 12:50 pm

George,
From Pierrehumbert’s ClimateVol1.pdf:
“A Nitrogen molecule N2 in isolation does not interact to any significant extent with infrared light; one might think that collisions do not change this picture, as N2 has no lines to be broadened by collisions. Nonetheless, during the time a collision is taking place the pair of colliding molecules momentarily behaves somewhat like a more complex four-atom molecule, which has transitions that can indeed absorb and emit infrared radiation. This leads to collision-induced absorption, whose associated absorption coefficient is generally a smooth function of wavenumber. Because of the lack of line structure, such absorption is referred to as a continuum. There are many possible processes through which collisions can induce absorption. The collision can impart a temporary dipole moment to a rotation or vibration that ordinarily had none, allowing it to absorb or emit a photon. The collision can break a symmetry, allowing transitions that are otherwise ”forbidden”
by symmetry principles”
So maybe you are 10% right 🙂

foinavon
March 3, 2009 1:08 pm

Dick H. Ahles (12:03:57)

Without negative feedback some values should grow continuously. So, if there has been a proven positive relationship between rising CO2 levels and rising temperature in the past, rising CO2 levels should have caused the increase of temperature “for ever”.

It’s easy to see why that’s incorrect. An increase in forcing (e.g. due to raised greenhouse gas levels) doesn’t cause an increase of temperature “for ever”. More likely it results in an increase in temperature towards a new equilibrium temperature.
Leave a pan of water in you kitchen which is at constant temperature. The pan will settle to a pretty constant temperature. Now move the pan onto the top of your central heating boiler. A constant forcing is being applied to the pan. Does it cause an increase of temperature “for ever”? No. The temperature rises towards a new equilbrium temperature. Are negative feedbacks required to stop the “value grow(ing) continuously”. No.

George E. Smith
March 3, 2009 3:42 pm

“”” lgl (12:50:53) :
George,
From Pierrehumbert’s ClimateVol1.pdf:
“A Nitrogen molecule N2 in isolation does not interact to any significant extent with infrared light; one might think that collisions do not change this picture, as N2 has no lines to be broadened by collisions. “””
Thanks Igl; so Peter Humbug to the rescue !
Well I always wondered what the source of thermal (hot body) radiation was whether BB or “pseudo BB”, because I knew it couldn’t be an infinite sequence of line spectra and somehow have anything like that Planck spectral behavior. Don’t remember which of my books pointed out that any material body has at least electric charges, which undergo accelerations as a result of the “molecular” temperature.
You have to remember that my graduation day was 52 years ago; when there wasn’t any quantum chromodynamics; hell the “8-fold way” was the cats meow. And to add insult to injury; ALL of my High school and college textbooks went into a box that supposedly went on a boat, but a month later, it failed to come off that boat on the Manhattan docks; so I have had to go by memory mostly since then; replacing only those that had any pertinence to my work needs which wasn’t much in the high tech industry.
Maybe 90% of what I learned in school was never ever used in my daily work; so there’s some cobwebs in there I know.
I’m intrigued by that transient supermolecule concept though. The lay public doesn’t know how much stranger than fiction the truth really is; maybe that’s what keeps me going.
George

George E. Smith
March 3, 2009 4:04 pm

We are always taught that nothing can go faster than the velocity of light; well actually, I believe the pedantic version is that nothing can go faster than (c); the velocity of light in vacuo.
For example, relativistic charged particles travelling very close to (c) upon crashing into a solid media; say a Cesium Iodide crystal, suddenly find themselves travelling faster than the local velocity of light (c/n), so if for no other reason they have to slam on the brakes, and during that braking deceleration you get Bremsstrahlung radiation which IS a cointinuous spectrum of “braking radiation”, which I think is the literal translation of Bremsstrahlung. Anna v is probably an expert on this stuff; I only remember it from my days messing around with scintillation detectors; although I used Anthracene and Stilbene Crystals instead oc CsI, since I was interested in Neutron detection; and my Po/Be neutrons always came with tons of gammas. We were too poor to be able to get Pu/Be neutron sources; not to mention that unmentionable word.
But somehow, I don’t think atmospheric IR radiation can be regarded as Bremsstrahlung of a sorts.
George

hotrod
March 4, 2009 1:51 pm

This reference shows a chart of the absorption characteristics of both oxygen and water vapor in the microwave spectrum.
http://ceos.cnes.fr:8100/cdrom-98/ceos1/science/dg/dg19.htm
If emissions are equal to absorption would not oxygen excited by collisions re-radiate that energy in these peak absorption bands?
That is what I recall reading but have not been able to recall what the source was.
Larry

Joel Shore
March 4, 2009 2:06 pm

George,
If you are still in this thread, I just had a question involving what might be an amusing coincidence: I was reading a paper today for my real job on light concentrators from 1990 by G. Smestad et al and noticed that the authors acknowledged one “George E. Smith of HP” in the acknowledgements section. Of course, I know that there is likely to be more than one George E. Smith in the world, but given the circumstances seemed similar enough (someone working in industry on some sort of physics-related stuff) that I was wondering if that might actually be you!?!

R Stevenson
March 10, 2009 7:13 am

All matter including CO2 molecules at temperatures above absolute zero radiate energy according to the Stefan Boltzmann Law. IR absorbed in !5 micron band of CO2 amounts to 19% of the total available at 40 C. CO2 molecules then reradiate spherically reducing the total absorption to 9.5%. the 2.6 and 4.5 micron bands are of no significance.